<<

Public Health of Ukraine Higher State Educational Establishment of Ukraine “Ukrainian Medical Stomatological Academy” , Virology and Department

SPECIAL MICROBIOLOGY AND VIROLOGY KROK -1 TESTS

Collection of tasks fo test examination preparation in Microbiology, Virology and Immunology

ТЕСТИ «КРОК-1» З СПЕЦІАЛЬНОЇ МІКРОБІОЛОГІЇ ТА ВІРУСОЛОГІЇ

Збірник завдань з мікробіології, вірусології та імунології для підготовки до тестового іспиту

Poltava – 2013

Special microbiology and virology Krok-1 tests. Collection of tasks for test examination preparation in Microbiology, Virology and Immunology/ [Loban G.A., Hancho O.V., Polyanska V.P. et al.] - Poltava, HSEEU “UMSA”, 2013. - 131 p.

INSTRUCTION:

Each of the numbered questions or uncompleted assertions is accompanied by the variants of answers or completed assertion. Choose one answer (the completed assertion), that is the most correct in this case.

Collection was prepared by:

LOBAN G.A. is the manager of Microbiology, Virology and Immunology department, doctor of medical sciences, professor HANCHO O.V. is the candidate of biological sciences, teacher POLYANSCA V.P. is the candidate of biological sciences, associate professor FEDORCHENCO V.I. is the candidate of biological sciences, associate professor ZVYAGOLSCA I.M. is the candidate of biological sciences associate professor VASINA S.I. is the candidate of medical sciences, teacher

It is RATIFIED by protocol №7 CMC UMSA in April, 27.2011.

Collection contains to the test of task, which are intended for the current, intermediate and eventual control of knowledge of dental and medical faculty students.

2

Tести «Kрок-1» з спеціальної мікробіології та вірусології. Збірник завдань з мікробіології, вірусології та імунології для підготовки до тестового іспиту. / [Лобань Г.А., Ганчо О.В., Полянська В.П. та ін.]. – Полтава: ВДНЗУ «УМСА», 2013. – 131 с.

ІНСТРУКЦІЯ: Кожне із пронумерованих питань або незавершених тверджень супроводжується варіантами відповідей або завершеним ствердженням. Виберіть одну відповідь (завершене ствердження), що є найбільш правильною у даному випадку.

Збірник підготували :

ЛОБАНЬ Галина Андріївна - завідувачка кафедри мікробіології, вірусології та імунології, доктор медичних наук, професор ГАНЧО Ольга Валеріївна – кандидат біологічних наук, ст. викладач ПОЛЯНСЬКА Валентина Павлівна - кандидат біологічних наук, доцент ЗВЯГОЛЬСЬКА Ірина Миколаївна – кандидат біологічних наук, доцент ВАСІНА Світлана Іванівна - кандидат медичних наук, ст. викладач

ЗАТВЕРДЖЕНО протоколом №7 ЦМК УМСА від 27 квітня 2011 р.

Збірник містить тестові завдання, які призначені для поточного, проміжного і кінцевого контролю знань студентів стоматологічного факультету.

3 Gr “+” cocci, which formed grape like clusters, were revealed in milk Microbiological diagnostics of products; but it was not possible to staphylococcal infection allocate an agent of infection from 1. In surgical department arise a patient‟s organism. The most probable suspicion about hospital staphylococcal cause of disease is an accumulation in infection presence, seat of disease is a products: medical personal. Which medium is A. α- necessary to inoculate with material B. Toxic protein A from nasopharynx of department C. Endotoxin personal for defining a carrier of D. pathogenic staphylococcus? E. Erythrogenic toxin A. Endo‟s medium 5. Some children, who have eat from B. Yolk-salt agar the milk-kitchen, has been revealed C. Meat-and-peptone broth with illness, which characterized by a D. Russell‟s medium nausea, vomiting, rise in temperature. E. agar Gr “+” cocci, which formed grape like 2. Child with a diagnosis clusters, were revealed in milk “staphylococcal sepsis” was been products; but it was not possible to hospitalized. On what of medium is allocate an agent of infection from necessary to inoculate a patient‟s blood patient‟s organism. What method gives for purpose of agent revealing? the most probable result for definition A. Ploskirev‟s medium an etiology of disease? B. Meat-and-peptone agar A. Biological test C. Sugar-and-peptone broth B. Neutralization test on animals D. Buchin‟s medium C. Precipitation test in test tube E. Yolk-saline agar D. Precipitation test in gel 3. А staphylococcal infection is educed E. PHAT with as complication at 36 yeared man, who diagnosticum has a chronic active hepatitis, S.aureus 6. 26 years old man, at third day after is very sensitive to tetracycline, appendectomy, has a hyperemia and and streptomycin. What drug infiltration on a stitch course, pus is is necessary to prescribe for treatment allocated from a wound at pressing. Gr of this patient? “+” cocci formed an abnormality A. Streptomycin colonies were revealed in pus. What B. Penicillin drug is necessary to use for a local C. Tetracycline specific therapy? D. Complexly using of all drugs A. Antistaphylococcal plasma of B. Antibiotic to which the this list infection E. Any drug of this list agent is sensitive 4. Some children, who have eat from C. Staphylococcal antiphagin the milk-kitchen, has been revealed D. Fluid staphylococcal with illness, which characterized by a bacteriophage nausea, vomiting, rise in temperature. E. Staphylococcal autovaccine

4 7. Golden staphylococcus was revealed E. Tetracocci on mucous of reproductive organs of 11. There are causes of purulent young woman with 8 monthed postoperative staphylococcal nature pregnancy. What drug is most complications have become frequent in necessary to use for specific prevention surgical hospital. How it is possible to of possible postnatal complications? define a source of staphylococcal A. Local using of antiseptic infection in hospital? drugs A. Defining of phage varies B. Staphylococcal autovaccine B. Defining of haemotoxin C. Staphylococcal anatoxin C. Defining of aggression D. Staphylococcal bacteriophage enzyme E. Antibiotic to which the D. Defining of biovares infection agent is most sensitive E. Defining of antibiotic 8. Gr “+” cocci located in grumps, sensitivity gived a positive plasmacoagulase test, 12. Food toxic infection eruption was fermented mannitol in anaerobic registered in a city which has arising conditions and had a lecithinase after taking of cream pastry. A activity were revealed at time of sets of pathogenic microorganism was stomatological instruments sterility revealed in the rests. What control. What microorganism was microorganism is most possibly can revealed in this case? cause this toxic infection? A. S. pyogenes A. E. coli B. S. epidermidis B. C. perfringens C. S. saprophyticus C. S. aureus D. S. aureus D. S. typhimurium E. Corynebacterium xerosis E. S. enteriditis 9. Enteritis-ill child‟s excrements were 13. There are almost all children had emulsifying in physiological solution, gastroenteritis signs in a few hours and drop of emulsion was inoculated after cheese curd intake in the on elective media: 10% milk-saline or kindergarten. At time of bacteriological yolk-saline agar. What microorganisms examination of vomiting masses and are provided to allocate? wastes of the cheese curd the golden A. Klebsiella staphylococcus was revealed. How is B. Coliform bacillі expedient to continue a research for a C. Streptococcі specification of diagnosis? D. Staphylococcі A. Make a lysotyping of E. Enterococcі allocated 10. Microorganisms, located as a strains cluster of grapes were revealed in a pus B. Define an ability of strains to from a furuncle. What organisms were produce revealed? C. Take a research of nutrition A. Micrococci unit B. Diplococci equipment C. Staphylococci D. Define a presence of D. Streptococci antibodies at

5 ill children coagulate rabbit‟s plasma and split a E. Take allergic test mannitol in anaerobic condition were 14. The microbiological research in revealed from purulent exudates of neonatal department of maternity home patient with odontogenous phlegmona. was made in view of suspecting on a What of next microorganisms can hospital infection. The golden cause a purulent complication? staphylococcus was revealed at some A. S. epidermidis children and on some comfort items. B. S. aureus What properties of allocated cultures C. S. pyogenes give a chance to characterize their D. S. viridans origin from the general source? E. S. mutans A. Antibioticogram 18. The children disease characterized B. Chromogenesis by acute beginning, nausea, vomiting, C. Antigen structure and diarrhea has developed after cheese D. Biochemical activity intake in kindergarten. Gr “+” E. Phage type microorganisms, which located in 15. The doctor detected Gr “+” cocci, bunch-shaped clumps were revealed at what locate in bunch-shaped clumps at microscopy of dab, made of cheese and time of microscopy of investigated vomiting masses. What will be your material, taken from patient with acute following actions for etiology purulent periostitis. What definition of this food intoxication microorganisms have this eruption? morphological signs? A. In addition to apply a A. Staphylococcuses bacteriological method of B. Sarcines diagnostics C. Tetracocci B. To make the conclusion that it D. Candida is E. Streptococci staphylococcus 16. Gr “+” cocci which have a round C. In addition to take an allergic form, form S-form colonies with test golden pigment on a dense nutrient D. In addition to define medium, produce a lecithinase, antibodies in plasmocoagualase, haemolysin and blood serum ferment a mannitol in anaerobic E. In addition to define a condition were revealed at patient with phagothype purulent phlegmona of maxillofacial of staphylococcus region. What kind of microorganism is 19. The staphylococcus differs from a causes suppuration? strain, which defined at patients, but A. S. aureus has an R-plasmid was revealed from B. S. pyogenes nasopharynx of nurse at time of C. S. mutans hospital infection eruption. What sing D. S. epidermidis can take a hospital strain as a result of E. S. sanguis conjugation? 17. Gr “+”round microorganisms A. Antibiotic resistance which have a lecitinase activity, B. Enterotoxin producing

6 C. Fecundity 23. Choose microorganism caused D. Staphylokines synthesis purulent diseases of oral cavity mucous E. Formation of colonies membrane: 20. S. aureus culture was revealed as a A. , result of bacteriological examination of B. Corynebacteria, sour cream material. How to prove C. Bacteroides, Enterobacteria what the isolated colony of S. aureus is D. Candida and Aspergillus an agent of food poisoning which E. Staphylococcus, arisen at sour cream consumers group? Streptococcus A. Detection of enterotoxin 24. There is a frequency of infection- B. Detection of plasmcoagualase inflammatory diseases caused by activity Staphylococcus aureus was increased C. Detection of haemotoxin in maternity hospital. What of D. Detection of saccharolytic epidemiological method is using for properties detection of source of this infection? E. Detection of lecithinase A. Defining of phage vary activity B. Defining of serovary 21. The examination of C. Defining of colicinotype carriers has been made for estimation D. Defining of antibioticogramm of sanitary state in maternity hospital. E. Defining of biovary What microorganisms are pathogenic 25. Gr “+” cocci located by clusters in for human? preparation were isolated from patient A. Pathogenic staphylococcus with osteomyelitis of maxilla. They B. Micrococcus ferment a mannitol to acid in anaerobic C. Sarcines conditions, coagualate a plasma of D. Lactic-acid bacteria rabbit. It is necessary to classify this E. Bifidumbacteria pure culture. 22. An agent, which gives round-form A. Staphylococcus aureus yellow colonies on blood agar was B. Staphylococcus saprophyticus isolated from patient with skin C. Streptococcus pyogenes pustules. These colonies have a D. Streptococcus mutans medium size and surrounded by active E. Staphylococcus epidermidis area. Gr “+” cocci located by 26. The sanitary-microbiological ungeometrical clumps were revealed in examination of air in medical dabs from colonies. This culture is establishment must be done every 4 oxydase-positive, catalase-positive, months. Which of next microorganisms ferment mannitol and synthesize is classifyed to sanitary-representative plasmocoagualase. What pathogenic air microorganisms of enclosed space? agent is revealed? A. C. perfrignens A. Staphylococcus aureus B. E. coli B. Staphylococcus saprophyticus C. E. faecalis C. Staphylococcus epidermidis D. P. aureuginosa D. Staphylococcus agalactiae E. S. aureus E. Staphylococcus pyogenes 27. It‟s necessary to find a source of infection in surgical department of

7 stomatologic clinic in view of C. Serological identification postoperative purulent complications D. Estimation of antibiotic cases, caused by golden susceptibility staphylococcus. What examination is E. Phagotyping necessary to make among the 30. A 65-year-old man has purulent personnel? abscess on his neck. Analyses revealed A. Examination of dysbacteriosis a culture of gram-positive cocci with B. Test of staphylococcal plasmocoagulase activity. This culture immunity relates most likely to: intensity А.Staphylococcus C. Inoculation of blood to a saprophyticus sterility В. Streptococcus agalactiae D. Detection of carriage С. Streptococcus pyogenes E. Examination of outwash of D. Staphylococcus hands epidermidis 28. Examination of a patient with E. Staphylococcus aureus pustular skin lesions allowed to isolate a causative agent that forms in the blood agar roundish yellow middle- Microbiological diagnostics of sized colonies surrounded by streptococcal infection haemolysis zone. Smears from the 31.Confinement to bed patient has a colonies contain irregular-shaped rise in temperature of body, cough clusters of gram-positive cocci. The occurrence with a lot of sputum culture is oxidase- and catalase- secretion, pain in the chest. Gr“+” positive, ferments mannitol and cocci, located in pairs were revealed in synthesizes plasmocoagulase. What the sputum. What microorganism is the causative agent was isolated? most probable agent of infection? А. Staphylococcus A. Legionella pneumophilla saprophyticus B. Staphylococcus aureus В. Streptococcus agalactiae C. Klebsiellа pneumonia С. Streptococcus pyogenes D. Mycoplasma pneumonia D. Staphylococcus E. Streptococcus pneumoniае epidermidis 32.The infection of gallbladder was E. Staphylococcus aureus caused by organisms, what have an 29. In the surgical department of a oval form, is located in pairs or short hospital there was an outbreak of chains. Proceeding from results of hospital infection that showed itself in serological tests, these organisms were often postoperative wound abscesses. identified as “D”group streptococci. Bacteriological examination of pus How they are called? revealed aurococcus. What A. Enterococci examination shall be conducted to find B. Green streptococci out the source of this causative agent C. Aerococci among the department personnel? D. β-hemolytic streptococci A. Biochemical identification E. Pneumococci B. Microscopical examination

8 33. Dick‟s toxin was intravenous erythrogenic toxin injected to 20 children, who had C. Disease is not caused by contacts with scarlet fever patient in the hemolytic streptococcus kindergarten. Three children have D. The serum must be enjected tumescence and reddening in a place of intravenously injection; other children haven‟t it. E. Child‟s immunity system is Estimate the results of reaction. very A. Children with positive Dick‟s poor reaction are the vectors of hemolytic 36. The culture of coccal bacteria was streptococcus revealed from a nasopharynx of a boy B. Children with positive Dick‟s with chronic tonsillitis. They are reaction have an to toxin of located by chains in the smear. What hemolytic streptococcus microorganisms it is can be? C. Children with positive Dick‟s A. Vibrio reaction have carried a scarlet fever in B. Staphylococcus past C. Escherichia D. Children with positive Dick‟s D. Clostridia reaction have no antibodies to E. Streptococcus toxin of hemolytic 37. Blood from a patient with streptococcus suspicions of a sepsis was inoculated E. At children with positive on a sugar broth. The long chain- Dick‟s reaction will located Gr“+” cocci was detecting in occurrence symptoms of the smear, what taken from sediment. scarlet fever in few days The small, transparent, round colonies 34. After carried scarlet fever of upper with active hemolytic area were grown respiratory tracts, the child have a pain on a blood agar. What organisms is in joints during long time, which presence in patient‟s blood? amplified in autumn. What A. Streptococcus microorganism is the most probable B. Micrococcus agent of infection? C. Staphylococcus A. S. aureus D. Tetracoccus B. S. pyogenes E. Sarcina C. C. diphtheriae 38. The short chain-located, round D. B. pertussis microorganisms were revealed in a E. Flu virus smear of tonsils mucous. What 35. Doctor has suspected a scarlet fever microorganisms were revealed? at 2 yeared child with catarrhal A. Staphylococcus appearance and eruption on skin. A B. Steptococcus little dose of serum to erythrogenic C. Micrococcus toxin of streptococcus was inoculated D. Diplococcus intradermal and eruption was E. Tetracoccus disappeared in the injection place. 39. A rheumatic heart‟s disease has What means results of reaction? developed at 12 years old boy after A. Final diagnosis is confirmed carried angina. Every next B. Child has a high sensitivity to streptococcal infection caused a

9 deterioration of patient‟s condition. method is need to use for diagnosis What drug is rationality to use for confirmation? complications prevention? A. Serological A. Autovaccine B. Bacterioscopical B. Streptococcal anatoxin C. Bacteriological C. Streptococcal bacteriophage D. Allergic D. Donor‟s γ-globulin E. Biological E. Penicillin 44. At examination of 2 years old boy, 40. Streptococcus of “A” group was physician has see an eruption on neck isolated from the patient with and upper part of chest, and also tongue glomerulonephritis. What of erythema (“crimson tongue”) and give streptococcal antigens provides a a diagnosis “scarlet fever”. What typical specificity and virulence? organism is an infection agent of this A. Vi-antigen disease? B. M-protein A. Streptococcus pyogenes C. T-antigen B. D. P-antigen C. E. OF-antigen D. Mycobacteria tuberculosis 41. The pathological material (a smear E. Corynebacterium diphtheriae of tonsils) was taken from 7 years old 45. Gr “+” cocci, which have a little child who has an angina, and oblong form, located in pairs or short inoculated on blood agar. The medium chains, produce a capsule, cause an α- became transparent around colonies of hemolysis in blood agar were revealed streptococcus in next day. The presence from oral cavity of clinical-healthy 25 what of pathology factor of an years old man. A vector what of infectious agent was revealed? disease he is? A. Hemolysin A. Streptococcus salivarius B. Endotoxin B. Streptococcus pyogenes C. Neuraminidase C. Streptococcus pneumoniae D. β-lactamase D. Streptococcus feacalis E. E. Peptostreptococcus 42. Gr “+”oval located in pairs 46. Gr “+”prolonged diplococci with microorganisms were revealed at pointed opposite ends were revealed in microscopic diagnostics of patient‟s sputum of patient with suspicion on sputum. What microorganisms were pneumonia. What microorganisms revealed in sputum more probably? were revealed in smear? A. Streptococcus pneumoniae A. Staphylococcus aureus B. Neisseria meningitidis B. Streptococcus pneumoniae C. C. Klebsiella pneumoniae D. Staphylococcus aureus D. Neisseria meningitidis E. Neisseria gonorrhoeae E. Neisseria gonorrheae 43. The physician has suspected a 47. At microscopy of sputum from progress of disease at 8 years old boy, patient with pneumonia, a lot of Gr “+” who often have an angina caused by lancet shaped diplococci, which are Streptococcus. What of microbiological surrounded by capsule, were revealed.

10 Which microorganisms were revealed A. Streptococci in smear? B. Micrococci A. Klebsiella pneumoniae C. Staphylococci B. Streptococcus pneumoniae D. Tetracocci C. Chlamidia pneumoniae E. Sarcina D. Staphylococcus aureus 51. A 7 year old child often suffers E. Escherichia coli from streprococcic angina. Doctor 48. In lumps of pus, taken from sputum suspected development of rheumatism of patient with clinical diagnosis “lung and administered serological fever” the blue-violet lancet shaped examination. The provisional diagnosis diplococci, which surrounded by will be most probably confirmed by capsule, were revealed. The presence of antibodies to the following bacteriologist`s conclusion was “S. streptococcic antigen: pneumonia is in the sputum”. What A. O- method was used for confirmation of B. C-carbohydrate clinical diagnosis? C. M-protein A. Microscopical D. Erythrogenic toxin B. Bacteriological E. Capsular polysaccharide C. Serological 52. During examination of a patient a D. Express-method dentist revealed a lot of "white spots" - E. Biological zones of enamel demineralization. 49. Gr “+” cocci, which can produce a What microorganisms take part in the capsule were revealed from sputum of development of this process? patient with suspicion on lung fever. A. Streptococcus mutans The microorganisms fermented an B. Streptococcus salivarius inulin and had been lysed by bile at C. Streptococcus pyogenes identification of pure culture. What D. Veilonella parvula organisms were isolated from this E. Staphylococcus epidermidis patient? 53. There was an episode of the A. Staphylococcus aureus hospital infection caused by B. Streptococcus haemolyticus S.pyogenes in the surgical department. C. Streptococcus pneumoniae What media can be used for pathogenic D. Streptococcus viridans Streptococci isolation? E. Streptococcus pyogenes A. Endo`s media 50. Blood of a patient with presumable B. Yolk-salt agar sepsis was inoculated into sugar broth. C. MPB There appeared bottom sediment. D. Ressel`s media Repeated inoculation into blood agar E. Bloody agar caused growth of small transparent 54. The child with a diagnosis round colonies surrounded by “Streptococcal sepsis” was hemolysis zone. Examination of a hospitalized. What media can be used smear from the sediment revealed for pathogenic Streptococci isolation gram-positive cocci in form of long from blood? chains. What microorganisms are A. Yolk-salt agar present in blood of this patient? B. Endo`s media

11 C. MPB other children and personnel of the D. Bloody agar kindergarten for the revealing of the E. Saccharine broth effaced forms of meningococcal 55. A doctor has suspected scarlet disease and meningococcal carriers. fever of a 5-year-old child with What kind of diagnostic should be catarrh and skin . A small used? quantity of serum against A. Bacterioscopy of streptococci erythrogenic toxin has cerebrospinal been injected intracutaneously. In the fluid place of injection the rash B. Revealing of meningococcal disappeared. What do reaction antigen in cerebrospinal fluid results indicate? C. Serum research A. The whole doze of serum D. Cultural research of can be injected nasopharyngeal mucus intravenously. E. Skin allergic test B. The child is 58. The 5-years old child recovers after hypersensitive to acute meningococcal meningitis. When erythrogenic toxin. will it be able to visit kindergarten C. The disease has been again? caused by a nonhemolytic A. After complete clinical streptococcus. convalescence D. The clinical diagnosis has B. When serum reactions will proved to be true. become negative E. Immune system of the C. If all children in a group will child is considerably receive a vaccination against impaired. meningitis 56. There are cases of purulent D. If meningococci will not be postoperative complications of the revealed in cerebrospinal fluid streptococcal nature which have E. If the result of bacteriological become frequent in a surgical hospital. examination of nasopharyngeal How to determine a source of a mucus is negative streptococcal infection in a hospital? 59. There was examination of children A. Determination of phagovaries and personnel with the purpose of B. Determination of serovaries revealing of meningococcal carriers in C. Determination of enzymes a kindergarten. Choose the method of D. Determination of biovaries microbiological research: E. Determination of sensitiveness to A. Bacteriological antibiotics B. Allergological C. Bacterioscopical Microbiological diagnostics of D. Biological meningococcal infection E. Serological 57. Two children from kindergarten are 60. A bacteriologist at research of in hospital with a diagnosis - nasopharyngeal mucus adhered to the meningococcal meningitis. It is set rule of maintainance of aetiological necessary to conduct examination of agent in material. At bacterioscopical

12 research the presence of gram-negative A. Puncture of cerebrospinal cocci which remind coffee grains and fluid, which flows out force-feed located by pairs and tetrads was and has an unpleasant smell determined. Name an aetiological agent B. Isolating of urinoculture of which was revealed by bacteriologist. Neisseria menigitidis from A. Stаphyloсoссus аurеus excrements B. Neisseria menigitidis C. Isolating of Neisseria C. Neisseria gonorrhoeae menigitidis from sputum D. Morаxеllа lасunаtа D. Research of vomiting masses E. Асinеtobасtеr саlсoасеtiсus E. Isolating of Neisseria 61. Gr- diplococci, having the same menigitidis from the mucus form as coffee grains were revealed membrane of the urinogenital during bacterioscopical research of system nasopharyngeal mucus of 2,5-years old 64. A doctor diagnosed child with nasopharyngitis. What “meningococcal nasopharyngitis” to organs of child most probably will be the child with nasopharyngitis. What affected if these microorganisms will method of laboratory diagnostics is the get to blood? best for confirmation of diagnosis? A. Meninges A. Allergological B. Mitral valves B. Biological C. Renal glomerules C. Serological D. Urogenital tract D. Microscopical E. Lymphatic nodes E. Bacteriological 62. From a child with cerebrospinal 65. The pure culture of gram-negative meningitis a turbid cerebrospinal fluid diplococci was isolated from the child, which contains plenty of leucocytes is which visited kindergarten where a received. What serum reaction does it case of meningococcal infection took cost to take advantage of for express- place. Which facts do testify that this diagnostic of disease? microorganism is Neisseria A. Precipitation reaction menigitidis? B. reaction A. Does not grow at 300C on the C. Complement binding assay serum agar D. reaction B. Grows at 370C on the serum E. Neutralization reaction agar 63. A 5-years old patient complains of C. Creates pigment great headache and vomiting. D. Grows on MPA Objectively: rigidity of the occipital E. Ferments saccharose muscles, continual vomiting, nausea, 66. The sick child`s diagnosis was herpetic rash on face, fever. What “epidemic cerebrospinal meningitis” on pathological material is needed to use the basis of the conducted inspection for bacteriological confirmation of the and using of microbiological preliminary diagnosis - cerebrospinal examination. What microorganism meningitis? caused this disease? A. Neisseria gonorroeae B. Stаphyloсoссus аurеus

13 C. Strеptoсoссus pyogenes cerebrospinal liquid of the patient with D. Neisseria menigitidis meningitis. What serum reaction must E. Psеudomonas aeruginosa be used for determination of serum 67. Gram-negative diplococci located group? in leucocytes were revealed in the A. Agglutination reaction smear made from cerebrospinal liquid B. Precipitation reaction of patient with festering meningitis. C. Indirect hemagglutination test What does provide the uncompleteness D. Complement binding assay of Neisseria menigitidis phagocytosis? E. Immune sorbent assay A. Capsule polysaccharide 71. The bacteriologist revealed small antigens located in pairs bean-shaped B. Capsule protein antigens microorganisms in a smear from C. Peptidoglycanes of the cell nasopharyngeal mucus. They became wall pink after Gram staining. Name these D. Proteins of the cell wall bacteria. E. Enzymes of pathogenicity A. Streptococcus 68. There is a child with diagnosis B. Staphylococcus “meningococcal nasopharyngitis” in C. Neisseria the kindergarten. What vaccine can be D. Enterococcus used with the purpose of emergency E. Micrococcus specific prophylaxis of meningococcal 72. From a child with cerebrospinal infection for contact children? meningitis there was obtained muddy A. Anatoxin spinal fluid with numerous leukocytes. B. Alive attenuated Which test should be used for express- C. From killed microorganisms diagnostics of the disease? D. Chemical a. . E. Combined B. Agglutination. 69. The young woman `s temperature C. test. suddenly rose to 39°C and great D. Hemagglutination. headache has appeared. At a review E. Neutralization. rigidity of the occipital muscles is determined. Spinal puncture was Microbiological diagnostics of performed. There are huge amount of gonococcal infection neutrophils and gram-negative 73. Bacteriological examination of diplococci staned by Gram in a smear purulent discharges from the urethra from a cerebrospinal liquid. Which of revealed the following bacteria could be the gram-negative bacteria looking like reason of this illness? coffee beans. They were localized in A. Stаphyloсoссus аurеus the leukocytes and could decompose B. Neisseria menigitidis glucose and maltose to acid. These are C. Hаеmophilus influеnzае the causative agents of the following D. Streptococcus pneumoniae disease: E. Psеudomonas aeruginosa A. 70. The culture of meningococci was B. Venereal defined in the sediment of lymphogranulomatosis

14 C. Gonorrhoea D. Destroys the synthesis of D. cytoplasmic membrane E. Trychomonadosis E. Destroys adhesines 74. A doctor-ophthalmologist 77. During the bacteriological examined a sick child with the examination of the patient with phenomena of purulent cerato- suspicion on a gonorrhoea growth of conjuctivitis and supposed that it is a microorganisms on special nutrient blenorrhea. What methods of mediums was not revealed. What laboratory diagnostics can be used to methods of diagnostics need to be used confirm the diagnosis? to confirm or exclude the preliminary A. Microscopic and diagnosis? bacteriological A. Precipitation test B. Microscopic and biological B. Use the biological method C. Microscopic and phage C. To conduct microscopic diagnostics research D. Bacteriological and biological D. Use IFT E. Phage diagnostics and allergic E. Use CBT, allergic test 75. Mucous consistency, transparent, 78. 28 year-old woman, who several even-edged protuberant colonies have months ago suffered from an acute grown on the special media after the and treated herself pus from an urethra inoculation. Gram- independently by penicillin, has an negative, bean-shaped diplococci were inflammation of appendages of uterus. found during the microscopy of The reaction Borde-Gangou is positive. preparations from these colonies. What What other organs and systems can be kind of disease can they cause? affected, if the course of medical A. Chlamidiosis treatment is not conducted? B. Gonorrhоea A. Joints C. Syphilis B. Liver D. Rabbit-fever C. Vessels E. D. Immune system 76. A pus from a patient‟s urethra with E. Central nervous system suspicion of gonorrhea, that 79. Gramnegative bean-shaped independently treated himself by diplococcі inside and outside of peniсillin, was taken for research. leucocytes were detected on During the research the L-shaped bacteriological examination of the gonococci were found. How does purulent exudates from the cervix of penicillin influence on gonococci? the uterus. Name the causative agent of A. Destroys the synthesis of purulent inflammation of the cervix of cellular the uterus. wall A. Chlamidia trachomatis B. Destroys the synthesis of B. Neisseria gonorroeae albumens C. Haemophilus vaginalis C. Destroys the synthesis of D. Trichomonas vaginalis amino E.Calymmatobacterium acid granulomatis

15 80. During the bacteriological tests of outside of leucocytes were detected on the purulent secreta from urethra there bacteriological examination of the were found bacteria, which according purulent exudates from the cervix of to Gram were negatively staining, the uterus. What results of process are looked like coffee beans. These observed in a preparation? bacteria were splitting glucose and A. Capsuleforming maltose to acid, they were located B. Metabolism inside the leucocytes. Name these C. Phagocytosis microorganisms. D. Sporeforming A. Neisseria meningitides E. Malignisation B. Neisseria gonorrhoeae 84. The sick woman has the clinical C. Staphylococcus aureus diagnosis "gonorrhoea". What from D. Streptococcus pyogenes listed below researches can be applied E. Enterococcus faecalis to diagnosis acknowledgement? 81. A doctor made the diagnosis of A. Microscopy of pathological gonorrhoea to the young man. It was material known from the anamnesis that a B. Infection of laboratory patient had had gonorrhoea before and animals he had been treated completely. What C. Test with phages type of infection can this new disease D. Hemagglutination assay be attributed to? E. Reaction of immobilisation A. Superinfection 85. A patient had delivered to the B. Reinfection hospital with previous diagnosis C. Secondary infection “chronic gonorrhea”. What type of the D. Relapse double-system serological reaction can E. Autoinfection be used for specific antibodies 82. A patient who came to the doctor revealing in the serum? because of his infertility was A. Agglutination test administered to make tests for B. Neutralization test toxoplasmosis and chronic gonorrhoea. C. Complement‟s binding test Which reaction should be performed to D. Radio-immune analysis reveal latent toxoplasmosis and chronic E. Immunoenzyme analysis gonorrhoea in this patient? 86. Gramnegative bin-shaped A. RIHA - Reverse indirect diplococcus inside and outside of hemagglutination assay leucocytes were detected on B. (R)CFT- Reiter's complement bacteriological examination of the fixation test purulent exudates from the cervix of C. RDHA - Reverse direct the uterus. Name the causative agent of hemagglutination assay purulent inflammation of the cervix of D. Immunoblot analysis the uterus. E. IFA - Immunofluorescence A.Calymmatobacterium assay granulomatis 83. The patient has addressed to the B. Chlamidia trachomatis doctor-dermatologist. Gramnegative C. Haemophilus vaginalis bean-shaped diplococcі inside and D. Trichomonas vaginalis

16 E. Neisseria gonorroeae rooms a colon bacillus was also 87. Clinical diagnosis of a female revealed. What research should be patient was gonorrhoea. What made for identification of strains? examination method can be applied for A. Morphological properties confirmation of this diagnosis? studying a. Infection of laboratory B. Cultural properties studying animals C. Biochemical properties B. Microscopy of pathological studying material D. Pathogenicity tests C. Test with bacteriophage E. Colicinotyping D. Hemagglutination reaction 91. 18 yeared patient has general E. Immobilization reaction weakness, dizziness, nausea, vomiting, diarrhea ( defecation up Microbiological diagnostics of to a 10 time per a day). A doctor escherichiosis made a previous diagnosis 88. E. coli culture of O-111 serotype "". However at the was revealed in the excrements of a bacteriological research of vomiting sick child, who had been given masses and excrements Shigella artificial food. Which diagnosis is was not found. On Endo‟s media the correct? colonies of raspberry color with A. Coli-enteritis metallic glitter have grown. Which B. Gastroenteritis of diagnostic serums the positive C. -like diseases reaction of agglutination of the D. Food poisoning isolated culture is the most E. Disentery-like diseases possible? 89. During bacteria conjugation A. Polyvalent OK-serum experiment the next strains were used: B. O- serum 1- streptomycin resistant auxotroph by C. Salmonellas N- serum lysine (F+) and 2 – streptomycin D. The O-cholera serum sensitive prototroph (F-). What media E. Serum for Yesinia, serotype should be taken for isolation of 03 recombinants? 92. The child with coli-enteritis has A. With lysine and streptomycin arrived to the infectious hospital. Colon B. With lysine without bacilla was isolated from excrements. streptomycin How to establish the bacilla accessory C. Without lysine, but with to pathogenic variants? streptomycin A. In reaction of agglutination D. Without lysine and with streptomycin O-serum E. With factors F+ and F - B. On the basis of biochemical 90. The causes of meningitis which properties were caused by a colon bacillus C. By phage typing were noted in the department for D. Microscopy of the stained new-borns. At the sanitary- smears bacteriological investigation of E. Due to growth on Endo media

17 93. 55 yeared man was hospitalized to establish the pathogenicity of the colon a surgical clinic with suspicion on a bacilla? sepsis. What material must be taken A. Due to antigenic properties. from patient for research and what B. Due to morphological media it should be used? properties. A. Blood, saccharine broth C. Due to cultural properties. B. Liquor, serum agar D. Due to toxigenic properties. C. Urine, meat peptone agar E. Due to phage sensitiveness D. Pus, yolk-salt agar 97. The 6-monthed child was entered to Е. Lymphatic node puncture, the infectious hospital with diagnosis cysteine agar “acute gastroenteritis”. On what 94.7 yeared boy has the cholerae-like nutrient medium it is necessary to sow disease (vomiting, diarrhoea). The faeces of ill child? same crimson colour colonies with A. Moncur‟s medium metal shine have grown on Endo media B. Blood agar after excrements inoculation. What C. Endo medium microorganism is the most possible D. Ploskirev‟s medium agent of disease? E. Serum agar A. 98. From the defecation of a 6-year-old B. enteritidis ill child, who has artificial feeding, the C. Escherichia coli intestinal bacillus with antigen D. structure 0-111 is excreted. What is the E. Vibrio diagnosis? 95.On bacteriological examination of A Food poisoning the defecation of a 4-months-old baby B Gastroenteritis with the symptoms of acute bowel C Cholera-like diseasis infection there were revealed red D Coli-enteritis colonies spread in the large quantity in E Disentery-like diseasis the Endo medium. What 99. Sanitary bacteriological research on microorganism can it be? water by the membrane filter method A. Escherichia revealed two red colonies on a B. Salmonella membrane filter () through C. Staphylococcus which 500 ml of analyzed water were D. Streptococcus passed. Calculate the coli index and E. Shigella coli titer of the analyzed water: 96. The 4 yeared child has arrived to A 2 and 500 the infectious clinic with the B 4 and 250 preliminary diagnosis “coli-enteritis”. C 250 and 4 Symptoms of the general intoxication, D 500 and 2 vomiting and diarrhoea were observed. E 250 and 2 At bacteriological research of 100. As a result of durative antibiotic excrements the red colour colonies therapy a 37-year old patient developed with metal shine have grown on Endo intestinal dysbacteriosis. What type of media. They were agglutinated by drugs should be used in order to polyvalent ОК-serum on slide. How to normalize intestinal microflora?

18 A Vitamins diagnosis is the best to confirm the B Sulfanilamides diagnosis? C Bacteriophages A Detachment of myeloculture D Autovaccines B Detachment of blood culture E Eubiotics C Detachment of feces culture 101. Among junior children of an D Detachment of urine culture orphanage an outbreak of intestinal E Detachment of pure culture infection with signs of colienteritis 104. On bacteriological study of was registered. In order to identify rinsing water of the patient with food isolated causative agent it is necessary poisoning, the pure bacterial culture to: was inoculated with the following A Study antigenic properties of the properties: gram-negative motile causative agent bacillus in the Endo environment B To determine sensitivity to grows like achromic colony. antibiotics Representative of what genus has C To study sensitivity to caused the illness? bacteriophages A Yersinia D To study biochemical properties of B Shigella the causative agent C Salmonella E To study virulence of the causative D Escherichia agent E Citrobacter 105. Effective diagnostics of the Microbiological diagnostics of intestinal infections agents is based on Salmonella infection antibodies to bacteria antigens 102. During the repeated Widal's revealing in indirect hemagglutination agglutination test it was noticed that the test. What standard preparation must be ratio of titers and O-antigens used in this reaction? {S.typhi} in the patient's serum had A. Erythrocyte diagnosticum with increased from 1:100 to 1:400. How adsorbed antigens of bacteria would you interpret these results? B. Antibodies against immunoglobulins A The patient has of the main classes B The patient is an acute carrier of C. Monoclone antibodies typhoid microbes D. Mono-receptor diagnostic serums C The patient is a chronic carrier of E. Ram‟s erythrocytes and hemolytic typhoid microbes serum D The patient previously had typhoid 106. Reaction of passive fever hemagglutination conducted with E The patient was previously erythrocytic typhoid Vi-diagnosticum vaccinated against typhoid fever helped to reveal some antibodies in the 103. A patient with complaints of 3- dilution of the patient's serum at a ratio day-long fever, general weakness, loss of 1:80 that exceeds the diagnostic of appetite came to visit the titer. Such result witnesses of: infectionist. The doctor suspected A Being ill with acute typhoid fever enteric fever. Which method of B Being a potential carrier of typhoid laboratory bacilli

19 C Typhoid fever recurrence specific adsorbed agglutinative serum D Incubation period of typhoid fever was chosen for final identification. E Reconvalescence of a patient ill with What type of agglutination test typhoid fever should be used? 107. A 50-year-old patient with typhoid A. Reaction of slide agglutination fever was treated with Levomycetin, B. Reaction of agglutination (variant of the next day his condition became Widal) worse, temperature rised to 39,60С. C. Reaction of agglutination (variant of What caused worthening? Gruber) A Reinfection D. Reaction of hemagglutination B Allergic reaction E. Reaction of passive C Irresponsiveness of an agent to the hemagglutination levomycetin 111. Serological diagnostics of D Secondary infection addition thyphoid is found on specific E The effect of endotoxin agent interaction of antigens and antibodies. 108. The pathogen was isolated from How reaction is called when patient‟s organism (patient suffers from electrolyte‟s presence microorganisms acute gastroenteritis). It must be are stuck together under a specific identified by antigen structure. What antibodies‟ influence? serologic reaction must be used? A. Reaction of precipitation A. Reaction of agglutination B. Reaction of agglutination B. Reaction of complement‟s binding C. Reaction of complement‟s fixation C. Reaction of neutralization D. Reaction of hemadsorption D. Reaction of precipitation E. Reaction of neutralization E. Reaction of opsonization 112. Serological diagnostics of 109. For serologic diagnostics of infectious diseases is found on specific infectious disease patient‟s blood was interaction of antigens and antibodies. delivered for analysis. Blood sera was How reaction is called when highly dissolved by isotonic solution. Patient‟s dispersive antigens are adsorbed on serum was used for reaction in erythrocytes? dilutions from 1:10 to 1:1280 and A. Reaction of neutralization erythrocytes, which were sensitized by B. Reaction of precipitation microbe antigen. What serologic C. Reaction of complement‟s fixation reaction was used? D. Reaction of hemadsorption A. Direct agglutination E. Reaction of indirect (passive) B. Precipitation hemagglutination C. Passive hemagglutination 113. The pure culture of bacteria was D. Coomb`s isolated at bacteriological research of E. Opsonization patient with the food poisoning. It has 110. The pure culture of bacteria was such properties: gramnegative mobile isolated from a pathological material. It rods on an Endo‟s media forms the was partially identified by colourless colonies. Name their genus. morphological, tinсthorial, cultural and A. Shigella biochemical properties of bacteria in B. Iersinia bacteriological laboratory. The type- C. Esherichia

20 D. Citrobacter A. Formation of colourless E. Salmonella colonies on Bismuth agar 114. For the serological diagnostics of B. Formation of red colonies typhoid the reaction, when to a with metallic brilliance on the different solubilisations of patient‟s Endo‟s media sera the diagnosticums of three types of C. Formation of colourless microorganisms are added is used, and colonies on an Endo‟s and the result of which is estimated on the Ploskirev‟s media presence of sediment from agglutinated D. Formation of hemolysis on bacteria. This reaction name is: bloody A. Ascoli agar B. Borde-Gangou E. Formation of tender tape on C. Wasserman alkaline pepton water D. Wright 118. A laboratory got the blood of E. Widal patient with typhoid (15th day of 115. Bacteriological examination of a illness) for rising of agglutination patient with food poisoning required reaction. Indicate a reagent, which inoculation of a pure culture of would give a positive reaction. bacteria with the following properties: A. The typhoid H-diagnosticum gram-negative movable bacillus that B. Typhoid O-diagnosticum gro on the Levin's medium in form of C. Diagnosticum typhoid with colourless colonies. A representative of Vi- which species corpuscules caused this disease? D. Sera to H-antigens of S. typhi A Esherichia E. Sera to O-antigens of S. B Shigella typhi C Iersinia 119. Patient with a suspicion on D Salmonella typhoid was sent to the E Citrobacter bacteriological research of blood 116. 55-year-old patient with typhoid by the doctor. What is observed fever was treated by chloramphenicol. during the first weeks of typhoid- The next day the patient's condition paratyphoid? deteriorated, the temperature rose to A. Septicopyemia 39.6 C. How do you explain the B. Bacteriemia deterioration condition of the patient? C. Toxinemia A. Allergic reaction D. Virusemia B. The effect of endotoxin E. Septicemia C. Irresponsiveness of an agent to 120. Tо infectious department of antibiotic hospital a person of 37 years old D. Secondary infection addition with the clinical signs of typhoid E. Reinfection was admitted. Тerm of disease is 5 117. From the patient`s blood the days. Bacteriological research of culture of typhoid agent was patient`s feces gave a negative isolated. What are the cultural result. What method of diagnostics characteristics of this bacteria?

21 will turn out most expedient for E. A patient has no gall-bladder clarification of clinical diagnosis? 123. Patient came to doctor-infectionist A. Bacteriological research of with complaints to 3 days long fever, blood general weakness, worsening of B. Bacterioscopical research of appetite. Doctor supposed that it was blood enteric fever. Which method of C. Serological research of blood laboratory diagnosis is the best to D. Bacteriological research of confirm the diagnosis? urine A. Detachment of blood culture E. Bacteriological research of B. Detachment of myeloculture bile C. Detachment of feces culture 121. A woman of 32 years old has a D. Detachment of urine culture fever, a headache during a week , E. Detachment of pure culture she is freezing, a liver and a spleen 124. Patient was admitted to the are megascopic, the elements of infectious clinic with a previous roseol rash are noticeable on the diagnosis “typhoid”. He feels sick skin of stomach. A doctor during three days. What method will diagnosed "typhoid". What give possibility to make a diagnosis? pathological material is necessary to A. Isolation of haemoculture send for clarification of diagnosis? B. Isolation of urineculture A. Sera of blood C. Rising of Widal`s reaction B. Stroke of blood from a finger D. Biological method C. Blood from an elbow vein, 10 E. Isolation of coproculture ml 125. Agglutination test was used for D. Bile serologic diagnostics of typhoid. What E. Feces component need to be used for this 122. A man of 57 years old, who has reaction except patient serum? typhoid for 2 years, rejects to pass A. Anatoxin the inspection of carrying the B. Diagnostic serum bacteria. He points that there is no C. Complement way he can be a transmitter of D. Hemolytic system Salmonella . A doctor revised E. Diagnosticum his medical card and agreed with 126. Inactivated typhoid vaccine was him. What is the conclusion based checked on rabbits and shows such on? results: antibodies titer before A. A man had the easy form of immunization was 1:5 and after typhoid immunization was 1:5. How it is B. A man got the inoculation by possible to explain it? TABte vaccine A. Titer of antibodies considerably C. Typhoid carrying of bacteria increases only at the secondary proceeds for half-year immune response D. A patient had is promoted B. Input of a typhoid vaccine forms the acidity cellular immunity, instead of humoral of gastric juice one

22 C. Presence of natural specific the infectious department of the immunity interferes with formation of hospital, duration of disease is 5 days. postvaccinal immunity Bacteriological examination of patient D. During the course of preparation the feces gave a negative result. What vaccine has lost immunogenic method of diagnosis would be more properties appropriate to clarify the clinical E. It is necessary to inoculate a vaccine diagnosis? together with adjuvant for more active A. The bacteriological examination of immunity formation blood 127. Special antibodies were appeared B. Bacterioscopic study of blood in the patient with typhoid fever on the C. Serological study of blood second week of illness. What are the D. Bacteriological study of urine mechanisms of their protective action? culture A. Opsonization, complement E. Bacteriological study of bile activation 131. A 32-yeared woman has a high B. The neutralization of fever, severe headache, fever, liver and C. Activation of T-killers spleen were enlarged and visible D. Activation of B-lymphocytes elements roseola rash on the skin of the E. Activation of T-suppressors abdomen during the week. The doctor 128. Reaction of passive diagnosed “typhoid fever?”. What hemagglutination conducted with pathological material should be sent for erythrocytic typhoid Vi-diagnosticum examination to clarify the diagnosis? was used to reveal some antibodies in A. Feces the dilution of the food business B. Blood smear from a finger workers's serum at a ratio of 1:80 C. Serum during planned survey. Is this man a D. Bile carrier? E. Blood from the cubital vein,10 ml. A.Yes, he is 132. The patient was arrived to the B. He is reconvalescent hospital on the eighth day with C. This is disease complaints of headache and weakness. D. He is not a carrier Blood was taken for serological E. He was vaccinated examination. Widal agglutination 129. In the area where the expected showed positive result in a dilution outbreak of typhoid fever, it is 1:200 with typhoid O-diagnosticum. necessary to carry out school children What is the diagnosis can be made on prevention. Which drug is more the appropriate to use? basis of this study? A. TABte vaccine A. Tuberculosis B. Vi-antigen riched typhoid vaccine B. Dysentery C. The typhoidal polyvalent C. Cholera bacteriophage D. D. Donor gammaglobulin E. Typhoid fever E. The correct antibiotic 133. After inoculation of feces 130. 37 yeared man with clinical specimen from a patient with typhoid symptoms of typhoid fever delivered to fever onto Endo media colonies of

23 different size and colour – big red and C. After 12 hours medium colourless – have grown. D. After a week Name the functional type of this media. E. From the onset of the disease A. Special 138. The pure culture of bacteria B. Elective isolated from a patient with suspected C. Differential diagnostic typhoid blood is studied in the D. Selected bacteriological laboratory. What E. Enriched serological tests should be used to 134. Widal agglutination test showed study the antigenic structure? patients serum antibodies titer to A. Precipitation S.typhi Vi-antigens rise from 1:100 to B. Agglutination 1:1800. How can we explain these C. CBT results? D. ELISA A. Has typhoid E. Flocculation B. An acute typhoid bacteria carrier 139. An outbreak of food poisoning C. A chronic carrier of typhoid bacteria associated with consumption of D. Had typhoid confectionery products which are E. Previously been vaccinated against stored at room temperature and typhoid manufactured from duck eggs was 135. In connection with the outbreak of registered. Which organisms can cause typhoid has become necessary to a poisoning? survey employees of a café. What A. E. coli reaction is used to diagnose the carrier? B. Salmonella A. The reverse hemagglutination C. Staphylococci B. The indirect hemagglutination D. Legionella C. Latex agglutination E. D. The inhibition of hemagglutination 140. The gramnegative medium size E. Vi-hemagglutination motile rods with rounded ends which 136. There are sporadic cases of are agglutinated with salmonella group typhoid fever in the city A. What B O-serum were revealed in patient at method of microbiological diagnosis bacteriological study of vomiting mass. can not be used to identify people The identical organisms were revealed infected with S. typhi? in meat the salad which recently was A. Biological eaten by all patients. A pathogen of B. Bacteriological which diseas is it in this case? C. Serological A. Salmonella-agent of acute D. Allergotest gastroenteritis 137. A patient`s blood with a B. Salmonella pathogen of typhoid preliminary diagnosis "Typhoid fever" C. Salmonella paratyphi A pathogen is studied in the laboratory. After D. Esherhii causing foodborne which time, since the serological E. Proteus foodborne pathogen diagnostics method of infectious 141. The boy, aged 12, remains in diseases can be most effective? hospital with suspected food poisoning. A. One month after A large number of colorless colonies B. After 3 days were grew on Endo medium after

24 patient faeces inoculation. Which agglutination organisms can most likely be E. The reaction of flocculation eliminated with the number of possible 145. Widal's agglutination test was causative agents of disease? used for patient with suspected typhoid A. after 2 weeks from the beginning of B. Salmonella enteritidis disease. The reaction was negative. C. What can be the reason of negative D. Escherichia coli result? E. Yersinia entercolitica A. Presence of pathogen in an intestine 142. The majority of guests who B. Presence of pathogen in a blood attended the banquet had vomiting, C. Presence of antibodies in a blood abdominal pain, diarrhea and fever D. Absense of typhoid in a patient after 12 hours. The anamnesis revealed E. Presence of pathogen in bilious that all of them ate meat salad, pickled 146. Inspection of school dining-room cucumbers, zucchini, potatoes, biscuits staff for typhoid carrying has been with cream and fruit juices. Which of carried out. There were revealed organisms can be identified by antibodies to Vi-antigen in the blood bacteriological examination of material serum of the cook. Which test has from the patients? been used? A. S.enteritidis A. Complement fixation test. B. C.botulinum B. Widal's test. C. S.aureus C. Indirect hemagglutination test. D. E.coli D. Enzyme . E. C.perfringens E. Immunofluorescence test. 143. The pure culture of bacteria 147. Identification of food toxical isolated from a patient with food infection causative agent has shown poisoning. It was identified as that according to biochemical Salmonella enteridis. In this case we properties it belongs to Salmonella are talking about: genus. What property of the causative A. Serotype agent testifies its specific belonging? B. Species A. Pathogenicity for laboratory C. Genus animals. D. Hemovary B. Phagotype. E. Biovary C. Culture properties. 144. The serological test was used for D. Antigenic structure. bacterial culture seroindentification in E. Morphologic and staining the bacteriological laboratory with the properties. material from a patient with suspected 148. Feces of a restaurant cook salmonella gastroenteritis. What type without any clinical symptoms of of reaction can be supplied with this disease are examined with the help of antigen? bacteriological methods. On A. The reaction of neutralization bismuth-sulfite agar little black B. Agglutination colonies with metallic sheen have C. The reaction of precipitation grown. What microorganisms can it D. The reaction of indirect be?

25 A. Salmonellae. frequent vicious urges on defecation. B. Shigellae. Indicate the most credible pathogen: C. Escherichiae. A. Escherishia coli D. Staphylococci. B. Salmonella enteritidis E. Streptococci. C. D. Entamoeba dysenteriae E. Yersinia enterocolitica Microbiological diagnostics of 152. The bacteriological method must be used to identify sources of infection 149. Sh.sonnei were isolated from during outbreaks of shigellosis. The feces of patient. Which additional research yielded no result. It was researches must be used for decided to use phagediagnostics. Such establishment of the source of research includes: infection.? A. Determination of phagocytes А. To applay the phagotyping of activity of examined blood isolated pure culture B. Detection of bacteriophages in В. To applay the antibioticogramm pathological material С. To applay the reaction of C. Determination of phage type agglutination RA D. Performances bacteriophage titer D. To applay the reaction of indirect increase reaction hemagglutination RIHA E. Detection of functional E. To applay immunofluorescence abnormalities in the digestive system reaction IFR 153. To resolve the issue retrospective 150. The culture of Shigella was diagnosis of bacterial dysentery was isolated in patient with dysentery. It transferred assigned to serological was established, for what phages it was study of blood serum to determine sensible to. However the using of these antibody titer to Shigella. Which of phages as the method of specific these reactions is appropriate for this? therapy turned out vain. What is the A. Passive hemagglutination most credible reason of failure? B. Complement fixation A. A pathogen can live inside the cells C. Precipitation B. In the humans‟ organism a D. Hemolysis pathogen does not form receptors for E. Bacteriolysis attachment of bacteriophage 154. Shigella with ability to produce C. Used phages are moderate exotoxins were isolated from a patient D. An infectious process is supported diagnosed with dysentery. What due to mutants species of Shigella talking about? E. In humans‟ organism phages are A. inactivated B. Shigella sonnei 151. In nursery school the flash of C. Shigella flexneri acute intestinal infection was D. Shigella dysentery registered. Children, who fell ill, E. Shigella New Castle suffered of diarrea, feces with mucus 155. In the infectious ward hospitalized and blood, a temperature was promoted patient complaining of nausea, liquid to 380С. They have stomach-ache, emptying with slime and blood streaks,

26 fever, weakness. The doctor suspected B. Carry out agglutination test with dysentery. What is the most expedient serum of the patient method of laboratory diagnosis to C. Use indirect hemagglutination confirm reaction the diagnosis? D. Carry out the reaction of molecular A.Bacteriological hybridization B.Serological E. Find thermostable antigens in ring C.Mycological precipitation test D.Microscopic 159. In a patient who became ill 3 days E. Protozoological ago and complained of fever (38 C), 156. In patients with symptoms of abdominal pain, frequent bowel colitis pure culture of bacteria, which movements, presence of blood in the on morphological, culturing and stool, a physician clinically diagnosed biochemical properties attributed to the bacterial dysentery. What method of genus Shigella were isolated. Which of microbiological diagnostics should be these reactions should apply for the apply in this case and what material serological identification? should be taken from a patient? A. Agglutination with diagnostic A. Bacterioscopic - blood serums B. Bacterioscopic - excrements B. Complement fixation C. Bacteriological - excrements C. Indirect hemagglutination D. Bacteriological - urine D. Precipitation E. Antibodies - blood E. Delays hemagglutination 160. In 4 days 10 children fell ill with 157. The patient was taken to the clinical signs of acute intestinal hospital with complaints of headache, infection in different age groups of fever, frequent bowel movements, kindergarten. Sonnei dysentery agent abdominal pain with tenesmus. The was revealed in feces of these patients. doctor made a clinical diagnosis of Due to the unfavorable epidemiological "dysentery" and sent pathmaterial situation in this children's group (excrement) in the bacteriological contact among children is necessary to laboratory. Which method for conduct prevention activities. Which diagnosing physician-bacteriologist drug for specific prevention should be was to confirm or refute the diagnosis? prescribed to children that were in A. Allergotest contact with these patients? B. Biological A. Dyzentery bacteriophage C. Bacterioscopic B. Antibiotics D. Serologically C. Sulfanilamides E. Bacteriological D. The vaccine TABte 158. Pure culture of dysentery agent E. Immunoglobulin was isolated from patient in the 161. The patient recovered after laboratory. What research should be dysentery Sonnei and reinfected by the carried out for final serological same pathogen. As this infection is identification? called? A. Use agglutination test with the A. Chronic infection standard serums B. Relapse

27 C. Superinfection A. D. Persistent infection B. Cholera E. Reinfection C. Crohn's disease 162. Shigella were not revealed in D. feces of patient with typical clinical E. Dysentery signеs of dysentery due to early use of 166. The culture of microorganisms, antibiotics. Antishigella antibodies titer which were isolated from a patient was increased 4 times in the reaction of brought in the conjuctival sack of direct hemagglutination in paired sera guinea-pig with the purpose of of this patient. What this means? diagnostics of bacterial dysentery by a A. Confirms the diagnosis of dysentery biological method. A result turned out B. Excludes the diagnosis of dysentery positive – conjunctivitis was developed C. Previously suffered dysentery in an animal. The presence of what D. Pseudoreaction factor of pathogenicity of dysenteric E. Vaccination reaction bacteria was exposed by this way? 163. Shigella Sonnei. was isolated from A. Enterotoxin patient`s defecation with intestinal B. Cytotoxin infection. Which of the following C. Endotoxin serological tests were used to identify D. Capsule an isolated pure culture? E. Pili A. Agglutination test 167. Colitis was exposed in the time of B. The reaction of precipitation dissection of the deceased child, who C. Fixation of complement suffered of diarrhoea. In the smear- D. Neutralization reaction imprint of mucus membrane gram- E. Lysis reaction negative bacteria were revealed. What 164. Phage prevention of shigellosis is the most credible diagnosis? must be done to group of students were A. Staphylococcus intestinal in contact with patients during infection outbreaks. What mechanism will B. Cholera ensure their protection? C. Salmonellosis A. Phage typing D. Dysentery B. Phagocytosis E. Typhoid C. Phage lysis 168. For the purpose of retrospective D. Pinocytosis diagnostics of recent bacterial E. Small diffusion dysentery it was decided to perform 165. Multiple brownish-green layers, serological examination of blood serum haemorrhage: the gut lumen mucus, a in order to determine antibody titer small amount of blood were revealed in towards Shiga bacilli. What of the the mucosa of rectum and sigmoid following reactions should be applied? colon of the 46 yeared man body after A Bacteriolysis opening, histologically - fibrinous B Bordet-Gengou test colitis. S. sonne was revealed at C Precipitation bacteriological examination of the D Hemolysis intestinal contents. What is the most E Passive hemagglutination likely diagnosis? 169. A patient recovered after Boyd

28 dysentery and was once more infected A. Nonspecific ulcerous colitis with the same causative agent. What B. Salmonellosis such infection form is called? C. Crohn's disease A Recidivation D. Yersiniosis B Reinfection E. Dysentery C Superinfection D Persisting infection E Chronic infection Microbiological diagnostics of 170. The pathogen of dysentery was cholerae isolated in the child with an acute 173. In the feces of a patient with acute intestinal infection. What gastroenteritis there were revealed morphological and tinctorial signs motile, slightly curved Gram-negative are characteristic for this rods, which grow onto alkaline 1% pathogene? peptone water in the form of tender A. Gram-negative, immobile bluish pellicle. What microorganisms monobacteria have such properties? B. Gram-negative, mobile A. Spirilla. monobacteria B. Spirochetes. C. Gram-positive, monobacilli C. Clostridia. D. Gram-positive, streptobacilli D. Bacilli. E. Gram-negative, vibrios E. Vibrios. 171. At the bacteriological laboratory a 174. As a result of feces inoculation request for acquisition of preparations onto 1% alkaline peptone water after 8 for diagnostics of intestinal infections hours' incubation with 37° C is aquared. Which of the preparations temperature a growth in the form of adopted below was included to the list tender bluish pellicle was revealed. uncorrectly? Microscopy revealed Gram-negative A. A serum choleraic O1 curved rods. What disease could these B. A phage choleraic El-Tor microorganisms cause? C. A luminiscent typhoid serum A. Shigellosis. D. Dysenteric polyvalent phage B. . E. Erythrocyte diagnosticum with C. Typhoid fever. Shigella flexneri D. . 172. A 71-year-old man had been E. Cholera. presenting with diarrhea for 10 days. 175. A patient is hospitalized to an The feces had admixtures of blood and infectious department with cholera mucus. He was delivered to a hospital suspected. What basic method of in grave condition and died 2 days research is necessary to use for the later. Autopsy of the body revealed the confirmation of the diagnosis? following diphtheritic colitis with A. Immunological. multiple irregularly-shaped ulcers of B. Bacteriological. different depth in both sigmoid colon C. Biological. and rectus. Bacteriological analysis D. Serologic. revealed Shigella.What was the main E. Allergic. disease?

29 176. Feces of a patient with cholera A. Cholera were delivered to a laboratory of B. Typhoid fever extremely dangerous infections. C. Salmonellosis gastroenteritis What method of microbiological D. Diphtheria diagnostics is to be used to confirm E. Intestinal form of plague or deny the diagnosis? 180. The one of identification stages of A. Virological. diagnostics of the patient with previous B. Allergic. diagnosis “Cholera” is the revealing of C. Bacterioscopic. infectious agent monotrichal mobility. D. Biological. What method of determining is used for E. Bacteriological. this purpose? 177. From the vomit mass of a patient A. Method of the "hanging" or there were isolated very motile, "crushed" drop slightly curved, Gram-negative rods B. Gram‟s method which react positively with Inaba's C. Loeffler‟s method diagnostic serum. What symptoms, most D. Method of peptone water probably, will appear with the inoculation treatment absent? E. Method of peptone agar inoculation A. Endotoxic shock. 181. 42-years old man has symptoms of B. Bacteremia. cholera. Bacterioscopic analysis of feces C. Fluid loss. and serological analysis of serum was D. Skin rash. confirmed a clinical diagnosis, while the E. Ulcerous damages of intestine. repeated attempts to isolate the pure 178. A patient with complaints of culture of Vibrio cholerae on ordinary repeated diarrhea and vomiting, pain for choleric vibrio media were not in muscles of legs, general weakness, successful. What is the most probable and vertigo is hospitalized to an reason of failures? infectious department. After A. The tool that was used for the examination a doctor has previously inoculation of the pathological material diagnosed cholera. Which method of had traces of disinfectants investigation of the specimen from the B. The anaerobic conditions of growing patient should be used for express- were not provided diagnostics? C. The agent is auxotrophic mutant A. Immunofluorescence test. D. Culture was infected with virulent B. Agglutination test. bacteriophage C. Bacteriological. E. Isolated culture only D. Allergic. morphologically similar to Vibrio E. Biological. cholerae 179. Patient with diarrhoea was 182. Culture of cholera vibrios was admitted to the infection unit. isolated from feces and vomit of Gramnegative curved rod-like patient. Conducting of which reaction bacteria were founded on will determine the type of microbe that bacterioscopic examination of faecal caused the disease? masses. What is the most likely disease A. Agglutination with serum in this patient? containing O-antibodies

30 B. Agglutination with serum containing pathogenesis of cholera. The main H-antibodies syndrome of the disease is dehydration. C. Passive hemagglutination with Select which of these pathogenic erythrocyte diagnostics factors cause dehydration is the main. D. Agglutination by Widal A. Defect of membrane phospholipids E. Precipitation B. Splitting of neuraminic acid 183. The outbreak of cholera was C. Destruction of hyaluronic acid marked in the region. It is necessary to D. Adenilatcyclase activating reveale which biovary of Vibrio E. Destruction of mucine cholerae is a pathogen. Which of the 187. Patients was hospitalized to the following methods give the most infectious department with suspected reliable results? cholera. What is the primary A. Biochemical identification diagnostics method should be used to B. Serological identification confirm the diagnosis? C. Bio test A. Immunologic D. The polymyxin test B. Bacteriological E. Morphological C. Biological 184. The 10-yeared child with diarrhea, D. Serological vomiting, dry skin, cyanosis, ischuria E. Allergic (urinary retention) was hospitalized to 188. The laboratory of extremely infectious department of the hospital. dangerous infections was performed Pathmaterial was delivered to the microscopic studies of smears made laboratory. What is the most probable from film that has grown on 1% pepton result of pathmaterial inoculation onto water within 6 hours of feces taken medium? from patients with suspected cholera A. Blue colonies on alkaline agar cultivation. On the basis of which the B. Red colonies on Endo medium morphological and tinctorial properties C. Colourless colonies on Endo it is possible to do a preliminary medium conclusion about the presence of D. Yellow colonies on yolk salt agar Vibrios in the grown film? E. Large mucous colonies on meat A. Mobile, Gram-negative curved pepton broth bacillus 185. Comma shape gram negative B. Movable, curved gram-variable bacteria were revealed in the smears bacillus from the patient`s feces. What C. Movable, curved gram-positive properties need to be studied firstly bacillus with microscope use for additional D. Mobile, Gram-negative straight information about microorganisms? fusiform rods A. The presence of cysts E. Mobile, gram-positive pair bacillus B. The presence of capsules 189. Cholerogen is a main pathogenesis C. The presence of spores factor of cholera. It determines D. Mobility dehydration of patients organisms with E. The primary fluorescence cholera. Mechanism of cholera gene 186. Exo-and endotoxins, enzymes of action is: aggression play significant role in the A. Blocks transferase-2

31 B. Causes damage of erythrocytes D. Aggression C. Causes salts hypersecretion E. Hemolysins D. Increases fluid and electrolyte 193. Sanitary Epidemiological Station balance of town S. is controlling the possibility E. Activates adenilatcyclase of V.cholerae detection in seawater. 190. The pure culture of Gram- What is the epidemiology of cholera in negative, slightly curved, motile rods this feature is taken into account? that fermented mannose and sucrose to A. Probability of hospital infection acid and was agglutinated by O1 B. Zoonotic choleraic anti-sera was isolated from C. Sapronotic patient with acute gastroenteritis feces. D. Probability of iatrogenic infection What media was used for isolated pure E. Vector borne transmission culture? 194. Microscopy study of patient with A. 1% peptide alkaline water, TCBS diarrhea defecation was revealed the media bent rod organisms that look like B. Blood meat pepton agar, Ploskirev`s flocks. Which bacteria were revealed? medium A. Vibrio C. 1% pepton alkaline water, Endo B. Bacteria media C. Bacillus D. Meat peptide agar, meat pepton D. Spirillum broth E. Actinomycetes E. Sugar and meat agar, alkaline meat 195. The cholera vibrios pure cultures pepton agar isolated from 31 yeared carrier K. were 191. Feces of a patient C. with studied in a laboratory of extremely cholera were delivered to a dangerous infections. Which laboratory of extremely dangerous properties of cholera agent can be infections. What method of express determined in unfixed preparation? diagnostics is to be used to confirm A. Monotryhic mobility the diagnosis? B. Peritryhic mobility A. Immunofluorescence C. Tinctorial properties B. Complement fixation test D. Capsule formation C. Agglutination test E. The arrangement of bacteria D. Precipitation 196. The bacteriophage titer increase E. Haemagglutination test reaction with the standard cholera 192. The patient with witnessed bacteriophages was used for sea water recurrent diarrhea, vomiting, rice broth quality examination. The reaction was like feces, lowered body temperature positive. This result is evidence of: and heart insufficiency was A. Escherichia coli presence hospitalized to the infection ward. His B. Absence of cholera pathogens in skin is bluish and wrinkled. Which samples pathogenicity factor lead to the C. Presence of organic residues in development of these disorders? samples A. Endotoxin D. Absence of organic water pollution B. Enterotoxin E. Presence of cholerae pathogens in C. Invasiveness samples

32 197. In the feces of a patient with D. Shigella intestinal decease there were revealed E. Helicobacter Vibrio. What group of morphological 201. The biopsies of the ulcer in properties this microorganisms belong mucous membrane were took in patient to? with stomach ulcer during A. Spiral fibrogastroscopy. Gram-negative B. helical bacteria were revealed during C. Cocci microscopy examination of biopsy D. Bacteria smears with positive urease activity E. Bacillus test. Which bacteria have been 198. Diagnosis “Helicobacteriosis” was revealed? put to the patient after bioptate A. Campylobacter jeuni esophagogastroduidenoscopy. What B. Spirilla minor characteristics of isolated from this C. Shigella flexneri patient bacteria was taken into account D. at cultivating? E. A. Lack of spores and capsules 202. The patient with acute colitis with B. The presence of the urease enzyme symptoms of mild intoxication, C. Colonization of the gastric type diarrhea, blood in stool was mucosal cell hospitalized. Gram-negative helical, D. Microaerophilic microaerophilic, catalase positive, E. The presence of six polar flagella urease positive and mobile bacteria 199. Patient S., aged 28, was without spores and capsules were hospitalized with symptoms of mild revealed during bacteriological intoxication and bloody diarrhea. examination. They resemble the wings Gram-negative helical bacteria were of a gull. Name these bacteria. revealed during bacteriological A. Escherichia coli examination of feces. They do not form B. Helicobacter pylori spores and capsules, are C. microaerophilic, catalase positive, D. urease positive and mobile. Name the E. Salmonella typhy bacteria, which are characterized by 203. The motile, gram-negative bacilli these properties. were revealed in the suspected foods. A. Proteus mirabilis They showed creeping growth on meat B. Escherichia coli peptone agar (MPA)in the 18-hour C. Haemophilus influenzae culture. Isolates produced gas H2S and D. Helicobacter pylori indole, fermented glucose, maltose and E. Salmonella typhy sucrose to acid and did not ferment 200. The bacteriological examination lactose, mannitol. Bacteriological study was proposed to patient to establish the showed that the isolated bacteria etiology of gastric ulcers. What belong to the genus: microorganisms would be reavealed? A. Proteus A. Salmonella B. Escherichia B. Listeria C. Pseudomonas C. D. Salmonella

33 E. Shigella general weakness, absence of appetite, 204. The symptoms of acute diarrhea pain in joints. During 10 days she had were developed in 7 patients from the treated flu. The infectionist suspected group of tourists (27 people) which . Using what reaction is it used water from the pond after 2 days. possible to diagnose brucellosis? The material was delivered to the А. Wright's. bacteriological laboratory to study B. Wasserman`s test. etiology of this disease. Which material C. . was used for the diagnosis of the D. Widal's test. disease study? E. Ouchterlony test. A. Food 208. The territory of the burial ground B. Blood of patients of cattle, which has not been used C. Urine for more than 50 years, is planned for D. Water and feces of patients house building. However, soil E. Sputum examination has shown the presence 205. Patient with diarrhoea was of viable spores of an especially admitted to the infection unit. dangerous disease causative agent. Gramnegative curved rod-like Name the microorganism which bacteria were founded on could have been preserved in soil bacterioscopic examination of faecal during such a long time. masses. What is the most A. . likely disease in this patient? B. Francisclla tularensis. A Cholera C. Bacillus anthracis B Typhoid fever D. Brucella abortus. C Salmonellosis gastroenteritis E. Mycobacteriuni bovis. D Diphtheria 209. During a biological test in touch E Intestinal form of plague smears from the organs of an 206. Patient with vomite was admitted animal streptobacteria surrounded to the infection unit. Gramnegative with a capsule were revealed. It curved rod-like gives the basis to diagnosis: bacteria were revealed at A. Brucellosis bacterioscopic examination of vomiting B. masses. What is the most likely disease C. Plague in this patient? D. Anthrax A. Salmonellosis E. Crupous pneumonia B. Cholera 210. During a scheduled C. Crohn's disease examination milkmaids had a D. Yersiniosis Burnet's intracutaneus allergy test. E. Dysentery This test is used to detect to: Microbiological diagnostics of A. Brucellin brucellosis and anthrax B. Tuberculin 207. During the swing of flu epidemic a C. Koch's tuberculin milkmaid referred to a doctor with D. Tularin complaints of high body temperature, E. Anthraxin

34 211. Veterinary attendant, working at a D. B.anthracis cattle farm complaints on joint pain, E. F.tularensis fever, indisposition and sweating at 214. The doctor has put a preliminary nighttime that he has been experiecing diagnosis “Anthrax. Dermal form” to for a month. Giving the regard to such the examined patient. In this case presentations and occupational history microscopy of carbuncle‟s exudates the doctor suspected brucellosis, smears stained by Gram will show: despite the patient works on a cattle A. Big violet bacilli with spores, which farm and despite all complaints. What are arranged in chain material taken from this patient is to be B. Violet bacilli, which are arranged analyzed in an ordinary angularly to each other microbiological laboratory? C. Pink bacilli, which are randomly A. Feces arranged B. Spinal liquid D. Pink bacilli with bipolar coloring C. Vomit masses E. Slightly curved pink bacilli D. Urine 215. A veterinary doctor with the E. Blood serum preliminary diagnosis “brucellosis” 212. A 34-year-old patient complained was delivered to the infectious hospital. of carbuncle on his face. On the basis of what serological test Examination revealed a painless thin this diagnosis can be confirmed? edema of subcutaneous fatty tissue A. Wright‟s agglutination reaction with a black eschar in the center, and B. Vidal‟s agglutination reaction vesicular eruption on the periphery. C. Ascoli‟s precipitation reaction Microbiologic examination revealed D. Veigl‟s agglutination reaction nonmotile capsule-forming E. Complement-fixation reaction streptobacilli. What microorganisms 216. A man referred to the reception are the causative agents of this ward of an infectious disease disease? hospital, having received by mail an A. Bacillus megaterium envelope with suspicious powder. The B. Staphylococcus aureus man was isolated, and the powder C. Bacillus athracoides was sent to the laboratory for D. Bacillus subtilis detecting the presence of anthrax E. Bacillus anthracis causative agent's spores. Which is the 213. A black-stained carbuncle has fastest method of detecting these appeared on the veterinary doctor's microorganisms? cheek after a dead cow examination. A. Precipitation in gel During a microscopic diagnostics the B. Complement-fixation test gram-positive bacilli in chains looks C. Luminescence immunoassay like a bamboo stick were revealed. D. Pure culture isolation What pathogen has these E. Biological assay on mice morphological and tinctorial 217. A child has diagnosis properties? “Brucellosis”. He/she (child) was not in A. P.vulgaris contact with the infected animals. How B. C.perfingens could the child was infected? C. Y.pestis A. Through fresh milk

35 B. Through unwashed fruits and animal raw-materials for the anthrax vegetables agent presence. What reaction is used C. Through water for this? D. Through dirty hands A. Neutralization E. During the injections B. Precipitation in agar 218. A strobiloid red stained infiltrate C. Agglutination with edema was found by doctor on the D. Passive hemagglutination skin hand of 42-yeared butcher. There E. Ring precipitation is a black stained scab in the centre of 222. Anthrax pathogen was inoculated the infiltrate. What disease is it? into gelatin and “inverted fir-tree” A. Flegmona liquifaction of gelatin was observed B. Abscess after incubation. What properties were C. Furunculosis studied in this case? D. Plague A. Proteolytic E. Anthrax B. Sacharolytic 219. A bacteriologist completed such C. Fibrinolytic processes for a complete confirmation D. Hemolytic of the preliminary diagnosis: 1) E. Cultural Inoculation of the pathological material 223. Rodlike bacilla with capsule, onto liver and sugar broth; 2) Isolators‟ located in chains were found during a sensibility checking (before antilinear microscopic examination of carbuncle staining agent‟s action); 3) Serological from patient with anthrax. What reactions of Wright and Haddlson; 4) microbes were revealed? Burnet‟s skin test. What A. Monobacillus diagnosis was confirmed with the help B. Streptobacillus of those bacteriological methods? C. Monobacteria A. Q-fever D. Streptobacteria B. Tularemia E. Diplobacteria C. Typhoid 224. In a laboratory the precipitation D. Salmonellosis test is used (Askoly test) for the E. Brucellosis examination of animal skins. An 220. A patient with brucellosis has a albescent ring formed in some positive Burnet's intracutaneus minutes after adding of the skin allergy test. Which immune system extract to the immune serum. What factor can induce inflammatory does this result indicate? reaction in the site of brucellin A. Presence of Bacillus anthracis introduction? antigens A. Ig A B. Presence of B. Sensitized T-lymphocytes toxin C. Ig E C. Presence of brucellosis causative D. Ig C agent E. Ig D D. Presence of Escherichia surface 221. A soluble thermostable antigen is antigen prepared in water-salt extract from E. Presence of Salmonella Vi-antigen raw-material in order to check the

36 225. A patient complained of painless 228. A patient with the symptoms of carbuncle on his face. Microbiologic tularemia was delivered to a city examination revealed nonmotile infectious hospital. Which method can capsule-forming streptobacilli. What be used for early diagnostics of microorganisms are the causative disease? agents of this disease? A. Allergy test A. Anthrax pathogen B. Biological B. Pathogenic Clostridies C. Serological (agglutination reaction) C. Fusobacteries D. Bacteriological (pure culture D. Plague pathogen isolation) E. Bacteroides E. Microscopic 226. A patient complained about a 229. A patient with symptoms of carbuncle on his face. Examination tularemia was delivered to the results: neither dense nor painful infectious hospital. Which method is edema of subcutaneous cellular tissue, used for an early diagnostics there is black crust in the middle of the nowadays? carbuncle and peripheral vesicular rash A. Bacteriological around it. Bacteriological examination B. Biological revealed presence of immobile C. Serological streptobacilli able of capsulation. What D. RIF (express-method) microorganisms are causative agents of E. Allergic this disease? 230. The lymph nodes material of 36- A. Bacillus antracis year-old man with a suspicion on B. Staphylococcus aureus plague were punctuated and inoculated C. Bacillus subtilis on MPA in Petri dishes. Sterile round D. Bacillus antracoides areas (1-1,5 mm) with homogeneous E. Bacillus megaterium growth of microorganisms have 227. A 34 year old male patient appeared after one day. How to explain consulted a doctor about face the appearance of these sterile spots? carbuncle. Objectively: a loose, A. Wrong selective medium painless edema of hypodermic tissue: B. Pathological material had a small black crust in the centre of carbuncle, quantity of causative agent vesicular rash around it. C. Cells in old cultures give an Microbiological examination revealed autolysis static streptobacilli capable building. D. Microorganism- antagonist was What microorganisms are the a isolated together with causative agent causative agents of this disease? E. Culture is lysogenic A. Bacillus antracis 231. Immunofluorescence reaction was B. Bacillus megaterium used for recognizing a plague bacteria C. Staphylococcus aureus in the smear. The preparation was D. Bacillus antracoides processed with anti-plague serum. E. Bacillus subtilis Glowing bacteria were revealed during luminescent microscopy. It was Microbiological diagnostics of plague estimated as presence of a plague and tularemia causative agent inside the smear.

37 Luminescence of plague bacteria is A. RPHA connected with: B. RA A. Anti-plague antibodies got C. Allergic test connected with the surface antigens of D. CFR (Complement-fixation plague bacteria and produce glow, reaction) because they are connected with E. RIF luminescent staining agent 235. Dwellers of a village noticed mass B. An antigen-antibody reaction took mortality of rodents in some farms. place on the surface of bacteria There is It was suspected that the C. Luminescent staining agent covered animals might have died from plague. bacteria inside the smear What post-mortal examination should D. Plague bacteria produce their own be conduced in order to establish the luminescence causative agent of the infection as soon E. An antigen-antibody complex fixed as possible? the complement on the surface of A. Complement fixation reaction plague bacteria B. Agglutination reaction 232. Gram-negative ovoid bacteries C. Passive agglutination reaction with bipolar filling were isolated from D. Ring precipitation reaction sputum of the patient with high E. Neutralization reaction temperature, shiver, headache and 236. During diagnostics of plague the cough. Bacteries are located in chains credibility of bacteriological forms inside the smear of broth culture examination increases while using and create R-form colonies on agar. immunofluorescence reaction. Describe Which disease is it? the microscopic picture at this reaction A. Plague using: B. Tuberculosis A. Small coccal type pink bacteria C. Anthrax B. Small ovoid rods with bright green D. Diphteria glow E. Streptococcal angina C. Big violet bacillus with chopped off 233. A group of Ukrainian dentists limbs must go to Africa on assignment. But it D. Small pink rods with rounded limbs is known that several hundreds persons E. Slightly arched red bacillus, situated have plague every year in this country. angularly to each other What vaccine (from listed below) must 237. A punctate of groin lymph nodes be used for plague prevention? was taken from a patient with suspicion A. CTI on plague. Material was inoculated into B. Alive vaccine EV solid nutritious media. What looks C. Toxoid must the colonies have if “plague” D. Combined vaccine diagnosis is confirmed? E. Chemical vaccine A. “Patterned kerchief” 234. What must be done to the hunter B. “Drops of mercury” hospitalized on the 5-th day of the C. “Drops of dew” disease for confirming the “tularemia” D. “Shagreen skin” diagnosis on the early stages of E. “Lion‟s mane” diagnostics:

38 238. Ovoid microorganisms about 2 E. A pure culture is isolated on fluid mcm lenth intensively stained on poles media were revealed during microscopy of the 241. A muskrat-hunter‟s temperature patient‟s sputum (the previous has increased to 39 degrees, headache diagnosis is “acute pneumonia”). What and weakness have appeared. A small is the most possible final diagnosis? exponent appeared on his neck skin. A A. Pulmonary plague form preparation was maid from it‟s B. Pneumococcal pneumonia scraping. Rodlike gram-negative C. Staphylococcal pneumonia bacteria, that are very small without D. Klebsiella pneumonia capsules and evenly stained were E. Diphtheria revealed. What causative agent can be 239. In a mountain settlement mass spoken of, considering microorganism death of rodents was observed. morphology, clinical picture and type Simultaneously the inhabitants of this of the patient‟s activity? area were ill. The illness was A. Tularemia accompanied by the fast rise of body B. Anthrax temperature up to 40° C, apparent C. Plague intoxication, increase of inguinal D. Brucellosis lymph nodes. In the touch smears of E. Leishmaniasis cadaveric material Gram-negative 242. A microbiological examination of bipolarly stained ovoid rods were pathologic material stained by Gins- revealed. What microorganisms are the Burri taken from a patient with a causative agents of this infectious suspicion on plague and delivered to disease? the laboratory of extremely dangerous A. Staphylococci infectious diseases was done. What B. Costridia property of causative agent this method C. Causative agents of tularemia allows to study? D. Causative agents of anthrax A. Acidity E. Causative agents of plague B. Spore formation 240. A doctor has suspected a bubonic C. Capsule formation form of tularemia in the patient and D. Alkaline resistance sent the examined material to E. Presence of volutin granules bacteriological laboratory for bacteriological method of diagnostics. Microbiological diagnostics of What is special for this method in this tuberculosis and actinomycosis case? 243. A 7-year-old child has an acute A. A pure culture is isolated from positive tuberculin Mantoux test for the infected laboratory animals first time. What does this result testify D. A pure culture is isolated on solid about? nutritious media A. About Hansens‟ bacillus infection C. A pure culture is isolated using the B. About tubercule bacillus infection elective media C. About BCG vaccination D. Isolated culture is identified using D. About Mantoux test had put before it‟s antigen structure E. About tuberculosis disease

39 244. During official registration of the 247. For prevention of what disease child to school for the decision of (from listed below) a vaccine of alive question about the necessity of attenuated bacteria is used? revaccination Mantoux test was A. Tetanus negative. What does the given result B. Botulism testify about? C. Cougher A. About absence of antibodies to the D. Tuberculosis tubercular bacteria E. Diphtheria B. About the presence of cellular 248. The pulmonary form of disease immunity to tuberculosis was revealed in anamnesis of a patient C. About absence of cellular immunity with tuberculosis. Microscopic to tuberculosis examination of phlegm was carried out D. About absence of antitoxic with the purpose of pathogen immunity to tuberculosis determination. What method of E. About the presence of antibodies to staining was used? the tubercular bacteria A. Neisser 245. At the study of phlegm, taken B. Gram from a patient with suspicon on C. Peshcov tuberculosis, preparation was made and D. Romanovscy-Giemsa stained by Ziehl-Neelsen. What E. Ziehl-Neelsen microscopic picture is seen at 249. Hard phlegmonal infiltrate of dark confirmation of the supposed blue-purple color with numerous diagnosis? fistulas, which excrete a pus with A. Red bacilli on a green background unpleasant smell, was revealed in a B. Microorganisms with the kernel of neck-jaw region of patient. For ruby-red color and blue cytoplasm confirmation of diagnosis C. Red bacteria on a white background “actinomycosis” during microscopic D. Chain like bacilli of violet colour examination of pus the bacteriologist E. Thin red bacteria on a blue must reveal: background A. Druzes 246. Mycobacteria tuberculosis were B. Gram-positive streptococci not revealed in the smear from a C. Gram-negative diplobacteria phlegm of patient with tuberculosis D. Acid resistance bacilli stained by Ziehl-Neelsen. With the help E. Gram-negative diplococci of what methods the probability of the 250. A 16-year-old patient from the bacterioscopic revealing of pathogen countryside has a negative Mantoux can be increased? test. What should the doctor do? A. To prepare preparation of hanging A. Repeat the reaction in a month drop B. To carry out tuberculosis B. To stain by other method serodiagnosis C. To prepare preparation of thick drop C. To quarantine the young man D. To use the luminescent microscopy from the collective urgently of hanging drop D. To conduct the accelerated E. By the methods of flotation and tuberculosis diagnostics by Price homogenization method

40 E. To give BCG injection B. Burri. 251. A 6-year-old child with active C. Gram‟s. tubercular process suspected had a D. Ziehl-Neelsen's diagnostic Mantoux test carried out. E. Ojeshko What immunobiological preparation 255. In the sputum smear stained by was used? Ziehl-Neelsen's method are revealed A. Tularin single or grouping red acid-fast rods. B. BCG vaccine The first growth signs were appeared C. APDT vaccine on nutrient media in 14 days. What D. Tuberculin species do the microorganisms belong E. ADТ vaccine to? 252. A vaccination against tuberculosis A. Mycobacterium tuberculosis is planned in a maternity hospital. B. Yersinia pseudotuberculosis Which preparation of listed below must C. Histoplasma dubrosii be used? D. Klebsiella rhinoscleromatis A. BCG vaccine E. Coxiella burnettii B. APDT vaccine 256. Mycrobacteria tuberculosis (tbc) C. Tuberculin stained by Ziehl-Neelsen were not D. ADТ vaccine revealed in the smear from a phlegm E. STI vaccine of patient with tuberculosis. With the 253. The sputum of a patient with help of what methods the probability of tuberculosis was delivered to the bacterioscopic revealing of bacteriological laboratory. What pathogen can be increased? staining method should be used for A. Enzyme assay microscopic examination of the smears B. Biological method for revealing tuberculosis C. Inoculation of materials on the mycobacteria? media of enrichment A. Ziehl-Neelsen D. Methods of enrichment of material B. Burri-Gins (homogenization and flotation) C. Zdrodovsky E. Serological methods D. Gram 257. Examination of the 36-yeared E. Romanovscy-Giemsa patient‟s sediment was made. Previous 254. Centrifuge urine of a patient diagnosis is kidney tuberculosis. Cord- with renal tuberculosis suspected factor was not revealed by the Price was used to prepare a smear. What method during the microscopy, but method of staining should be used bacteria with resistance to acids were for pathogen detecting? A presented. What examination would preparation for microscopy was confirm or disprove the previous prepared from centrifugate of urine diagnosis? allowance, which was taken from a A. Laboratory animals infection patient who is suspected on renal B. Toxigenicity study tuberculosis. Which method of filling C. Phagotyping of the isolated culture the preparation is used for determining D. Serological identification of the the causative agent? causative agent A. Loeffler's. E. Skin allergy test

41 258. A child underwent Mantoux test. B. Candidiasis 24 hours after allergen injection there C. Tularemia appeared a swelling, hyperaemia and D. Brucellosis tenderness. What are the main E. Actinomycosis components of in the development of 262. The first form pupils went through this reaction? a medical examination aimed at A. Granulocytes, T-lymphocytes and Ig selection of children needing G tuberculosis revaccination. What test B. Mononucleare leukocytes, T- could be used? lymphocytes and lymphokines A. Mantoux C. Plasmatic cells, T-lymphocytes and B. Sheek lymphokines C. Skin test with tularin D. B-lymphocytes, Ig M D. Burnett E. Macrophages, B-lymphocytes and E. Anthraxin test monocytes 263. Mantoux test (with tuberculin) 259. Red rod-like bacteria formed was made to a 10-year-old child. A twisted rodings were revealed by Price papule (8 mm in diameter) appeared on method examination of patient‟s the place of injection in 48 hours. What sputum. What substance provides type of hypersensitivity reaction has conglutination and wisp-like growth of developed after injection of tuberculin? these bacteria? A. Hypersensitivity reaction type IV A. RRD B. Artuse phenomenon type reaction B. Alttuberculin C. Serum disease type reaction C. Phosphatit (Phtyonic acid) D. Atopic reaction D. Tuberculostearic acid E. Hypersensitivity reaction type II E. Cord-factor 264. The immunity to tuberculosis is 260. The medical examination of the 1- preserved until there are live bacteria of st form children included Mantoux. 15 vaccine strain in the body after BCG children out of 35 had negative vaccination of infants. Name this kind reaction. What action should be taken of immunity. against children with the negative A. Non-sterile Mantoux test? B. Humoral A. Repeat the test C. Type specific B. Inject anti-toxic serum D. Innate C. Inject rabies vaccine E. Crossed D. Inject BCG vaccine 265. A patient is curing from chronic E. Examination of the blood serum pneumonia for a long time. Red, singly 261. Druses were revealed in a dark situated (sometimes in small clusters) blue-purple color phlegmonal infiltrate length about 0.25-0.4 mcm bacilli were with numerous fistulas of patient neck- revealed during the microscopy of the jaw region after Gram staining and sputum stained by Ziehl-Neelsen. What microscopy. They were gram-positive disease does the patient have? in the center and gram-negative bulb- A. Lung tuberculosis like. What disease causative agent is it? B. Pneumococcal pneumonia A. Fusobacteriosis C. Lung actinomycosis

42 D. Flue pneumonia 270. The tubercular blebs were E. Candidiasis appeared on the pulmonary 266. Mantoux test was used for tuberculosis patient`s gum jaw region. medical examination of pupils. What What method of staining was used for specific factor the positive reaction acid fast bacteria revealing? cause? A. Neisser A. T-lymphocytes B. Gram B. B-lymphocytes C. Ziehl-Neelsen C. Antibodies D. Romanovscy-Giemsa D. Erythrocytes E. Peshcov E. Leukocytes 271. It`s need to stain the slide with 267. A patient has diagnosis “leprosy” patient`s sputum by Ziehl-Nilsen after complex examination. What skin- method for “tuberculosis” diagnosis allergic test was used for the confirmation. The following reagents diagnostics? were prepared for this purpose: A. Coombs' test carbolic fucsine and methylene blue. B. Molonnie‟s test Which reagent is needed too? C. Deeck‟s test A. 5% sulphuric acid D. Mittsude‟s test B. 3% hydrogen peroxide E. Shick‟s test C. 70% ethyl alcohol 268. The 44 yeared milkmaid with D. Iodine solution complaints to changes in the skin of the E. Vezuvin neck was consulted by the doctor. The 272. The patient‟s sputum with dense blue-claret infiltrate fistula with suspicion on tuberculosis has been unpleasant smell pus was formed. The delivered to the bacteriological fibrous structure granular formations in laboratory. The smears need to be diameter 20-50 microns were revealed stained by Ziehl-Nilsen. Ziehl`s in the pus during microscopy. What fuchsine, 5% solution of H2SO4 and disease can be suspected? methylene blue were used. Name the A. Actinomycosis aim of sulphuric acid using. B. Tularemia A. Neutralization of alkaline staining C. Microsporia agent D. Anthrax B. Smear staining E. Cryptococcosis C. Decolourisation of the acid sensitive 269. Arranged in clusters like the pack bacteria of cigars bacillus have been revealed D. Increasing the perception of during microscopic examination of mycobacterium to staining agent biopsy material from the damaged area E. Dilution of the sputum of oral cavity mucosa. What type of 273. The patient‟s sputum with causative agent was in biopsy material? suspicion on tuberculosis has been A. Mycobacterium tuberculosis collected and delivered to the B. A. israili bacteriological laboratory for C. Mycobacterium leprae bacteriological method. When should D. Mycobacterium avium these results be ready? E. A. bovis A. In 3-4 weeks

43 B. In 3-4 months blue, Water C. In one week 278. While enrolling a child to school D. In 2 days Mantoux's test was made to define E. On the next day whether revaccination was needed. The 274. Calmet and Geren used media test result is negative. What does this with unfavourable substance (bile) for test result mean? vaccinal strain of tubercular bacillus А.Absence of antitoxic immunity to the attenuation. What bacteria properties tuberculosis did they want to change in this way? В. Presence of antibodies for tubercle A. Tinctorial bacillus B. Antigenic С. Presence of cell immunity to the C. Morphological tuberculosis D. Virulent D. Absence of cell immunity to the E. Cultural tuberculosis 275. Slide stained by Ziehl-Neelsen E. Absence of antibodies for tubercle was prepared from a patient sputum bacillus with suspiction on tuberculosis. What 279. A consumptive patient has an properties of the causative agent will be open pulmonary form of disease. important for its identification? Choose what sputum staining should be A. Toxigenicity and immunogenicity selected for finding out the tubercle B. Cultural and enzymatic (Koch's) bacillus? C. Pathogenic and virulence A. Method of Ziehl-Neelsen D. Biological and antigenic B. Method of Burry-Gins E. Morphological and tinctorial C. Method of Gram 276. The patient‟s sputum with D. Method of Romanowsky-Giemsa tuberculosis was sent to a laboratory. E. Method of Neisser Ziehl-Nilsen method was used. For this 280. Tuberculosis can be treated by purpose carbolic fucsine, 5% H2SO4 means of combined chemotherapy that and methylene blue were used. What includes substances with different method was used? mechanisms of action.What A. Ziehl-Neelsen antituberculous medication inhibits B. Gins-Burri transcription of RNA into DNA in C. Gram mycobacteria? D. Peshkov A. Streptomycin E. Neisser B. Rifampicin 277. The proper sequence of staining C. Isoniazid by Ziehl-Neelsen`s method are: D. Ethionamide A. Crystal violet, Alcohol, Safranin E. Para-aminosalicylic acid B. Ziehl`s Fuxin, H2SO4, Water, 281. At the study of phlegm, taken Methylene blue from a patient with suspicon on C. Methylene blue, H2SO4, Ziehl`s tuberculosis, preparation was made and Fuxin stained by Ziehl-Neelsen. What D. Ziehl`s Fuxin, Water, Methylene microscopic picture is seen at blue, Alcohol confirmation of the supposed E. Ziehl`s Fuxin, H2SO4, Methylene diagnosis?

44 A. Red bacilli on a green background of sulphuric acid. What staining B. Microorganisms with the kernel of method was applied? ruby-red colour and blue cytoplasm A. Peshkov`s C. Red bacteria on a white background B. Burri`s D. Chain like bacilli of violet colour C. Ziehl-Neelsen E. Thin red bacteria on a blue D. Gram`s background E. Niesser`s 282. For prevention of which from the 286. Microscopical examination of an enumareted diseases is a vaccine infiltrate removed from the from living attenuated bacteria are submandibular skin area in a 30 y.o. used? man revealed foci of purulent fluxing A. Tetanus surrounded by maturing granulations B. Sausage-poisoning and mature connective tissue, the pus C. Whooping-cough contains druses consisting of multiple D. Tuberculosis short rod-like elements with one end E. Diphtheria attached to the homogenous centre. 283. The pulmonary form of disease What disease is it? was revealed in anamnesis of a patient A. Angina with tuberculosis. Microscopic B. Syphilis research of phlegm was carried out C. Candidosis with the purpose of determination of D. Actinomycosis pathogen. What method of staining was E. Tuberculosis used? 287. A consumptive patient has an A. Neisser open pulmonary form of disease. B. Gram Choose what sputum staining should be C. Peshcov selected for finding out the tubercle D. Romanovscy-Giemsa (Koch's) bacillus? E. Ziehl-Neelsen A Method of Gram 284. In a sick man hard phlegmonal B Method of Romanowsky-Giemsa infiltrate of dark blue-purple color with C Method of Ziel-Neelsen numerous fistulas, which excrete a pus D Method of Neisser with unpleasant smell, was revealed in E Method of Burry-Gins a neck-jaw region. For confirmation of 288. While registering the child to the diagnosis “actinomycosis” during school Mantu's test was made to define microscopic research of pus the whether revaccination was needed test bacteriologist must reveal: result is negative. What does this result A. Druzes of the test mean? B. Gram-positive streptococci A Absence of antitoxic immunity to the C. Gram-negative diplobacteria tuberculosis D. Acid resistance bacilli B Presence of cell immunity to the E. Gram-negative diplococci tuberculosis 285. Specimen of a patient`s sputum C Absence of antibodies for tubercle was stained with the following dyes bacillus and reagents Ziehl`s solution, D Absence of cell immunity to the methylene blue solution, 5% solution tuberculosis

45 E Presence of antibodies for tubercle B Mantoux test bacillus C Supracutaneous tularin test 289. Study of bacteriological sputum D Burnet test specimens stained by the Ziel-Neelsen E Anthraxine test method revealed some bright-red acid- 293. A 10-year-old child had the resistant bacilli that were found in mantoux tuberculin test administered. groups or singularly. When 48 hours later a papule up to 8 mm in inoculated onto the nutrient media, the diameter appeared on the site of the signs of their growth show up on the injection. What type of hypersensitivity 10-15 day. These bacteria relate to the reaction developed after the tuberculin following family: injection? A Yersinia pseudotuberculosis A Type II hypersensitivity reaction B Micobacterium tuberculosis B Arthus phenomenon C Histoplasma dubrosii C Seroreaction D Klebsiella rhinoscleromatis D Atopic reaction E Coxiella burnettii E Type IV hypersensitivity reaction 290. A 15 y.o. boy from a countryside 294. Planned mass vaccination of all entered an educational establishment. newborn 5-7 day old children against Scheduled Manteux test revealed that tuberulosis plays an important role in the boy had negative reaction. What are tuberculosis prevention. In this case the the most reasonable actions in this following vaccine is applied: case? A TAB A To repeat the reaction in a month B Diphteria and tetanus toxoids and B To perform BCG vaccination pertussis vaccine C To perform serodiagnostics of C Diphtheria and tetanus anatoxin tuberculosis vaccine D To isolate the boy temporarily from D Adsorbed diphtheria vaccine his mates E BCG E To perform rapid Price diagnostics 291. Microscopy of stained (Ziehl- Microbiological diagnostics of Neelsen staining) smears taken from diphtheria. Microbiological the sputum of a patient with chronic diagnostics of diseases caused by pulmonary disease revealed red bacilli. bordetellas What property of tuberculous bacillus 295. A patient with suspected was shown up? diphtheria went through bacterioscopic A Alcohol resistance examination. Examination of throat B Alkali resistance swab revealed rod-shaped bacteria with C Acid resistance volutin granules. What etiotropic D Capsule formation preparation should be chosen in this E Sporification case? 292. The first grade pupils were A. Interferon examined in order to sort out children B. Bacteriophage for tuberculosis revaccination. What C. Diphtheria antitoxin test was applied for this purpose? D. Antidiphtheric antitoxic serum A E. Eubiotic

46 296. Microorganism which is identical D Meningococcal nasopharyngitis to Corynebacterium diphtheriae E Epidemic parotitis according to morphological and 299. From pharynx of a child with biochemical signs was isolated from suspected diphtheria a pure culture of the nasopharynx of a 5-year-old child. microorganisms was But this microorganism did not isolated. Their morphological, produce . As a result of what tinctorial, cultural and biochemical process can this microorganism properties appeared to be typical for become toxigenic? diphtheria causative agents. What study A. Passing through the organism of the should be conducted in order to drow a sensitive animals conclusion that this is a pathogenic B. Cultivation in the telluric diphtheria bacillus? environment A Estimation of toxigenic properties C. Phage conversion B Estimation of proteolytic properties D. Chromosome mutation C Estimation of urease activity E. Growing with antitoxic serum D Estimation of cystinous activity 297. In order to estimate toxigenity of E Estimation of ability to decompose diphtheria agents obtained from starch patients the cultures were inoculated on 300. Inoculum from pharynx of a Petri dish with on either patient ill with angina was inoculated side of a filter paper strip that was put into blood-tellurite agar. It resulted in into the centre and moistened with growth of grey, radially striated (in antidiphtheric antitoxic serum. After form of rosettes) colonies 4-5 mm in incubation of inoculations in agar the diameter. Gram-positive bacilli with strip-like areas of medium turbidity clublike thickenings on their ends were found between separate cultures placed in form of spread wide apart and the strip of filter paper. What fingers are visible by microscope. What immunological reaction was microorganisms are these? conducted? A Diphtheria corynebacteria A Precipitation gel reaction B Botulism clostridia B Coomb's test C Diphtheroids C Agglutination reaction D Streptococci D Rings precipitation reaction E Streptobacilli E Opsonization reaction 301. A sample taken from the pharynx 298. While examining a patient an of a patient with angina was inoculated otolaryngologist noticed hyperaemia on the blood-tellurite agar. This and significantly edematous tonsils resulted in growth of grey, radially with a grayish film upon them. striated (in form of rosettes) colonies Microscopical examination of this film up to 4-5 mm in diameter. revealed some gram-positive bacilli Microscopically there can be seen placed at an angle with each other. gram-positive rods with club-shaped What disease might be suspected? ends arranged in form of spread A Diphtheria fingers. What microorganisms are B Angina these? C Scarlet fever A. Diphtheroids

47 B. Streptococci C. Lethal units C. Corinebacteria diphtheriae D. Bacteriostatic units D. Clostridium botulinium E. Hemolytic E. Streptobacilli 305. The pure culture of 302. To determine the toxigenicity of microorganisms was isolated from a diphtheria causative agents isolated child with suspicion on diphtheria. The from patients, cultures are inoculated morphological, tinctorial, cultural and onto a Petri dish with nutrient agar biochemical properties which were on both sides of the centrally located typical for diphtheria agent were filter paper strip sodden with studied. What examination must be antidiphtheric antitoxic serum. What made for the pathogenicity of the must be revealed after cultures diphtheria bacillus revealing? incubation in the agar between A. Urease activity separate cultures and filter paper B. Proteolytic properties strip? C. Toxigenicity properties A. Merging precipitation lines D. Cystine activity B. Crossing precipitation lines E. The starch fermentation C. Zones of diffusion opacification 306. For diphtheria causative agent D. Precipitation lines absence isolation the material must be E. Precipitation ring inoculated on such media: 303. In order to estimate toxigenicity of A. Ploskirev diphtheria agents obtained from patient B. Roux, Loeffler, blood-tellurite agar the cultures were inoculated on Petri C. Levin (Endo) dish with nutrient agar on either side of D. Kitt-Tarocci a filter paper strip that was put into the E. Hiss centre and moistened with 307. A specific pathogenetic treatment antidiphtheric antitoxic serum. After is recommended for a patient with incubation of inoculation in agar the diphtheria. Name this medication. strip-like areas of medium turbidity A. Antibiotics were found between separate cultures B. Bacteria-phages and the strip of filter paper. What C. Anatoxin immunological reaction was D. Anti-toxic serum conducted? E. Sulfanilamides A. Agglutination 308. Among the children of a boarding B. Coomb`s test school there are cases of angina. C. Precipitation in gel Microscopy of tonsil smears stained D. Ring-precipitation by Neisser method revealed yellow E. Opsonization rods with dark-brown terminal granules 304. The patients usually receive arranged in the form of V, W, and X accurately-calculated doses of antitoxic letters. What infection can be serum. In what activity units the suspected in this case? antitoxic antidiphtherial serum is A. Infectious mononucleosis. defined? B. Diphtheria. A. Flocculation units C. Listeriosis. B. International units D. Tonsillitis.

48 E. Scarlet fever. D. Alive weakened diphtheria 309. What localization of diphtheria is causative agents mostly wide-spread now? E. Killed diphtheria bacillus A. Membranous nasal diphtheria 314. A 5 yeared girl has high B. Diphtheritic conjunctivitis temperature and sore throat. C. Ear diphtheria Objectively: soft palate edema, tonsils D. Faucial diphtheria are covered with grey films that can be E. Surgical diphtheria hardly removed and leave deep 310. Anti-toxic serum is used for bleeding tissue injuries. What disease is treatment of: the most probable? A. Gonorrhea A. Lacunar tonsilitis B. Cougher B. Pharingeal diphtheria C. Dysentery C. Infectious mononucleosis D. Diphtheria D. Vincent`s preudomembranous E. Tuberculosis tonsilitis 311. The toxoid was injected to 20- E. Necrotic tonsilitis year-old man. For what disease 315. In the smear from the tonsils of prevention it was done? a patient with diphtheria suspected A. Tuberculosis there have been revealed blue rods B. Diphtheria with thickenings on the ends. C. Scarlet fever What method of staining has been D. Meningitis used? E. Cougher A. Loeffler's. 312. The 5-yeared boy was vaccinated B. Ziehl-Neelsen. according to plan and get an injection C. Gins'. of 1/40 Dlm in his D. Gram's. forearm during diphtheria epidemy. E. Neisser's. There was no reaction after 24-48 316. Bacillus with thickenings on the hours in the place of injection and ends similar to C.diphtheria stained doctor, who cured the child, was with methylene-blue were revealed satisfied of this result. What was the during microscopy. What method must purpose of this test? be used for confirming the suggestion A. Revaccination additionally? B. Anti-toxic immunity estimation A. Neisser C. Skin allergic test B. Kozlovskiy D. Desensitization C. Ziehl-Neelsen E. Bacteria carrier‟s revealing D. Zdrodovsky 313. Vaccine for diphtheria prevention E. Ozheshko and antitoxic immunity providing 317. A dentist has revealed the grey consist of: films on tonsils of the examined child A. Diphtheria toxoid with suspected atypical form of B. Small dose of diphtheria toxin diphtheria. Slide and inoculation on C. Diphtheria toxin and anti-diphtheria nutrient mediums were made and serum together toxigenicity of the isolated pure culture

49 was established. What test was used for B. Check all members for diphtheria diphtheria bacillus toxigenicity study? bacillus carrying A. Hemolysis C. Establish an antibody level against B. Agglutination diphtheria C. Complement fixation test D. Check all medical documentation D. Precipitation in gel about vaccination E. Ring precipitation E. Check the immune status against 318. A child with diphtheria diagnosis diphtheria bacillus was delivered into a hospital. What 322. A passive hemagglutination test medication for specific therapy you was used for level of the anti-diphtheria will use? immunity of child determining. What A. Anti-diphtheria anti-toxic serum must erythrocytes should be sensitized B. Diphtheria anatoxin, antibiotic by to solve the task? C. The “Codivak” vaccine, A. Hemolytic serum sulfanilamide B. Diphtheria anti-toxin D. Diphtheria vaccines: APDT, ADT, C. Antigens of diphtheria bacillus AD D. Anti-diphtheria serum E. Diphtheria bacteria-phages E. Diphtheria toxoid 319. The otolaryngologist has revealed 323. A boy has received the diagnosis hyperemia, edema and the grey films “diphtheria” in the children department on tonsils of the examined patient. of infectious hospital. What preparation Gram-positive arranged angularly to should be injected first? each other bacilli have been revealed at A. Anti-diphtheria anti-toxic serum microscopy. What disease is it? B. Diphtheria anatoxin A. Meningonasopharengitis C. APDT B. Angina D. ADP C. Scarlet fever E. TABte D. Diphtheria 324. Dark blue bacilli with thickenings E. Epidemic parotitis on poles were revealed in the dab made 320. A group of students must be from tonsills incrustation of patient injected for diphtheria prevention. with suspicion on diphtheria. What What preparation must be used for smear staining method was used? creation of the artificial active A. Loeffer immunity? B. Burri A. APDT vaccine C. Gins B. Anti-diphtheria serum D. Gram C. Specific immunoglobulin E. Neisser D. Diphtheria anatoxin 325. The doctor has noticed a grey- E. Vaccine with inactivated bacteria yellow films on the examined child 321. Antidiphtherial immunity must be tonsils that can be hardly removed. examined for necessity of collective Previous diagnosis is “diphtheria?” has vaccination. What examination must be made considering complaints of the done in this case? patient to pains during swallowing and A. Establish the antitoxins title in high temperature. With the help of IHAT

50 what method this diagnosis can be diphtheriae according to morphological confirmed or disproved? and biochemical signs were isolated A. Biological from the tonsills of a two-year-old B. Serological child without planned APDT injection. C. Bacteriological Gel precipitation reaction with anti- D. Allergic test process toxic serum had shown negative result. E. Microscopic What form of the infectious process 326. A pure culture of C.diphtheria was can this agent cause in a child‟s isolated from the patient. What organism if the treatment will not start immunologic test should be used for at the moment? toxigenicity of the bacteria study? A. Unsymptomal carrying of the A. Complement fixation test bacteria B. Agglutination B. Light nontoxic disease type C. Precipitation in agar C. Toxic and complicated disease type D. Hemagglutination inhibition test D. Persist infection E. Indirect hemagglutination test 330. Rod-like bacillus with volutin 327. An antitoxic antidiphtherial granules were revealed during the serum was inoculated to a child with patient‟s smear examination (patient is diphtheria. A rash on a skin, an itch, suspected on diphtheria). What turgidities and a pain in joints were ethiotropic preparation must be chosen appeared in 10 days after injection and in this case? protein was revealed in urine. What are A. Bacteria-phage the reasons of these phenomena? B. Anti-diphtheria anti-toxic serum A. Serum disease C. Diphtheria anatoxin B. Anaphylactic reaction D. Eubiotic C. Atopy E. Interferon D. Hypersensitivity of the delayed type 331. The doctor has suspected a E. Contact allergy diphtheria in a sick child who had a rise 328. Examining a 6-year-old child a in temperature of a body to 38 degrees, doctor has noticed on pharyngeal a pain in a throat at swallowing, a face tonsils a grayish "pseudo-membrane". turgidity, adynamy and grey-white Attempt to remove it causes moderate films on tonsils,. What microbiological bleeding. Bacterioscopy of tonsils methods can confirm the previous swabs has shown Gram-positive club- diagnosis? shaped bacteria. What symptoms can A. Microscopic and allergy test appear within the next few days B. Microscopic + bacteriological without specific treatment? C. Microscopic + serological A. Papular rash on skin D. Allergic + serological B. Pneumonia (Lungs edema) E. Biological + serological C. Very strong attack-like cough 332. From a nasopharynx of the child D. Toxic affection of the cardiac with suspicion on a diphtheria the pure muscle, liver, kidneys culture of microorganisms is allocated E. Intermittent fever with morphological, tinctorial, cultural 329. The microorganisms which is and biochemical properties which are identical to Corynebacterium typical for all diphtheria causative

51 agents were revealed. But isolated bacillus were revealed at culture was not toxic. As a result of bacteriological examination of a what process this culture can become material from an oral cavity of the 5 toxic? yeared boy which in 5 months have A. Phage conversion made a planned an APDT injection and B. Cultivation on the tellurite media revaccination when he was 2 years. C. Putting through the organisms of How can the causative agent‟s presence sensible animals inside the immunized child be D. Fermentation in anti-toxic serum explained? presence A. Only anti-toxic anti-diphtheria E. Chromosome mutation immunity was formed 333. Doctor has suspected diphtheria in B. The APDT vaccine creates unsterile the sick child. A bacteriological immunity examination was completed and this C. After APDT injecting immunity diagnosis was confirmed. What lasts not more than for 2 years nourishing media is used for D. Child is suffering from C.diphtheria? immunodeficiency A. MPB E. Isolated bacteria is a nonpathogenic B. Endo diphtheroid C. Ploskirev 337. A patient has intoxication, D. MPA headache, hyperemia and a pharynx E. Roux swelling. There are grey films on 334. A patient was admitted to a tonsils which may be hardly deleted. clinic in grave condition, fever, The diagnosis is Diphtheria. What is heavy breathing. Preliminary diagnosis the main factor of pathogenicity of the of diphtheria croup was made by the causative agent identifies the described microscopy of throat specimen. What symptoms of the disease? method of staining was used? A. Neuraminidase A. Peshkov's. B. Intracellular toxin B. Ziehl-Neelsen's. C. Hyaluronidase C. Burri-Gins'. D. Exotoxin D. Neisser's. E. Protease E. Ozheshko's. 338. A 20-year-old man was injected 335. A specific prophylactics of with anatoxin in purpose of diphtheria must be made in a prophylactics. What disease was the kindergarten. Which preparation must injection against? be used for this? A. Meningitis A. Anatoxin B. Tuberculosis B. Antibiotics C. Scarlet fever C. Probiotics D. Diphtheria D. Corpuscular vaccine E. Cougher E. Immune serum 339. A 7 y.o. girl was admitted to the 336. The microorganisms, which infectious diseases hospital with fever, morphological and biochemical sore throat, common weaknees. A properties looked like diphtheria doctor suspected diphtheria. What

52 (from listed below) is decisive for the agents can be discovered during the diagnosis confirming after the pure microscopy of the preparation? culture of causative agent isolation? A. Inclusions A. Phagolysability B. Sarments B. Cystinase test C. Capsules C. Toxicity test D. Pili D. Detection of volutin granules E. Spores E. Hemolytic ability of the causative 343. C. diphteriae was isolated in the agent teacher during inspection of bacteria 340. A pure culture from a pharynx of carriers in the children's institutions. C. the sick 9 years old child with diphteriae strain did not produce suspicion on diphtheria was isolated. It exotoxin. What reaction was used for was identified as C. diphtheriae, toxigenicity diphtheria bacillus nontoxic strain. APDT vaccination was examination? done earlier accordingly to a calendar A. Precipitation in agar gel of injections. What is the most possible B. Ring precipitation reaction reason for the child‟s anti-bacterial C. Immunofluorescence reaction anti-diphtherial immunity absence? D. Complement fixation reaction A. A postvaccinal immunity is anti- E. Agglutination reaction toxic 344. A child is presumably ill with B. Child has primary diphtheria. A specimen of affected immunodeficiency mucous membrane of his pharynx was C. Child has secondary taken for analysis. The smear was immunodeficiency stained and microscopical examination D. Child has phagocytic revealed yellow rods with dark blue immunodeficiency thickening on their ends. What E. Revaccination was made by off- structural element of a germ cell was grade vaccine revealed in the detected 341. A toxic culture of causative agent microorganisms? was isolated from a patient C. with the A. Spores previous diagnosis – diphtheria. It is B. Plasmids known, that this activity is connected C. Volutin granules with diphtheria bacteria lysogenicity. D. Capsule What factor determines the E. Flagella lysogenicity of bacteria? 345. In order to determine toxigenicity A. Antibodies of diphtheria bacilli a strip of filter B. Antibiotics paper impregnated with antitoxic C. Phagocytes diphtherial serum was put on the dense D. Factors of self-lysis nutrient medium. There were also E. Temporal phages inoculated a microbal culture under 342. A fauces smear was taken from examination and a strain that is known patient K. and microslide was stained to be toxigenic. If the microbal culture by Neisser. Which structure under examination produces exotoxin, components of the diphtheria causative this wil result in formation of: A Haemolysis zones

53 B Precipitin lines C. Antigens of diphtheria bacillus C Zones of diffuse opacification D. Anti-diphtheria serum D Zones of lecithovitellinous activity E. Hemolytic serum E Precipitin ring 349. In purpose of active prophylactics 346. In order to estimate toxigenity of of diphtheria, tetanus and cougher an diphtheria agents obtained from APDT vaccine is used. What does this patients the cultures were inoculated on vaccine include and what protects Petri dish with nutrient agar on either organism from cougher? side of a filter paper strip that was put A. Attenuated Bordetella petrussis into the centre and moistened with B. Cougher exotoxin antidiphtheric antitoxic serum. After C. Inactivated Bordetella petrussis incubation of inoculations in agar D. Cougher intracellular toxin the strip-like areas of medium turbidity E. Anatoxin were found between separate cultures 350. A 5-year-old boy was delivered and the strip of filter paper. What into a city infectious hospital with the immunological reaction was previous diagnosis “cougher”. Sputum conducted? was taken for a bacteriological A Agglutination reaction examination. Which nourishing media B Coomb's test are used for Bordetellas cultivating? C Precipitation gel reaction A. Serum agar D Rings precipitation reaction B. Levin media, Endo media E Opsonization reaction C. MPB 347. Inoculum from pharynx of a D. Casein-charcoal agar patient ill with angina was inoculated E. Tumanskiy media into blood-tellurite agar. It resulted in 351. A 5-year-old child is having a growth of grey, radially striated (in cougher clinic symptoms. The form of rosettes) colonies 4-5 mm in pathological material (scrape from diameter. Gram-positive bacilli with pharynx) was inoculated on sugar clublike thickenings on their ends meat-pepton agar for this diagnosis placed in form of spread wide apart confirming. No signs of growth fingers are visible by microscope. What appeared in one day. Mark the most microorganisms are these? possible reason for this. A Diphtheria corynebacteria A. Wrong pathologic material B Botulism clostridia B. Wrong nourishing media C Diphtheroids C. A long period of time from taking D Streptococci the material to seeding E Streptobacilli D. Colonies of cougher causative agent 348. In order to establish the level of grow only on the 4-th day anti-diphtheria immunity in a child it E. This disease is not a cougher, it is a was decided to use a passive virus respiratory infection hemagglutination test. This task should 352. The cougher causative agents be completed by the sensibilization of cultures were isolated from patients. erythrocytes by: They gave colonies of different types: I A. Diphtheria antitoxin phase – smooth, consist of virulent B. Diphtheria anatoxin capsulated microorganisms, IV phase –

54 rough, consist of avirulent non- 356. A 6-year-old child has clinical capsulated microorganisms, II and III symptoms of cougher (convulsive phases have average properties. What period). What method of term characterizes this phenomenon microbiological diagnostics can prove most accurately? to be the most effective and be used? A. Dissociation A. Serological B. Transformation B. Bacteriological C. Phage conversion C. Biological D. Adaptable change ability D. Allergic E. Mutation E. Microscopic 353. A patient had high temperature 357. There is an infective cougher and cough attacks for 10 days. Doctor episode in the kindergarten. On the proposed the inoculation of material background of episode a 4-year-old from rhinopharynx onto CCA media. child got catarrhal phenomenon. Child What microorganism is supposed to be is sick since second day. What isolated? microbiological method is most A. Influenza bacillus possibly able to be used for confirming B. Cougher bacillus the “cougher” diagnosis in this case? C. Listeriosis A. Allergic D. Staphylococcus B. Serological E. Klebsiella C. Microscopic 354. A 4-year-old child has clinical D. Biological symptoms of cougher. For serological E. Bacteriological diagnostics of the it 358. For serological diagnostics of the was made large-scale reaction with whooping cough it was made large- parapertussis and pertussis scale reaction with parapertussis and diagnosticums. At the bottom of the pertussis diagnosticums. At the bottom test-tubes with diagnosticum of of the test-tubes with diagnosticum grain-like sediment of grain-like formed. What antibodies have this sediment formed. What antibodies have reaction revealed? this reaction revealed? A. Agglutinins A Precipitins B. Precipitin B Agglutinins C. Bacteriotropins C Opsonins D. Bacteriolysins D Bacteriolysins E. Anti-toxins E Antitoxins 355. There is a vaccination against the 359. A patient has been suffering from cougher planned in a kindergarten. elevated temperature and attacks of What preparation is used for this? typical cough for 10 days. Doctor A. Type-specific serum administered inoculation of mucus B. BCG vaccine from the patient's nasopharynx on the C. APDT vaccine agar. What microorganism is D. Normal (common) gamma-globulin presumed? E. ADP anatoxin A Pertussis bacillus B Pfeiffer's bacillus

55 C Listeria rod shape grampositive D Klebsiella microorganisms were revealed during E Staphylococcus microscopy of wound‟s festering 360. In a kindergarten vaccination allocation. What media is used for the against pertussis is planned. Which clean culture isolation of the infectious preparation is used for the agent? immunization? A. Media Kitt-Tarozzi A. Type specific serum. B. Media Endo B. BCG vaccine. C. Media Levin C. APDT vaccine. D. Meat-pepton agar D. Normal γ-globulin. E. Milk-salt agar E. ADT vaccine. 364. The case of anaerobic infection after the planned surgical operation took place in a hospital. What material Microbiological diagnostics of should be sent for bacteriological anaerobic infection. examination for establishment of this 361. A doctor suspected possible case? development of wound‟s anaerobic A. Blood infection in a patient suffered in B. Urine accident. What preparation was most C. Bandaging, stitch material (silk, expediently to apply for specific catgut) treatment before the establishment of D. Pieces of staggered tissue laboratory diagnosis? E. Tissue liquid A. Bacteriophage 365. The microorganisms that grew on B. Polivalent specific serum glucose-blood agar in anaerobic C. Typospeсific immune serum conditions for 7-10 days, were isolated D. Native plasma during bacteriological examination E. Placental gammaglobulin from the pus operation wound. They 362. A patient with the shin‟s gas were S-form black brilliant colonies gangrene was delivered into the with an unpleasant smell. Polymorphic surgical department. The etiology was gramnegative rods were revealed not revealed. What specific treatment during the microscopy. What should be used for this patient? microorganisms could cause this A. To use the polyvalent antitoxic process? antigangren serum A. Bacteroides B. To make surgical treatment of B. Clostridia wound C. Fuzobacteria C. To prescribe the high doses of D. Veilonella sulfanilamid preparations E. Colon bacilla D. To make vaccination 366. The gas anaerobic infection of the E. To prescribe the high doses of left lower extremity was suspected at antibiotics 39 yeared mechanization expert. 363. During the patient‟s examination Clostridium perfringens was isolated of the jaw-facial necrotic phlegmone a from a wound. The determination of doctor suspected a gas gangrene. The

56 what enzyme has the deciding value D. Living gas anaerobic infection during the identification? agent A. Lecitinase C E. Killed gas anaerobic infection B. Dezoxyribonuclease agent C. Collagenase 371. A laboratory received a material D. Proteinase from a patient's wound. Preliminary E. Hyaluronidase diagnosis is gaseous gangrene. What 367. A patient with the anaerobic microbiological method should be infection of lower extremity in the applied to determine species of defeat area has the edema, gasification, causative agent? necrosis of tissues, general A Allergic intoxication. What factors of B Bacteriological pathogenicity were absent in the C Bacterioscopic infectious agent and do not take part in D Serological development of pathology? E RIA A. Endotoxin B. Patogenic enzimes Microbiological diagnostics of C. Hemotoxin tetanus and botulism D. Necrotoxin 372. Microscopical examination of a E. microbal culture revealed fusiform 368. A patient, who had been injured spore-forming microorganisms that get in car accident, was suspected by the violet-blue Gram's stain. What doctor to be suffering from a possible microorganisms were revealed? wound anaerobic infection. What kind A Clostridia of medication for specific treatment B Streptococci should be applied prior to laboratory C diagnosis determination? D Actinomycete A. Polyvalent specific serum E Diplococci B. Bacteriophage 373. A specimen stained by Ozheshko C. Typespecific immune serum method contains rod-like D. Native plasma microorganisms stained blue with E. Placenta hammagloboulinum round terminal components stained red. 369. Antitoxic sera is used for What are these components called? treatment of: A Spores A. Tuberculosis B Cilia B. Brucellosis C Flagella C. Gonorrhea D Capsules D. Dysentery E Mesosomas E. Gas gangrene 374. Patient with vomiting, dizziness, 370. Vaccine is used for the prophylaxis sensation of dubble vision, difficult of gas anaerobic infection. It consists swallowing was admitted to the of: hospital. Doctor suspects botulism. A. Anatoxin What diagnostic methods should be B. Small dose of toxin used for diagnosis approving? C. Sera A Allergic test, bacteriological

57 B Allergic test, serological A. Reaction of agglutination C Bacteriological, mycological B. Reaction of neutralization D Protozoological, microscopical C. Reaction of precipitation E Biological test, bacteriological D. Reaction of complement fastening 375. Bacteriological laboratory E. Reaction of imunofluoresсent examines canned meat whether it 379. The diagnostics of tetanus were contains . For this made in a laboratory. What method of purpose an extract of test specimen and sterilization can we use for the killing antitoxic antibotulinic serum of A, B, E of the agent of tetanus? types were introducted to a group of A. Pasterization mice under examination; a control B. Boiling group of mice got the extract without C. Tindalization antibotulinic serum. What serological D. Dry heat reaction was applied? E. Autoclaving A Neutralization 380. At a student C. appeared the B Precipitation symptoms of the food poisoning after C Complement binding the canned meat meal in domestic D Opsono-phagocytic terms: diplopia, violation of language E Double immune diffusion and breathing paralysis. What such 376. What group of the vaccine, symptoms of botulism are conditioned applied for the tetanus prevention, this by? preparation does belong to? A. By neurotoxin A. Inactivated vaccine B. By histotoxin B. Attenuated vaccine C. Secretion of enterotoxin C. Chemical vaccine D. Enterotoxic shock D. Anatoxin E. Activation of adenilatacyclise E. Genoingeneering vaccine 381. A sick man is hospitalized with 377. After canned food use a sick man complaining on vomiting, dizziness, N had diplopia, headache, dysphagia, doubling in eyes, violation of infregements of breathing, muscular swallowing. A doctor suspected weakness. A diagnosis is botulism. botulism. What methods of diagnostics What factor of pathogenicity did these should be expediently used for clinical displays of the disease confirmation of diagnosis? connected with? A. Protozoologic, microscopic A. Plasmacoagulase B. Allergic test, serologic B. Agressines C. Biologic test, bacteriological C. Endotoxin D. Bacteriological, serologic D. Exotoxin E. Bacteriological, micologic E. Fibrinolysine 382. For the victim in a motor-car 378. The symptoms of bulbar paralysis accident with the lower jaw trauma the appeared in a patient after canned antitetanus serum was inoculated mushrooms meal: ptosis, diplopia, urgently to a patient. However 2 aphonia and dysphagia. Previous months later a patient was delivered diagnosis was botulism.What reaction into infectious department with the helps to determine the type of toxin? symptoms of „later‟ tetanus. How

58 would it be correctly to do the E. Cholera prevention of tetanus to avoid the noted 386. A patient with the opened break of complications? lower jaw was delivered into the A. To use antitetanus human stomatology-surgical department. What gamagloboulin preparation should be used with the B. To the large dose of antitoxic serum purpose of active immunization against C. To conduct the active-passive tetanus? prevention of tetanus A. Tetanus anatoxin D. Blood transfusion B. Antitetanus immunogloboulin E. To do autohemotherapy C. Antitetanus serum 383. A jerked fish of domestic D. APDT preparation which caused serious food E. Antitetanus vaccine poisoning was examined in a 387. A patient was delivered to a bacteriological laboratory. hospital with a previous diagnosis Microscopy of the culture isolated «botulism». What serologic reaction on Kitt-Tarozzi medium revealed should be used for the botulin toxin‟s microorganisms akin to a tennis- revealing in the examined material? racket. What diagnosis is possible? A. Reaction of precipitation A. Tetanus B. Reaction of agglutination B. Salmonellosis C. Reaction of complement binding C. Cholera D. Reaction of neutralization D. Dysenteria E. Reaction of immunofluorescent E. Botulism 388.7 days later after the plastic 384. A patient N. was delivered to the operation made by a dentist, a tetanus hospital with complaints on vomiting, appeared at a patient. There was doubling in eyes, headache, violation suspicion, that a reason was of swallowing, hard breathing, contaminated by the tetanus infectious muscular weakness. A doctor suspected agent stitch material which delivered in botulism. What methods of diagnostics a bacteriological laboratory for should be used for confirmation of examination. What nourishing media diagnosis? should be used for the primary A. Biological test, bacteriological inoculation? B. Allergic test, serologic A. Kitt-Tarozzi C. Bacteriological, mycologic B. Loeffler D. Protozoological, microscopic C. Levenstain-Iensen E. Bacteriological, serologic D. Endo 385. During patient‟s examination a E. Ploskirev dentist marked a tonic abbreviation of 389. A bacteriological laboratory masticatory muscles and opening of conducts a research of canned meat for mouth was limited. What infectious the presence of botulinum toxin. An disease has such symptoms? extract from the researched material A. Diphtheria with antitoxic antibotulinic serum of A, B. Flu B, E types was injected to the C. Tetanus experimental group of mice. An D. Leptospirosis extract without antitoxic serum was

59 injected to the control group. What 393. A patient after infected product serologic test has been used? use had got the urgent prevention of A. Opsono-phagocytal botulism. Indicate, which from the B. Precipitation noted preparations should be used? C. Complement‟s A. Polivalent antitoxic serum D. Neutralization B. Interferon E. Doubled immune diffusion C. Monovalency antitoxic serum 390. The tetanus symptoms appeared at D. Anatoxin a patient in 7 days after motor-car E. Placent gammaglobulin accident trauma. The course of 394. A 20 dayed baby has dead after a treatment by an antibotulinum serum tetanus infection. Where this infectious was appointed for him and a patient agent can be revealed? began to convalesce. In two weeks a A. Blood temperature has risen, limphatic nodes B. Spinal cord were multiplied, swelled joints, rash, C. Umbilical wound itch and violation from the side of heart D.Gastro-intestinal tract vascular system were appeared in a E. Muscles patient. How the state of a patient is 395. A patient with botulism‟s named? symptoms was delivered to a hospital. A. Serum illness C.botulinum toxin was revealed from B. food stuffs. It is known few people C. Anaphilactic shock used that meal too. Which preparations D. Disbacteriosis must be used for urgent prevention of E. Quink`s edema disease? 391. The grampositive rods with the A. Antitoxic serum terminal located spore that had the B. Anatoxin appearance of «drumstick» were C. Killed vaccine isolated during the bacteriological D. Antibiotics examination of stitch material. What E. Normal immunoglobulin type of bacteria has such 396. A patient addressed to the dentists morphological description? with difficulty of chewing. The doctor A. C.tetani guesses tetanus, cause 3 weeks ago the B. C.botulinum patient unjured his leg with a rusty nail. C. B.anthracis What material is necessary to send to D. C.perfringens bac. laboratory to confirm the E. Y.pestis diagnosis? 392. The infectious agent of tetanus A. Serum of the patient produces exotoxin with different B. Blood of the patient biological action effects. What clinical C. Dab from the surface of wound symptoms can this toxin cause? D. Washings from the nail A. Disorders of vision E. Necrotic tissue pieces B. Lackjaw 397. A patient with the previous C. Diarrhea diagnosis botulism has been D. Rash on a skin hospitalised. What serological reaction E. Nausea should be used to determine

60 botulinum toxin in test specimens? B, E types was injected to the A. Complement fixation test. experimental group of mice. An B. Agglutination. extract without antitoxic serum was C. Neutralization. injected to the control group. What D. Precipitation. serologic test has been used? E. Irnmunofluorescence test. A. Complement fixation. 398. After eating canned mushrooms a B. Precipitation. patient has bulbar paralysis C. Neutralization. symptoms: ptosis, diplopia, aphonia, D. Opsonophagocytic. and dysphagia. Botulism was E. Double immune diffusion. previously diagnosed. What test 402. A student, having eaten meat, should be used to determine the toxin tinned in domestic conditions, has type? symptoms of food poisoning, A. Neutralization. caused by : B. Agglutination. diplopia, dysphrasia, respiratory C. Precipitation. paralysis. What caused such D. Complement fixation test. symptoms of botulism? E. Immunofluorescence test. A. Enterotoxin secretion. 399. A jerked fish of domestic B. Botulinum invasion into preparation which caused serious food intestinal epithelium. poisoning was examined in a C. Neurotoxin action. bacteriological laboratory. D. Endotoxic shock. Microscopy of the culture isolated E. Activation of adenylate cyclase. on Kitt-Tarozzi medium revealed 403. Jerked fish, being the cause of microorganisms akin to a tennis- food poisoning, has been delivered to racket. What diagnosis is possible? a bacteriological laboratory. The A. Botulism. sample was cultu red under anaerobic B. Salmoncllosis. condition. After inoculation into Kitt- C. Cholera. Tarozzi medium a bacteriologist has D. Shigellosis. revealed microorganisms akin to a E. Typhoid fever. tennis-racket. What disease do they 400. The causative agent of tetanus cause? produces exotoxin with different A. Typhoid fever. biological action effects. What clinical B. Salmonellosis. symptoms can this toxin cause? C. Dysentery. A. Disorders of vision. D. Staphylococcal toxic infection. B. Lackjaw. E. Botulism. C. Diarrhea. D. Skin rash. Microbiological diagnostics of E. Nausea. Syphilis 401. A bacteriological laboratory 404. A patient had Syphilis 2 years conducts a research of canned meat for ago. On reproductive organs an ulcer the presence of botulinum toxin. An (shancer) was revealed again. What extract from the researched material form of infection is it? with antitoxic antibotulinic serum of A,

61 A. Reinfection 408. What method of serologic B. Secondary infection examination is not used for Syphilis C. Relapsis diagnostics? D. Superinfection A. Treponema imobilization test E. Autoinfection B. RMP 405. Wassermann test was positive in C. Wassermann test 8-monthes pregnancy woman blood D. Agglutination serum during preventive examination. E. Indirect IF How does the authenticity of serologic 409. Thin spiral pink microorganisms examination of Syphilis should be with 12-14 scrolls and sharp ends were confirmed? revealed in a regionar lymphatic node A. Test must be repeated with 10-15 punctat of patient stained by days interval Romanovscy-Giemza. What disease the B. Again examination after the got infectious agent in this case cause? result A. Leptospirosis C. Again examination after treatment B. Typhus D. To use the sedimentary Kann`s test C. Campilobacteriosis E. To use the sedimentary Zax- D. Sodocu Vitebsky test Е. Syphilis 406. A vein blood was taken for 410. A 30-year-old patient's Wassermann test at the planned Wassermann test is positive (++++). examination of pregnant woman in What infectious disease diagnostics maternity welfare centre. The test was is this test used for? positive. The pregnant woman and her A. Poliomyelitis husband deny extramarital sexual B. Brucellosis relations. What should be done for C. Tuberculosis Syphilis confirmation or refutation of D. Syphilis diagnosis? E. Influenza A. To use the treponema 411. The laboratory assistant has taken immobilization test the blood serum from a patient with B. To take a dab from an urethra previous diagnosed «Syphilis» for the C. To repeat the Wassermann test immune test based on antibodies D. To use the sedimentation reactions which inhibit the motility of E. To use the complement binding test treponemes and kill them revealing. 407. A patient with Syphilis has passed What test has been used for complete course of antibiotic therapy diagnostics? and was fully cured. After some time A. Immobilization he was infected again with Treponema B. Complement binding pallidum. How can be named this form C. Agglutination of infection? D. Precipitation A. Complication E. Neutralization B. Recidive 412. The pinky spirohaetes were C. Secondary infection revealed during the microscoping of D. Superinfection dabs from shancer of a patient. What E. Reinfection method of dab staining was used in this

62 case? B. Wasserman A. Romanovsky-Giemza C. Immobilization B. Burri-Gins D. Indirect IF C. Ziehl-Nilsen E. Agglutination D. Morozov 417. Microscopy of the specimen of a E. Gram regional lymph node punctate 413. A patient was delivered to a stained by Romanowsky-Giemsa hospital with suspicion on Syphilis. showed thin microorganisms with 12- What method should be used for 14 regular curls and pointed ends of detecting the infectious agent in hard light rose color 10-13 micrometers shancer? long. What disease could this agent A. Microscopy of material stained by cause? Gram‟s method A. . B. To isolate of pure culture B. Toxoplasmosis. C. Darkfield microscopy C. Leptospirosis. D. Wasserman test D. Syphilis. E. Agglutination E. Leishmaniasis. 414. The laboratory assistant prepared 418. A patient with syphilis suspected such reagents for Wassermann test: has arrived to a hospital. What method cardiolipid antigen, alcoholic extract of should be used for detecting the lipids from the cardiac muscle of bull causative agent in hard chancre? with cholesterol, inactivated by an A. Microscopy of material stained by ultrasound treponema antigen, Gram's method. hemolytic system, NaCl, examined B. Isolation of pure culture. serums. What component should be C. Using dark-field microscopy. used for Wasserman test at serologic D. Wassermann test. diagnostics of Syphilis? E. Agglutination test. A. Complement 419. Wassermann test is positive in B. Live treponema a 20-yeared man. What infectious C. Ram`s erythrocytes disease diagnostics is this test used D. Diagnostic serum for? E. Antiglobulin serum A. Brucellosis. 415. The dark-field microscopy of the B. Syphilis. 28 yeared patient shancer was used in C. Tuberculosis. the laboratory of dermatovenerologic D. Poliomyelitis. dispanser. What property of Syphilis E. Influenza. treponema was studied? 420. A laboratory assistant has taken A. Active motility blood serum from a patient with B. Spiral shape preliminary diagnosed syphilis for the C. Number of scrolls immune reaction based on detecting D. Primary character of scrolls antibodies, which inhibit the motility E. Secondary character of scrolls of treponemes and lead to their death. 416. What type of serological tests is What test has been used for not used for Syphilis diagnostics? diagnostics? A. Microprecipitation test A. Complement fixation.

63 B. Immobilization. this dangerous illness. What source of C. Agglutination. the infection is the most dangerous? D. Precipitation. A. Rodents. E. Neutralization B. Dairy products. 421. The patients which have delivered C. Cattle. to surgical stomatology department D. Meat products. need to give blood for Wassermann E. Ticks. test. For what purpose this reaction are 425. A patient was hospitalized on used? the 5th day of illness with jaundice А. Detection of antibodies to signs, pain in muscles, rigor, nose Treponema bleeding. A bacteriologist used dark- В. Detection of antibodies to Typhus field microscopy. In the patient's agent blood there have been revealed С. Detection of antibodies to HIV motile spiral microorganisms. Name D. Detection of syphilis agent the causative agent. Е. Detection of antibodies to hepatitis A. Bartonella bacilioformis. agents B. Borrelia duttoni. 422. The patients of surgical C. Calymmatobacterium granulomatis. department need to give blood for D. . Wassermann test. What type of E. Rickettsia mooseri. reaction this test belong to? 426. A patient with a periodically A. Immobilization repeating fever is hospitalized to an B. Complement binding infectious department. In blood C. Agglutination specimen (a thick drop), stained by D. Precipitation Romanovsky-Giemsa method, spiral E. Neutralization microorganisms with sharp ends of blue-violet color are revealed. Which Microbiological diagnostics of disease is caused by this agent? recurrent typhus and leptospirosis A. Relapsing fever. 423. In the micropreparation made B. Typhoid fever. from patient's regional lymph node C. Malaria. punctate and stained by Romanovsky- D. . Giemsa method, the doctor found out E. Leptospirosis. thin microorganisms with 12-14 equal 427. A man died from an acute ringlets and pale-pink sharp pointes 10- infectious disease accompanied by 13 mkm in length. Name the pathogen fever, jaundice, agent. haemorrhagic rash on the skin and A mucous membranes as well as by acute B. Borrelia dutlonii renal insufficiency. Histological C. Calymmatobacterium granulomatis examination of renal tissue (stained by D. Leptospira interrogans Romanovsky-Giemsa method) revealed E. Rickettsia mooseri some convoluted bacteria looking like 424. In the endemic region of C und S letters. What bacteria were leptospirosis the population is ill with revealed? A. Treponema

64 B. Leptospira 432. The patient was hospitalized after C. Spirilla the fifth day of illness with displays D. Borrelia spleen and pains in muscles fever and E. Campilobacteria nasal bleeding. The bacteriologist has 428. A sick man with high temperature performed the dark field microscopy of and a lot of tiny wounds on the body the patients blood. Name the infection has been admitted to the hospital. Lice agent. have been found in the folds of his A.Bartonella bacilliformis clothing. What disease can be B. Borrelia dutlonii suspected in the patient? C. Calymmatobacterium granulomatis A. Scabies D. Leptospira interrogans B. Malaria E. Rickettsia mooseri C. Plague 433. The doctor has revealed blue- D. Tularemia violet thread microorganisms with E. Relapsing typhus several big twist long from 10 to 30 429. A patient with the final diagnosis microns and more at microscopy of recurrent typhus had delivered to an blood micropreparation stained by infectious diseases hospital. What Romanovsky-Giemza. For what material must be taken first of all? infectious disease such microscopic A Liquor picture is characteristic? В. Urina А. Recurrent typhus С. Blood В. Syphilis D. Excrements С. Leptospirosis Е. Smear from a nasopharynx D. Tripanosomosis 430. The population has leptospirosis Е. Leshmaniosis in the endemic zone of this dangerous 434. Recurrent typhus, caused B. disease. What source of this infection caucasica, meets in the definite areas can be most probable? only. Electorobius tick is transmitter. А. Meat products How is such infection called? В. Milk products А. Exotic С. Horned cattle В. Endemic D. Rodents С. Sporadic Е. Pincers D. Pandemic 431. The veterinary surgeon with Е. Epidemic suspicion on brucellosis was arrived 435. A sick man with a fever which to an infectious diseases hospital. sometimes repeated been has admitted During his examination the diagnosis to the infectious hospital. Blue-violet «Relapsing tick typhus» has been put. helical bacteria with sharp ends were What way of transmission the patient revealed in the blood preparation (thick could be invaded? drop) stained by Romanovsky-Giemza. А Through malarial mosquito sting Name the infection agent. В. Through a dog tick sting А Bartonella bacilliformis С. Through a rural tick sting B. Borrelia dutlonii D. Through a mosquito sting C. Calymmatobacterium granulomatis Е. Through an ixodes tick sting D. Leptospira interrogans

65 E. Rickettsia mooseri have been found in his flat. Which of 436. Doctor made clinical diagnosis them may be a carrier of the pathogen recurrent typhus to patient. The of epidemic typhus? microscopy was prescribed for A. Lice laboratory diagnosis confirmation. In B. which material the infectious agent C. Houseflies could be revealed? D. Bed-bugs А. In urine E. Cockroaches В. In sweat 441. A sick man with high temperature С. In blood and a lot of tiny wounds on the body D. In excrement has been admitted to the hospital. Lice Е. Smear from a nasopharynx have been found in the folds of his 437. Examined patient has diagnosis clothing. What disease can be “tick typhus”. How the patient could be suspected in the patient? infected? A. Scabies А Through malarial mosquito sting B. Malaria В. Through a dog tick sting C. Plague С. Through an ixodes tick sting D. Tularemia Д. Through a mosquito sting E. Epidemic typhus Е. Through a rural tick sting 442. Parents with ill child came to the 438. The specific prevention of infectionist. They worked in one of the leptospirosis is planned to members of Asian countries for a long time. Child a society “Hunting and fishing”. What has eathy colored skin, loss of appetite, preparate should be used? laxity, enlarged liver, spleen, peripheral А. Inactivated vaccine of several sera- glands. What protozoan disease can groups this child have? В. Live vaccine of several sera-groups A. Visceral leishmaniasis С. Chemical vaccine B. Balantidiasis D. Anatoxin C. Amebiasis Е. Antitoxic serum D. Toxoplasmosis 439. Blue-violet thread spiral E. Lambliasis microorganisms stained by 443. Patients with similar complaints Romanovsky-Giemza at microscopy of applied to the doctor: weakness, pain in a material from patient with fever. the intestines, disorder of GIT. They were identified as Borrelia Examination of the faeces revealed that recurrentis. What pathological material one patient with four nucleus cysts was taken? should be hospitalized immediately. А. Pus For what protozoa are such cysts В. Blood typical? С. Urine A. Dysenteric amoeba D. Sputum B. Intestinal amoeba Е. Lymph nodes punctate C. Balantidium 440. A patient with suspicion on D. Trichomonas epidemic typhus was admitted to the E. Lamblia hospital. Some arachnids and insects 444. 2 weeks since the blood

66 transfusion a recepient has developed A. Sporozoites fever. What protozoal disease can it B. Shizontes be? C. Gametocytes A. Trypanosomоsis D. Merozoites B. Malaria E. Microgamete C. Amebiasis 448. Slime, blood and protozoa 30-200 D. Toxoplasmosis microns of length have been revealed E. Leishmaniasis in a man's feces. The body is covered 445. A malarial plasmodium with cilias and has correct oval form (haemamoeba) - the pathogene of vivax with a little bit narrowed forward and malaria - has two strains: southern and wide round shaped back end. On the northern. They differ by the duration of forward end a mouth is visible. In their incubation period: the southern cytoplasm there are two nucleuses and has short and the northern - long one. two short vacuoles. For whom are the What selection works in this case? described attributes typical? A. Cutting A. Balantidium B. Stabilizing B. Lamblia C. Artificial C. Intestinal amoeba D. Moving D. Trichomonas E. Sexual E. Dysenteric amoeba 446. A patient who came to the doctor 449. A businessman came to India because of his infertility was from South America. On examination administered to make tests for the physician found that the patient was toxoplasmosis and chronic gonorrhoea. suffering from sleeping-sickness. What Which reaction should be performed to was the way of invasion? reveal latent toxoplasmosis and chronic A. Through dirty hands gonorrhoea in this patient? B. With contaminated fruits and A. RIHA - Reverse indirect vegetables hemagglutination assay C. As a result of mosquito's bites B. IFA - Immunofluorescence assay D. After contact with a sick dogs C. RDHA - Reverse direct E. As a result of bug's bites hemagglutination assay 450. A patient has been brought to the D. Immunoblot analysis hospital with the complaints of E. (R)CFT- Reiter's complement headache, pain in left hypochondrium. fixation test He has been ill for 1,5 weeks. The 447. A journalist‟s body temperature sudden disease began with the increase has sharply increased in the morning of body temperature up to 39,90C. In 3 three weeks after his mission in India, hours the temperature decreased and it was accompanied with shivering and hydropoiesis began. The attacks repeat bad headache. A few hours later rhythmically in 48 hours. The patient the temperature decreased. The attacks had visited one an African country. The began to repeat in a day. He was doctors have suspected malaria. What diagnosed with tropical malaria. What method of laboratory diagnostics is stage of development of Plasmodium is necessary to use? infective for anopheles-female?

67 A. Urine examination gynecologist. Which protozoa can B. Examination of vaginal and urethral it be caused by? discharge A. Trichomonas vaginalis. C. Stool examination B. Toxoplasma gondii. D. Blood examination C. Plasmodium malariae. E. Immunological tests D. Entamocba coli. 451. A woman who was infected with E. Lamblia intestinalis toxoplasmosis during the pregnancy 455. A duodenal content smear of a has a child with multiple congenital patient with indigestion contains defects.This is a result of: protosoa 10-18 mcm large.They have A. Teratogenesis piriform bodies, 4 pairs of filaments, B. Chemical mutogenesis two symmetrically located nuclei in the C. Biological mutogenesis broadened part of body. What kind of D. Recombination the lowest organisms is it? E. Cancerogenesis A Lamblia 452. A duodenal content smear of a B Dysentery ameba patient with indigestion contains C Trichomonas protosoa 10-18 mcm large. They have D Intestinal ameba piriform bodies, 4 pairs of filaments, E Balantidium two symmetrically located nuclei in the 456. Giemsa stained protozoa, which broadened part of body. What kind of have half moon shape with acuminate the lowest organisms is it? ends, blue cytoplasm and ruby-red A. Balantidium nucleus, were revealed during B. Intestinal ameba microscopical examination of spinal C. Dysentery ameba liquid smears. What protozoal disease D. Trichomonas can it be? E. Lamblia A. Leishmaniasis. 453. Slime, blood and protozoa 30-200 B. Malaria. microns long have been revealed in a C. Toxoplasmosis. man's feces. The body is covered with D. Trypanosomosis. cilias and has correct oval form with a E. Amoebiasis. little bit narrowed anterior and wide 457. What are the ways of infection at round shaped posterior end. At the malaria? anterior end a mouth is visible. In A. Alimentary. cytoplasm there are two nucleui and B. Droplett-aerogenic. two short vacuoles. What are the C. Transmissive. described features typical for? D. Contact. A. Lamblia E. Dusty-aerogenic. B. Balantidium 458. 32 yeared woman with C. Trichomonas asymptomatic disease had second dead D. Dysenteric amoeba child was borned with microcephaly. E. Intestinal amoeba What disease can it be? 454. A woman with complaints A. Listeriosis characteristic of inflammatory B. Brucellosis process in vagina has referred to a C. Histoplasmosis

68 D. Toxoplasmosis D. Widal reaction E. Syphilis E. Wassermann reaction 459. Patients had come to the doctor 463. Giemsa stained protozoa, which with similar symptoms: weakness, have half moon shape, protoplasm is intestinal pain, alimentary tract colored blue, nucleus is colored red, disorders. One of the patients needs were revealed during microscopical urgent hospitalization, because after examination of the blood smear. What feces examination four nuclear cysts protozoa can be a pathogen? were revealed. For which protozoa A. Toxoplasma. these cysts are typical? B. Trypanosome. A. Trichomonas C. Leishmania. B. Lamblia D. Lamblia. C. Amoeba dysenteric E. Balantidium. D. Balantidium 464. A patient has addressed to the E. Amoeba intestinal doctor. The doctor detected painless 460. Protozoa were revealed during an ulcers, covered by brown-red peels on examination of the patient‟s Giemsa the opened parts of the body. A stained blood smear. The doctor put surface, covered by granulation was diagnosis: Crus-Chagas disease. What under the ulcers. Round and oval is the pathogen of the disease? microorganisms were revealed during A. Leishmania tropica. Giemsa stained micropreparation B. Toxoplasma gondii. microscopical examination. The C. Leishmania donovani. disease lasts more then year. What D. Trypanosoma crusi. protozoa can be a pathogen of the E. Trypanosoma brucci. disease? 461. The woman with symptoms A. Trichomonas hominis. similar to the inflammatory process of B. Leishmania tropica var. major. the vagina has addressed to the C. Leishmania donovani. gynecologist. What protozoa can be a D. Lamblia intestinalis. pathogen of the disease? E. Leishmania tropica var. minor. A. Trichomonas vaginalis. 465. Round form microorganisms B. Toxoplasma gondii. which cytoplasm has erythrocytes and C. Plasmodium malariae. small-shaped, four nucleus cysts were D. Entamoeba coli. revealed during microscopical E. Lamblia intestinalis. examination of native preparation from 462. The pregnant woman with patient‟s feces. Feces have bloody- complains, similar to the mucus consistency. What is the toxoplasmosis, has addressed to the pathogen of the disease? doctor. The blood examination was A. Entamoeba histolytica. proposed to confirm a clinic diagnosis. B. Entamoeba coli. What serological reaction must be done C. Lamblia intestinalis. in this case? D. Trichomonas intestinalis. A. Complement fixation test. E. Leishmania donovani. B. Precipitation test. 466. A patient has addressed to the C. Neutralization test. doctor because of his infertility. The

69 patient was appointed to toxoplasmosis B. Fluorescent serum against human and chronical gonorrhea examination. immunoglobulin. What reaction must be done to detect C. Human immunoglobulin of hidden toxoplasmosis and chronical normal serum. gonorrhea in this patient? D. Diagnostical serum ,which has A. Immunofluorescence reaction. toxoplasmic antibodies. B. CBT. E. Fluorescein labeled C. Immunoblotting. toxoplasmical antibodies serum. D. Reversed passive hemagglutination 470. A patient with a liver suspicion test has entered the surgical department of E. the hospital. The patient was at the Countercurrentimmunoelectrophoresis. business trip in the one African country 467. Patient H., 40 years old came back for a long period and he had acute home after some month swimming in forms of alimentary tract diseases Western Africa. He feeled weakness, many times. What protozoal disease headache, fever 15 days. The doctor can it be? put diagnosis: malaria. What method of A. Leishmaniasis. laboratory examination can confirm B. Trypanosomosis. this diagnosis? C. Malaria. A. Microscopical, serological. D. Amoebiasis B. Bacteriological, allergic. E. Toxoplasmosis. C. Bacteriocsopical, biological. 471. Protozoa, which size are 10-18 D. Serological, biological. mcm, were revealed in the duodenum E. Microscopical, cultural. content smear. The body has pear 468. A patient with inflammation of shape, four pairs of flagella. There are bile ducts has entered the two symmetrically located nucleuses in gastroenterological department. Lively the front enlarged part of the protozoa protozoa which have pear shape, two body. What protozoa can it be? nucleuses and have supporting rod- A. Amoeba dysenteria axostyle were revealed in bile portions. B. Balantidium. What protozoa disease can it be? C. Amoeba intestinal A. Lambliasis. D. Lamblia. B. Amoebiasis intestinal. E. Trichomonas. C. Balantidiasis intestinal. 472. Patient C., referred to the D. Dysentery infectional department of the hospital E. Trichomoniasis. with his skin ulcer, which appeared 3 469. Indirect Immunofluorescence month ago during his trip to Middle reaction was used to toxoplasmical Asia country. Lanceolated protozoa antibodies detecting in the patient‟s with flagellas were revealed after ulcer blood serum. The first step is to cover material on blood defibrinated agar fixed toxoplasmical smear by inoculation. What protozoa can it be? experimental serum. By what will you A. Trichomonas vaginalis. cover your preparation at the second B. Leishmania donovani. step? C. Toxoplasma gondii. A. Fluorescein solution. D. Lamblia intestinalis.

70 E. Leishmania tropica. Mycoplasma was not revealed after 473. Toxoplasmosis was suspected in second time pathological material the newborn. Enzyme linked inoculation onto the media. Possible immunosorbent assay was used for reason of negative result is wrong specific antibodies detection in the prepared culture medium. Absence of umbilical blood. Presence of what what substance in culture medium immunoglobulin will confirm would be the most possible reason of intrauterine infection? negative result? A. IgA. A. Agar-agar B. IgM. B. Axerophthol (retinol) C. IgG. C Glicerine D. IgD. D. Glucose E. IgE. E. Cholesterol 474. Patient H with complains for 477. Bacteriologic examination of prostate pains and large excretion from urethral discharge of 40-yeared man urethra has addressed to the with chronic inflammation of urinary- infectionist. The doctor thinks that it is genital system was performed. After the trichomoniasis. What method of inoculation of pathological material laboratory examination can confirm onto the serum agar with cholesterol, this diagnosis? small colonies with a dense center A. Enzyme-linked immunosorbent ingrown into culture medium and with assay, ELISA, for antibodies detection semilucent periphery grown were B. Immunofluorescence reaction for appeared. What causative agent it was? antigen detection. A. C. An allergic test B. Gonococci D. Excretion smear microscopical C. Mycobacterium examination. D. Gardnerella E. Bactereological test. E. Mycoplasma 475. Two weeks after hemotransfusion 478. 11-yeared girl, who cared for the a patient had fever. What protozoal animals in a school ''live corner”, has a disease can be suspected? flu like disease (weakness, fever, loss of A. Leishmaniasis. appetite and great headache).The B. Trypanosomosis. symptoms of bronchopneumonia C. Malaria. appeared later. The causative agent was D. Amoebiasis revealed after patient‟s blood injection E. Toxoplasmosis. of chicken embryos into a yolk sac. Skin test with chlamidin was positive in Microbiological diagnosis of this sick girl and in three schoolboys, chlamidiosis, mycoplasmosis and that worked in a «alive corner».What animal could be the most possible 476. The patient with clinical signs of source of this infection? Mycoplasmosis was examined. The A. Hamsters diagnosis was confirmed by B. Rabbits bacteriologic method in a patient with C. Parrots the clinical signs of mycoplasmosis. D. Tortoises

71 E. Cats Antibiotic with what mechanism of 479. The cytoplasmа inclusions in the action should not be used for treatment? form of a typical little “cap” above the A. Antibiotics inhibiting synthesis of nucleus of the cell were revealed in the cellular wall epithelial cells during the microscopy of B. Antibiotics disturbing the urethral smears. What disease should be permeability of plasma membrane surmised? C. Antibiotic disturbing protein A. Chlamydiosis synthesis B. Gonorrhoea D. Antibiotic disturbing nucleic acids C. AIDS synthesis D. Siphilis E. Antibiotic disturbing the oxidative E. Genital herpes phosphorylation 480. The epithelial cells with reticular 484. A 30-yeared man has urethritis and and elementary little bodies stained by prostatitis. With the aim of Romanovsky-Giemza were revealed microbiological diagnostics culture during microscopy of scrape of mucous isolation was performed. The growth of membrane of a patient with urino- causative agent was achieved only on genital infection. Which pathogenic the culture medium with 10 % urine microorganisms is it? addition. What group of A. Mycoplasmas microorganisms does a causative agent B. belong to? C. Rickettsia A. Chlamydia D. Viruses B. Neisseria E. Fungi C. Mycoplasma 481. was D. Gardnerella isolated from the genital tract of 35- E. Staphylococci yeared man with chronic urethritis. 485. A patient with an urethritis within What other organs can impress this a week independently treated oneself by causative agent? the penicillin, but the state of patient A. Gastrointestinal tract did not improve. The bacteriological B. Kidneys examination showed that a causative C. Joints agent is mycoplasma. Why medications D. Central Nervous System accepted by the patient were not E. Eyes effective? 482. What research method is the most A. Membrane of mycoplasma contains informative at the control of Chlamydia cholesterol infection treatment? B. Causative agent multiply inside the A. Determination of antibody titer in cell ELISA C. Mycoplasma produces the enzyme B. Direct IF destroying penicillin C. PCR D. Mycoplasma does not have a cellular D. Western blot wall E. Microscopic smears E. Mycoplasma does not form 483. Pneumonia of mycoplasma corresponding transport proteins etiology was diagnosed in a patient.

72 486. A patient with suspected C. Salmonella typhi rickettsiosis was delivered to the D. hospital. What method of diagnostics E. Pneumocystis carini must be applied for confirmation of diagnosis? Myсological diagnostics of a A. Cultivation in a chicken embryo candidiasis and mycosis B. Microscopic 490. A child with clinical signs of C. Cultivation on the artificial candidiasis was hospitalised to the nourishing media clinic. What laboratory examination is D. Serological necessary for the diagnosis E. Biological "candidiasis" confirmation? 487. The method more frequently used A. Microscopical, mycological in bacteriological laboratory for B. Allergic, mycological diagnostics of the epidemic spotted C. Histological, mycological fever is: D. Biological, mycological A. Serological method E. Serological, mycological B. Infection cells cultures 491. A pregnant woman complained of C. Infection chicken embryos itching and genital tracts discharges. D. Bacteriological method Bacterioscopy of vaginal discharges E. Allergic method smear showed big Gram-positive 488. The 73-yeared woman has shallow oval oblong cells which form descents on a skin, high temperature pseudomycelium. What is the most and dizziness. Chicken embryos yolk possible way of transmission? sack infection with blood of the patient A. Sexual showed the presence of ultrafine Gr- B. Endogenous infection bacillary and coccoidal form C. Contact indirect microorganisms. The reaction of D. Transmissive Weighl-Felix is positive. In childhood E. Wound infection the patient had typhus, but she does not 492. Microscopy of the smear of remember which one. What method of vaginal discharges of a woman with diagnostics will allow to find out if this chronic colpovaginitis revealed round disease is epidemic or and oval budding cells 3-6 Brill disease? micrometers in size. Name the fungal A. Infection laboratory animals disease, which can be caused by these B. Reaction of complement binding microorganisms. with the paired sera A. Epidermophytium. C. Reaction of microagglutination B. Coccidiosis. D. Allergic skin test C. Candidosis. E. Definition IgM and IgG D. Microsporia. 489. A patient has diagnosis Brill E. Cryptococcosis. disease as a result of blood serological 493. Choose among mentioned below examination. What microorganism is the most probable reasons of acute the agent of this disease? pseudomembranous candidiasis (milky A. disease) development. B. Pseudomonas aeruginosa A. Long antibiotic therapy

73 B. Immunodeficiency of B-system preparations-dabs. Name this agent. immunity A. Fuzobacterii C. Long immunomodulating therapy B. Staphylococci D. Chronically inflammatory processes C. Diphtheroides E. Atopic status D. Actinomycetes 494. A whitish touch scraping from an E. Candida oral cavity mucous membrane was 498. A child was hospitalized to a directed to the laboratory. As a result clinic. On the mucous membrane of of microscopic examination it has been cheeks, palate, and tongue there was found out, that the microorganism revealed white and yellowish fur belongs to the list of opportunistic typical of candidiasis. What material mycoses agents. What of the resulted should be taken for examination? microorganisms is the contagious A. White exudates (plaques) from mycosis agent, instead of an different sites of oral cavity. opportunistic one? B. Hair and nails. А. Microsporum cаnis C. Blood. В. Aspergillus niger D. Mucus from nasopharynx. С. Candida albicans E. Urine. D. Pneumocystis carinii 499. The scraping of albescent fur Е. Mucor mucedo from oral cavity mucous membranes 495. Gram-positive big oval budding was sent to a laboratory. The cells which are located chaotically and material was inoculated onto in the form of chains were revealed Sabouraud's medium. Sour cream- from mucous membranes and sputum like colonies' growth was revealed. of the patient who used Bacterioscopic method has shown immunosuppressants long time. What short budding strings. What group agent has been isolated? of infections can we refer this disease A. Actinomycetes to? B. Streptobacterii A. Mycosis. C. Yersinii B. Spirochetosis. D. Candida C. Rickettsiosis. E. Streptococci D. Mycoplasmosis. 496. The oral cavity candidiasis was E. Chlamidiosis. diagnosed in the child. What 500. The dark purple, oval, roundish, preparation is used for candidiasis budding cells were revealed at treatment? microscopy of scrape from a tongue A. Nystatin surface stained by Gram's method. B. Gentamycin What disease agent is it? C. Penicillin A. Diphtheria D. Tetracyclin B. Actinomycosis E. Cyfran C. Streptococcal infection 497. Whitish exudates which remind D. Staphylococcal infection curds milk was revealed in the child`s E. Candidiasis cheeks mucosa and tongue. Gram- 501. After long antibiotic therapy a positive, oval yeast were revealed in mouth mucous membrane sites with

74 white fur which did not deleted with from fur has revealed large, roundish, spatula have been revealed in patient. Gram positive various sizes For what disease such signs are microorganisms. What it is necessary characteristic? to make for continuation of A. Parodontitis microbiological diagnostics? B. Parotitis A. Yolk-salt agar inoculation C. Candidiasis B. Ploskirev agar inoculation D. Flue C. Kitt – Tarozzi media inoculation E. Typhus D. Endo agar inoculation 502. A pediatrician examining a child E. Saburaud agar inoculation has noticed that the mucous 505. The oral cavity candidiasis is membrane of the oral cavity and the diagnosed in the child. What tongue are covered with dense preparation should be used for whitish fur. In the material taken candidiasis treatment? from the site of affection a A. Cifran bacteriologist has revealed yeast. B. Gentamycin What mycosis will he suspect in this C. Penicillin. case? D. Tetracyclin A. Actinomycosis. E. Nystatin B. Favus. 506. The patient had a sensation of C. Epidermophytia. burning in a mouth, his tongue was D. Candidiasis. covered with dense whitish cheese E. Trichophytosis like fur. The milk disease was 503. The white creamy plaques which diagnosed. What preparation should easily deleted leaving blood erosion be used for treatment? have been revealed on a mouth mucous A. Grizeofulvin membrane of pupil С during routine B. Nystatin inspection of schoolboys by the doctor- C. Gentamycin stomatologist. The doctor has D. Amfotericin suspected pseudomembranous E. Tetracyclin candidiasis and has appointed to the 507. A child was hospitalized to a patient mycological examination. What clinic. On the mucous membrane of of the listed nutrient mediums should cheeks, palate, and tongue there was be used for revealing of the candidiasis revealed white and yellowish fur agent? typical of candidiasis caused by A. Saburaud Candida albicans. What of the listed B. Roux preparations is used for candidiasis C. Endo treatment? D. Kitt-Tarozzi A. Penicillin E. Hiss B. Cifran 504. Patient Т, 68 years old, has passed C. Nystatin a long course of antibiotic. After D. Gentamycin treatment the patient had a white fur on E. Tetracyclin an oral cavity mucous membrane. 508. Microscopic examination of a Microscopic studying of dabs prepared Gram-stained scrape from patient's

75 tongue revealed oval, round, A. Mycosis. elongated chains of dark-violet B. Spirochetosis. gemmating cells. What disease can be C. Rickettsiosis. caused by this causative agent? D. Mycoplasmosis. A. Actinomycosis E. Chlamidiosis. B. Candidosis 512. On the mucous membrane of C. Streptococcic infection cheeks and tongue of a child albescent D. Staphylococcic infection stains were revealed. In the prepared E. Diphtheria smears Gram-positive oval yeast-like cells 509. A child was hospitalized to a are found. What causative agent is this? clinic. On the mucous membrane of A. Corinebacterium diphtheriae. cheeks, palate, and tongue there was B. Staphylococci. revealed white and yellowish fur C. Fungi of Candida genus. typical of candidiasis. What material D. Actinomycetes. should be taken for examination? E. Fusobacteria. A. White exudates (plaques) from 513. A pregnant woman complained of different sites of oral cavity itching and genital tracts discharges. B. Hair and nails. Bacterioscopy of vaginal discharges C. Blood. smear showed big Gram-positive D. Mucus from nasopharynx. oval oblong cells which form E. Urine. pseudomycelium. What is the most 510. A pediatrician examining a child plausible causative agent of the has noticed that the mucous disease? membrane of the oral cavity and the A. Staphylococci. tongue are covered with dense B. Fungi of Candida genus. whitish fur. In the material taken C. Streptococci. from the site of affection a D. Diplococci. bacteriologist has revealed E. Sarcina. psudohypha. What mycosis will he 514. Candidosis is diagnosed in a suspect in this case? child. What preparation should be A. Actinomycosis. used for treatment? B. Favus. A. Cifran. C. Epidermophytia. B. Gentamycin. D. Candidiasis. C. Penicillin sodium salt. E. Trichophytosis D. Tetracycline hydrochloride. 511. The scraping of albescent fur E. Nystatin from oral cavity mucous membranes 515. Scraps of the mycelium of a was sent to a laboratory. The fungus, spores, air bubbles and fat material was inoculated onto drops were discovered on microscopy Sabouraud's medium. Sour cream- of the patient's hair excluded from the like colonies' growth was revealed. infected areas. For what fungus disease Bacterioscopic method has shown is this microscopic picture short budding strings. What group characteristic? of infections can we refer this disease A. Favus to? B. Microspory

76 C. Trichophytosis after long-term treatment with D. Epidermophytosis ceftriaxone. Smear microscopy E. Sporotrichosis revealed large round cells with a 516. A doctor asked the patient, the nucleus. Which media are used for skin of the scalp which were affected agent isolation? area of hair chippy near the skin A. MPA surface. Small spores were revealed B. Nutrient agar inside the hair at microscopy. Which C. Saburaud disease is it? D. Yolk salt agar A. Rubromycosis E. Roux B. Favus 520. Man 70 y.o. developed prosthetic C. Trichophytia stomatitis. In addition, there was D. Candidosis pronounced corners of mouth lesions. E. Microsporia What microorganisms most likely were 517. 40 yeared man has the damaged the leading etiological factor for this areas between the toes on his feet: skin injury? get wet, peel, crack appeared. When A. Streptococci sowing scrapes skin. The fluffy white B. Staphylococci top and a greenish-yellow underneath C. Neisseria colonies were grew on Saburaud D. Candida media. Conidium as "cudgel‟ with 1-5 E. Clostridium perfringens cells is visible n smears from the top of 521. Dentist suspected Candida the colony. What other organs most etiology stomatitis in children 4 years likely can affect this fungus? old and appointed microscopic study. A. Mucous of genitalies What are the structures that can be B. Hair found in the smears from the damaged C. Under the skin tissue mucosa, confirm the preliminary D. Lymphatic vessels diagnosis? E. Nails A. Cells that form buds 518. A doctor revealed threads of B. Ovoid cells formed ascospores mycelium showed double-circuit, as C. Branching threads round and square disputes at D. Long cells form psudohypha microscopy of affected areas E. Small spores epidermis pieces of skin taken from the 522. Select from the following below interdigital folds and soles of patients the most likely causes of acute (coal miners). A fungal pathogen of pseudomembranous candidiasis which disease may be referred to in this (thrush). case? A. Chronic inflammation A.Microsporia B. B-system immunedeficiency B.Candidosis C. Long-term immunomodulatory C.Trichophytia therapy D.Epidermophytia D. Prolonged antibiotic therapy E.Scabiasis E. Atopic state 519. A dense whitish fur which easily 523. Psudohypha was revealed in a deleted was revealed in the oral cavity smear of the studied material taken

77 from patients with suspected fungal infection. What micro organism is it? A. Aspergillus B. Penicillum C. Candida D. Actinomycetes E. Mycobacterium 524. Prolonged use of antibiotics in the patient K., 43 years, causing oral mucosa candidiasis. What type of Candida most commonly causes oral candidiasis? A. S.aureus B. C. tropicalis C. С.albicans D. S.pyogenes E. C.pylori

78 525. After continuous treatment with B. Do not have a cellular structure, antibiotics a patient got symptoms of not capable of growth and binary stomatitis. Examination of specimens fission,they have only one type of of oral mucous membrane revealed nucleic acids some oval polymorphous Gram- C. They have both types of nucleic positiv microorganisms arranged in acids, have no cell structure, not clusters. What microorganism may be capable of binary fission the causeof such manifestations? D. They able to grow and to binary A. S.aureus fission, have cellular structure, have B. C. tropicalis only one type of nucleic acids C. С.albicans E. Do not have a cellular structure, D. S.pyogenes have both types of nucleic acids, E. C.pylori capable of growth and binary fission 528. Dimensions of virions different Methods of viruses cultivation, viruses vary widely: indication and identification A. From 45 to 500 nm 526. Viruses don‟t have a cellular B. From 0,5 to 500 nm structure, but they carry the basic C. From 25 to 600 nm functions of living organisms. What D. From 15 to 400 nm features of living organisms are E. From 5 to 200 nm typical for? 529. Capsids of virions have a clearly A. Heredity, variation, capacity for ordered structure. What are the types reproduction of symmetries underlying? B. Parasitic mode of existence, A. Rhabdoid, combined heredity, variability, inability to B. Spiral, spherical binary fission C. Spiral, cubic, combined C. Parasitic mode of existence, D. Cubic, Spermatozoon-shaped heredity, variation, the ability to have E. Spermatozoon-shaped, spherical only one type of nucleic acids 530. Due to the interaction of virus D. Heredity, capacity for with the host cell can develop reproduction, parasitic way of productive cell type of virus infection. existence, the ability to have only one Thus there: type of nucleic acids A. Integration of viral and cellular E. The ability to have only one type of genome nucleic acid, variability, capacity for B. Fine virus reproduction, parasitic way of C. Inhibition of virus reproduction existence D. 527. Viruses belong to the Vira E. Enhancement of virus reproduction kingdom . What characteristics 531. The bacteria is called lysogenic: distinguish them from viruses pro-and A. In condition of phages‟s eukaryotes? reproduction A. Do not have a cell structure B. Lysed in the allocation of phages capable of growth and binary fission C. Infected with virulent phages have only one type of nucleic acids D. Have on their surface adsorbed phages

79 E. Have in its genome prophage hemagglutination 532. To isolate the virus used chick E. The reaction of complement embryos aged: fixation A. From 3 to 10 days 536. The high variability of influenza B. From 6 to 15 days A viruses, implemented through the C. From 10 to 15 days mechanism of the "shift" is associated D. From 5 to 20 days with features of the virus E. From 10 to 20 days A. The virus has a fragmentirium 533. Most effective in combating viral genome infections, which have aerogenic B. The virus has a negative RNA transfer mechanism are: C. The virus will infect both human A. Measures aimed at the source of and animal infection D. The structure of the virion consists B. Measures, interrupt transfer of lipids and carbohydrates mechanism E. The virus has spiral RNA C. Measures to create a specific 537. Infection of chick embryos is the immunity main method of isolation of influenza 534. Material from patient A., 4 years, virus. When injected into the chicken with suspected enterovirus infection embryo test material (nasopharyngeal were infected with cell culture to wash) is added prior to it: virus accumulation. What type of A. Streptomycin and penicillin tissue culture is represented by tumor B. Periyodat of potassium cells and is able to divide C. A solution of Needle indefinitely? D. Ethanol A. Transplantable cell line E. Ether B. culture 538. On the basis of clinical C. Culture of primary cells symptoms and epidemiological data, D. Diploid cell lines the doctor put the patient the diagnose E. None of these of the flu. Which family contains viruses? Laboratory diagnostics of A Picornavirus Ortomyxoviral, Paramixoviral and B. Herpes Rhabdoviral infections. Lesions of C. Gepadnovirus the oral cavity under conditions of D. Ortomiхovirus influenza and measles. E. Togavirus 535. In the laboratory smears were 539. A pediatrician, conducting a taken of the mucous membrane of the conversation with parents about nasal cavity in a patient with measles, noted that a certain category suspicion of influenza. What are the of children has a natural passive methods of investigation must be immunity to the disease. Which ones carried out to identify the virus? children had a physician in mind? A. Immunoassay analyze A. Over 14 years B. The reaction of agglutination B. Newborns C. The reaction of precipitation C. Those who had measles in the first D. Reaction of indirect year of life

80 D. Those who received routine D. Chemical vaccine immunizations E. Antiidiotypic vaccine E. Those whose parents had not 544. For the specific prophylaxis of measles influenza among students and those 540. Pathological material working in enterprises in the service (nasopharyngeal mucus) was taken is used: from a patient with suspicion of flu, A. Live vaccine which have infected the chicken B. Killed vaccine embryos in chorion-alantoisnus C. Anatoxin cavity. Through which the reaction is D. Chemical vaccine the most expedient to prove that HAP E. Antiidiotypic vaccine actually accumulated the influenza 545. The composition of the outer virus and to determine the type of shell processes of spikes flu include: virus? A. gp120, gp41 A. Precipitin B. p24, p17 B. Double C. , neuraminidase C. Immunofluorescence D. Hemolysin, coagulase D. Нemagglutination E. Hemagglutinin, hialuronidase E. Neutralization 546. The composition of the outer 541. Typical influenza virus shell processes spikes parainfluenza belonging is determined: virus include: A. The neutralization reaction A. gp120, gp41 B. The reaction of complement B. p24, p17 fixation C. hemagglutinin, neuraminidase C. The reaction of agglutination D. hemolysin, coagulase D. The reactions of precipitation E. hemagglutinin, hialuronidase E. The reaction of hemagglutination - 547. For the specific prophylaxis of inhibition parainfluenza is used: 542. The serological reactions are A Live vaccine used to establish the subtype of the B. Killed vaccine hemagglutinin of influenza virus: C. Anatoxin A. The reaction of agglutination D. Chemical vaccine B. The reaction of precipitation E. Is not developed C. The reaction of complement 548. Mumps virus belongs to the fixation family: D. The reaction of neutralization A Picornavirus E. The reaction of hemagglutination - B. Rhabdovirus inhibition C. Ortomyxovirus 543. For the specific prophylaxis of D. Paramyxovirus influenza in people aged 65 and older E. Rhinoviruses with chronic, cardiovascular disease is 549. Transmission of mumps: used: A. Alimentary A.Live vaccine B. Sexual B. Killed vaccine C. Respiratory C. Anatoxin D. Parentheral

81 E. Tranplacental C. Prodromal period, the period of 550. Entrance gate of infections with eruption mumps: D. The last day of incubation, A. The mucous membrane of the prodrome, the period of eruption nasopharynx E. The last 5 days of incubation, B. The salivary glands prodrome C. The mucosa of the intestine 556. An early pathogenesis symptom D. Damaged skin for measles is to identify: E. The mucosa of genital organs A. Cells of Babesh- Negri 551. For mumps the complications are B. Spots of Bielski-Filatov-Koplik characterized by: C. Granule of Babesh-Ernst A cardiovascular system, pancreas, D. Cells of Guarnieri testes, E. Listing of Prowazeki- B. Liver, thyroid, testicular Galbertshedter C. The nervous system, mammary 557. For the specific prophylaxis of glands, testes, measles apply: D. The thyroid and pancreas, liver A. Live vaccine E. Cardiovascular system, mammary B. Killed vaccine glands, testes, C. Anatoxin 552. For the specific prevention of D. Chemical vaccine mumps shall apply: E. Is not developed A. Live vaccine 558. For a specific treatment for B. Killed vaccine measles apply: C. Anatoxin A. Live vaccine D. Chemical vaccine B. Killed vaccine E. Is not developed C. Anatoxin 553. The measles virus belongs to the D. Immunoglobulin family: E. Is not developed A. Ortomyxovirus 559. Cytopathic effect of mumps B. Рaramyxovirus virus to sensitive tissue culture is C. Picornaviruses characterized by: D. Poxviruses A. The appearance of simplast with E. Rhabdovirus multiple cores 554. The main route of transmission B. destruction and formation of of measles is: granulation in the infected cells A. Respiratory C. Improving the refraction of B. Alimentary affected cells, which are then rounded C. Parentheral and separated from the D. Contact-home glass E. Tranplacental D. The formation of giant cells, which 555. Infectious period for measles is: have intranuclear inclusions A. The last 5 days of incubation, E. Stimulation of cell proliferation prodrome, the period of eruption 560. In a study of the sick 5 years old B. The last day of incubation, child by the dentist on the basis of prodrome clinical data has been diagnosed with

82 mumps. Which of the following C. The reaction of agglutination methods can be transmitted the D. The reaction of precipitation disease? E. The reaction of opsonization A. Airborne 564. Serological diagnostics of B. Transmissible influenza involves the identification C. Animals through the bite of growth titer of antibodies to the D. Through fecal-oral patient serum. How many times E. Parentheral should the antibody titer increase in 561. To prevent seasonal flu rise in paired sera,that results are considered hospitals of the city Sanitary credible? Epidemic Station obliged to A. In 3 times immunize health care workers. What B. In 2 times of the following drugs should be C. In one time immunized? D. In 4 or more times A. Subunit vaccine E. In a half-titer B. Interferon 565. The influenza virus contains an C. Gamma globulin internal antigens - the nucleoprotein D. Remantadin (NP), polymerase (P1, P2, P3), matrix E. Amantadine protein (M) and external antigens - 562. In the infectious diseases hemagglutinin (H) and neuraminidase hospital the patient admitted with (N). Some of them a key role in signs of pneumonia, which developed establishing immunity to influenza on the 6th day of the flu. Which infection? method is most authentically A. Hemagglutinin and neuraminidase confirmed influenza etiology of B. Nucleoprotein antigens pneumonia? C. Matrix protein A. Detection of influenza virus D. polymerase proteins antigens in sputum by ELISA E. Neuraminidase B. The study paired sera 566. Post-mortem department of C. Infection of chick embryos infectious diseases hospital the corpse D. Immunofluorescent study smears of a dead person was suddenly sent from the nasal passages with a clinical diagnosis "flu." What E. Detection of antibodies against research is needed to confirm influenza virus hemagglutinin specified for a reliable diagnosis? 563. During an outbreak of acute A. Electron microscopic detection of respiratory infection in order to viruses establish the diagnosis of influenza B. Isolation and identification of rapid diagnosis is carried out, based influenza virus on the identification of specific viral C. Detection of intracellular antigen in the test material (nasal inclusions at light microscopy wash). What serological tests used for D. Isolation of high titers of this? antibodies in the reaction of A. Immunofluorescent assay hemagglutination -inhibition B. The reaction of complement E. Isolation of antibodies in the fixation complement-fixation

83 567. The boy of 6 years old of monolayer, infected with infectious moderate fever, enlarged parotid material, glands. A virus of saliva was isolated physician assistant diagnosed the that replicates in chicken embryos or respiratory- syncytial virus infection. cell cultures, has hemagglutination What changes will this virus cause in properties and causes the formation of cell culture? simplast in cell culture. What other A. Total destruction of the cell organs are most likely to be affected monolayer as a consequence of infection with B. Round cellular degeneration this virus? C. The formation of polynuclear cells A. Lungs D. The presence of cell Babesh- Negri B. Liver E. Detachment of the monolayer C. Gonads 572. The department has increased D. pharyngeal tonsils dramatically the incidence of newborn E. Brain children SARS, caused by the 568. The epidemic of flu is in the city. different groups of viruses. In order to Which of the following drug can be prevent the spread of infection are recommended to people for non- recommended appointment of human specific prevention of the disease? leukocyte interferon. Which way is A. Leukocyte interferon this product introduced? B. Flu vaccine A. Subcutaneously C. Penicillin B. In the nasal passages D. Influenza immunoglobulin C. Inhalation E. Flu serum D. Oral 569. To prevent seasonal flu which E. Intramuscularly rises in the educational institutions of 573. What strain of influenza virus the city-station must hold an active with a pandemic antigenic structure is immunization of children and caused now? adolescents. What of the following A H1N1 drugs should be immunized? B. H3N4 A. Killed vaccine C. H1N2 B. Normal human immunoglobulin D.H2N3 C. Rimantadine E. H2N2 D. Oxolin 574. Boy, 1.5 years old, who did not E. Interferon receive routine vaccinations in contact 570. In the kindergarten routine with the measles. For the purpose of measles vaccination were conducted. emergency specific prevention of What method can verify the formation child was entered donor gamma of postvaccinal immunity? globulin. What type of immunity was A. Serological established at the same time? B. Virological A. The natural C. Bacteriological B. Passive D. Bacterioscopic S. antitoxic E. Allergic D. Post-vaccination 571. When virusoscopy cell E. Local

84 575. What type of influenza virus has E. Adenoviruses the greatest antigenic variation? 579. For effective vaccination is A. A necessary to provide probable virus B. B that causes epidemic. As a rule, the C. C agent is the variant of the virus D. D against which most people have no E. E antibodies. What kind of reaction is 576. A child 7 years has acute illness. advisable to apply for the detection of During the inspection it was revealed: antibodies in the sera of people? that the hyperemia and edema of the A Complement fixation pharyngeal mucosa, which is covered B. The indirect hemagglutination with lots of mucus. On the mucous C. Passive hemagglutination membrane of cheeks whitish spots. D. Neutralization of the Then a blotchy rash appeared on the cytopathogenic child‟s face, neck and body. What is E. Нemagglutination inhibition the disease most likely evolved in this 580. From individuals with case? respiratory infection clinic, residents A. Meningococcemia of south-east Asia, has been isolated B. Measles influenza virus from sharp changes in C. Atopic the surface glycoproteins - D. Scarlet fever hemagglutinin and neuraminidase. E. Diphtheria The laboratory staff regarded the virus 577. From the patient's clinical as a virus with pandemic potency. In material (flush with the nasopharynx), the result of which process could form with a preliminary diagnosis of an antigenic variant of influenza influenza epidemiology, conduct virus virus? isolation in chicken embryo. Which of A. Antigenic shift these reactions can be used to detect B. Antigenic drift the virus in the chick embryo? C. Conjugation A. The reaction of hemagglutination D. Transduction inhibition (RGGA) E. Transformation B. The reaction of agglutination (PA) 581. In a 32-year-old man 'flu-like C. The reaction of indirect syndrome" developed with fever, hemagglutination (Phragmites) headache, sore throat and muscle D. The reaction of hemagglutination aches. Which of the following is the (RHA) best studied material for virus E. The reaction of immune hemolysis isolation - the agent of this infection? 578. In the diagnosis of salivary gland A. Feces inflammation in the patient was B. Nasopharyngeal wash isolated mumps virus. Which family C. Urine is this a virus? D. Blood A Paramyxovirus E. Saliva B. Ortomyxovirus 582. The kindergarten child got sick C. Picornavirus of measles. What medication can D. Herpes simplex virus prevent measles outbreak in a group?

85 A Human immunoglobulin, normal the cells: B. Measles vaccine A. Paschen C. Sulfonamides B. Babesh- Ernst D. Interferon C. Prowazek E. Antibiotics D. Gvarnier 583. The patient has the flu where a E. Babesh-. Negri defeat of the epithelial cells of the 588. To detect the cells of Babesh- nasal mucosa is, which helps activate Negri by histological method the the body autoflora and the emergence staining is used: of secondary bacterial infection. What A. By Neisseria kind of bacteria can cause B. By Gins complications? C. By Morozov A. Coli bacillus D. By Turevich B. Pseudomonas aeruginosa and E. By Peshkov Proteus 589. For specific rabies is applied: C. Bacillus and Clostridium A. Killed vaccine D. Staphylococcus and Streptococcus B. Live vaccine E. Fuzobacteries and Treponema C. Anatoxin 584. The incubation period is a D. Chemical vaccine minimum in the penetration of the E.Antiidiotypic vaccine agent of rabies through the damaged 590. The patient turned to the clinic skin: because of dog‟s bites. The dog A. Head managed to catch it, and it turned out B. Neck that the animal is sick with rabies. C. Upper limb Which vaccine should be used for D. Body specific prophylaxis of rabies in E. The lower extremities humans? 585. The incubation period is the A. Chemical maximum penetration of the agent of B. Anatoxin rabies through the damaged skin: C. Living A. Head D. Recombinant B. Neck E. The synthetic C. Upper limb 591. A man turned to the surgery who D. Body was bitten by unknown dog. E. The lower extremities Extensive lacerations are located on 586. For the laboratory diagnostics of the face. What is the treatment and rabies the methods are used: prophylaxis care is required to A Histological, allergic, biological provide for the prevention of rabies? B. Immunofluorescent, histological, A. Post immunized with rabies biological vaccine C. Serological, biological, and allergic B. Assign a combined antibiotic D. Biological, allergic, histological C. Urgently introduce DTP E. Allergic, serological, D. To hospitalize and keep under the immunofluorescent supervision of a physician 587. Specific for rabies is to identify E. Extra enter the normal gamma-

86 globulin the rabies observed luminosity. What 592. For which of these diseases are is defined for a given reaction? characterized by the cell of Babesh- A. The cells of Paschen Negri in host cells? B. Antibodies A. Aseptic meningitis C. The cells of Babesh- Negri B. Rabies D. Corn Fly C. Congenital Rubella E. Corn volutin D. mumps 599. The patient with a torn wound E. Infectious mononucleosis after being bitten by rabid animal 593. The rabies virus belongs to the admitted to the hospital. Which family: vaccine should be introduced for A. Reovirus prophylaxis of rabies? B. Retrovirus A. Rabies vaccine C. Togavirus B. DTP D. Picornavirus C. DT E. Rhabdovirus D. BCG 594. In Ukraine, the main source of E. TABte human infection with rabies is: 600. Hunter turned for medical help A Sick Man because of being bitten by the B. Dogs wounded fox. What kind of help S. Wolf should be given to him in order to D. Fox prevent rabies specific? E. Cattle A. Introduction of rabies vaccine 595. Entrance gates in rabies are: B. Surgical treatment of wounds A. Upper respiratory tract C. The introduction of antibiotics B. The intestinal tract D. Washing the wound with soap and C. Damaged skin water D. Reproductive organs E. Treatment of wounds with an E. The salivary glands alcoholic solution of iodine 596. The body of a sick man rabies 601. A child of 2 years of catarrhal virus stands out: conjunctivitis, pharyngitis, A. From the saliva larengotraheobronhitis appeared., The B. From the feces white spots have appeared on the C. From urine mucous membrane of the cheeks, and D. From the nasopharyngeal mucus later spotted rash on the face, trunk, E. Not released extremities. What kind of illness 597. In the patient's body is fixed should think about? rabies virus in the cells: A. Influenza A. Cardiovascular System B. Fever B. Nervous System C. Measles C. Liver D. Scarlet fever D. Blood E. E. Skin 602. The patient has the flu and there 598. In setting up the FTA with the is a defeat of the epithelial cells of the slices of dog‟s brain died because of nasal mucosa, which helps activate

87 the body autoflora and the emergence Laboratory diagnostics of HIV of foci of secondary bacterial infection. The defeat of the oral infection. What kind of bacteria can cavity at AIDS cause complications? 606. In a study of young man in the A. Bacillus and Clostridium middle of the fight against AIDS has B. Pseudomonas aeruginosa and been obtained positive results with Proteus ELISA antigens of HIV. Complaints C. Staphylococcus and Streptococcus about the state of health are not D. Coli bacillus available. What may indicate a E. Fuzobacteries and Treponema positive result of ELISA? 603. From individuals with A. About HIV respiratory infection clinic, residents B. About AIDS of south-east Asia, has been isolated C. About the HBV infection influenza virus from sharp changes in D. About the disease of AIDS the surface glycoproteins - E. About the persistence of HBV hemagglutinin and neuraminidase. 607. The diagnostics of HIV infection The laboratory staff regarded the virus examined serum for detection of as a virus with pandemic potency. As specific antibodies by ELISA. What a result of what process could form an enzyme labeled antibodies are used in antigenic variant of influenza virus? this case? A. Transformation A. Against the gp120 protein B. Antigenic drift B. Against HIV antigens C. conjugation C. Against the human D. Transduction immunoglobulin E. Antigenic shift D. Against protein gp 17 604. In a 32-year-old man 'flu-like E. Against protein gp 41 syndrome "developed with fever, 608. Human immunodeficiency virus headache, sore throat and muscle differs from other viruses by: aches. Which of the following is the A. The complexity of the structure best material under study to isolate B. The presence of reverse the virus agent of this infection? transcriptase A. Nasopharyngeal wash C. The ability to integrate into the B. Feces genome of the cells C. Urine D. The presence of two types of D. Blood nucleic acids - DNA and RNA E. Saliva E. The ability to grow in chick 605. The kindergarten child got sick embryo of measles. What medication can 609. The main method of laboratory prevent measles outbreak in a group? diagnostics of HIV infection is A measles vaccine A. Allergic B. Human immunoglobulin normal B. Virusological C. sulfonamides C. Biology D. Interferon D. Serological E. Antibiotics E .Electron microscopy 610. Target cells for HIV are:

88 A. T-killers B. Immunoblot (immunoblotting) B. T-suppressor C. Enzyme Immunoassay C. T-helper D. Immunofluorescence D. B-lymphocytes E. Clinical and immunological study E. NK-cells 616. At autopsy of the deceased in 611. After infection with HIV the the skin of distal lower extremities antibodies appear in serum in: revealed crimson-red patches, A. 2-3 weeks plaques, and nodes (Kaposi's B. 3-6 months sarcoma). Identified as an acute C. 10-12 months pneumonia caused by D. 3-6 years pnevmotsistami. What disease is E. 8-10 years characterized by the changes? 612. The patient was informed that A. Influenza serological testing for AIDS consists B. Diphtheria of two stages. What reaction is used C. Measles to study serum antibody against the D. Anthrax virus in the first stage? E. AIDS A. RIA 617. HIV belongs to the family: B. Immunoblotting A Picornavirus C. ELISA B. Reovirus D. FTA C. Rhabdovirus E. RNTA D. Retroviruses 613. What kind of protein of HIV is E. paramyxovirus bounded the mechanism of reverse 618. HIV belongs to the subfamily: transcription? A. Oncoviruses A. Endonuclease B. Lentiviruses B. Protease C. Spumavirus S. Neyrominidaza 619. LTR of proviral HIV DNA D. RNA polymerase encodes: E. Revertaze A. Internal structural proteins 614. Specify the transmission of HIV. B. Regulatory proteins A. Respiratory C .Type-specific protein of the outer B. Contact shell C. Alimentary D. Do not encode any proteins D. The parenteral E. Virus specific enzymes E. Transmissible 620. The gene env of HIV proviral 615. At the initial examination of DNA encodes: blood donors at blood transfusion A. Internal structural proteins stations ELISA to detect antibodies to B. Regulatory proteins human immunodeficiency virus with C.Type-specific protein serum from one of them produced a D. Do not encode any proteins positive result. Which method is E. Virus specific enzymes recommended to confirm the 621. The gene pol of HIV proviral diagnosis of HIV infection? DNA encodes: A. Electron Microscopy A. Internal structural proteins

89 B. Regulatory proteins B. The involution of parathyroid C .Type-specific protein of the outer gland, activation of B cells, reducing shell levels of interleukin-2 and interferon D. Do not encode any proteins C. The involution of parathyroid E. Virus specific enzymes gland, activation of B-lymphocytes, 622. The greatest variability of increased levels of interleukin-2 and proviral DNA are the genes of HIV: interferon A env D. Hyperfunction of parathyroid B. pol inhibition of B-lymphocytes, reducing C. gag the level of interleukin-2 and D. sor interferon E. tat E. The involution of parathyroid 623. Very high level of variability of gland, activation of B-lymphocytes, HIV is associated with proteins: increased levels of interleukin-2 and A. p24 interferon B. p18 628. In AIDS patients are observed: C. p13 A. Reduced levels of IgG and IgA, the D. gp120 number of immune complexes, E. gp160 interleukin-2 624. Major role in the interaction of B. Reduction of IgG and IgA, HIV with target cells is: increased number of immune A. p24 complexes, interleukin-2 B. p18 C. Increased levels of IgG and IgA, C. gp41 the number of immune complexes, D. gp120 interleukin-2 E. gp160 D. Increased levels of IgG and IgA, 625. In healthy people the ratio of reduction of immune complexes, T4/T8 is as follows: Interleukin-2 A. 0, 2-0, 5 E. Increased levels of IgG and IgA, B. 0, 7-1, 2 the number of immune complexes, C. 1, 9-2, 4 reduction of interleukin-2 D. 2, 5-3, 2 629. The structure of HIV processes E. 3, 5-4, 2 include: 626. In patients with AIDS the ratio A. gp120, gp41 of T4/T8 is: B. Hemagglutinin, neuraminidase A. 0, 2-0, 9 C. Hyaluronidase, coagulase B. 1, 9-2, 4 D. Hemolysin, coagulase C. 2, 5-3, 2 E. p24, p17 D. 3, 5-4, 2 630. For specific prophylaxis of AIDS E. 4, 5-5, 0 is used: 627. In AIDS patients are observed: A. Live vaccine A. The involution of parathyroid B. Killed vaccine gland, inhibition of B-lymphocytes, C. Toxoids reducing the level of interleukin-2 D. Chemical vaccine and interferon E. Not developed

90 631. High risk for HIV infection AIDS. What are the diagnostic include: methods must be used to confirm the A. Patients with chronic medical diagnosis? conditions A. Immunofluorescence assay, B. Recipients of blood immunoblotting, C. Patients with tuberculosis B. Complement fixation test D. Parturient C. Agglutination test E. Blood Donors D. Reaction gemadsorbtion 632. If any clinical signs of disease E. The reaction of hemagglutination and showed an HIV test: 637. When testing blood donors for A. Acute Diarrhea blood transfusion station in the serum B. Acute pneumonia of one of them had antibodies to C. The increase in groin human immunodeficiency virus. D. Increase in cervical lymph nodes Which method is recommended to E. The increase in lymph nodes of confirm the diagnosis of HIV several locations infection? 633. For the first time AIDS was A. Immunofluorescence registered as an infectious disease in B. Electron microscopy man: C. ELISA A. 1977 D. Immunoblot (immunoblotting) B. 1979 E. C. 1981 638. HIV-infected patient is examined D. 1983 periodically for signs of revitalization E. 1985 process. What are the most significant 634. The results of the allocation of sign that points to the transition of AIDS pathogen group of French HIV infection to AIDS. scientists were first published in: A. Kaposi's sarcoma. Number of T- A. 1977 helper cells below 200 cells / mm B. 1979 B. Reducing the number of C. 1981 neutrophils D. 1983 C. Reducing the number of T-helper E. 1985 D. Number of T-helper cells below a 635. Priority in allocation of AIDS critical level pathogen belongs to a group of E. Detection of antibodies to gp 41 researchers led by: 639. In a specialized clinic patients A. P. Frosch and T. Francis assigned the combination of drugs B. P.Parkmen and T. Uellerg that inhibit the reproduction of HIV. C. G. Doldorf and G. Sikls Indicate which group includes drugs D. L.Montan‟ye and R. Gallo that are necessarily comprehensive E. D. Enders and E. Popper antiviral treatment. 636. In a patient with fever of A. Crixivan unknown etiology, B. Broad-spectrum antibiotics immunodeficiency, C. Interleukin damage to the nervous and digestive D. nucleoside analogues systems previously diagnosed with E. Biseptol

91 640. A patient was treated for a long What illness occur this protein? time for pneumonia of unknown A. Syphilis etiology, resistant to standard therapy. B. Viral hepatitis B Anamnesis revealed that he was a C. Tuberculosis long time on a business trip in the D. HIV infection U.S.. While traveling he was injured, E. Polio was treated in hospital after his 644. The patients turned to the dentist recovery he returned home. with manifestations of oral Evaluating the medical history, candidosis, which always recurs and clinical picture, the doctor suspected no cure. When questioning revealed AIDS in patients. Results of what that the patient has a long time there laboratory diagnostic methods let is fever, weight loss. What research confirm that pre-diagnosis in this should be done to the patient? patient? A. Immunological and serological A. Widal reaction tests for HIV infection B. Immunosorbent assay B. Bacteriological studies on C. Complement fixation test dysbiosis. D. Electron Microscopy C. Select a pure culture of the E. HAI-hemagglutination inhibition pathogen and to investigate the test sensitivity to antibiotics. 641. The patient had been diagnosed D. Check the status of humoral AIDS for 20 years -. What is the cell immunity. population most sensitive to human E. Investigate the level of specific immunodeficiency virus? antibodies to the fungus Candida A. Epithelial cells 645. Acquired immunodeficiency B. Hepatocytes syndrome (AIDS) is the human C. Endothelial cells immunodeficiency virus. Which cells D. T helper are targets for this virus?: E. B-lymphocytes A. Eosinophils 642. It is known that human B. Natural killer cells immunodeficiency virus belongs to C. T-helper lymphocytes the family D. Basophiles retroviruses. Specify the main features E. Plasma which characterized the family. 646. Human immunodeficiency virus, A. The presence of the enzyme which has on its surface antigens gp reverse transcriptase 41 and gp 120 interacts with the target B. Contain less RNA cells of the body. Choose among these C. Simple viruses that infect only antigens of human lymphocytes, human which complementarily binds gp 120 D. The nucleic acid does not integrate virus. into the host genome A. CD 3 E. The reaction of enzyme B. CD 4 immunoassay antigen detection C. CD 8 643. In carrying out Western blot test D. CD 19 in serum was found protein gp120. E. CD 28

92 647. In the study of blood serum in fungi of the genus Candida. When a patients with signs of disease is found multiple infections immunodeficiency were identified opportunistic microorganisms? antibodies of proteins gp 120 and gp A. AIDS 41. What infection in a patient B. Diabetes confirms this result? C. Sepsis A. HIV infection D. dysbacteriosis B. NLTV-1 infection E. Medical agranulocytosis C. TORCH-infection 651. Retroviruses are unique enzyme, D. HB - infection which distinguishes them from other E. ECHO-infection viruses. Which of the following 648. After laboratory examination of enzymes of retroviruses replicate? patients with recurrent common viral, A. RNA-dependent DNA polymerase bacterial and fungal opportunistic B. DNA-dependent RNA polymerase infections diagnosed "HIV infection". C. DNA polymerase The results of investigations made it D. RNA polymerase possible to put such a diagnosis? E. Restrictase A. Immunosorbent assay. 652. To the city center of research an B. Complement fixation test. anonymous citizen N. has addressed C. Gel precipitation test. to conduct a survey on HIV infection, D. The reaction of hemagglutination after which the patient's blood was inhibition. sent to the laboratory for serological E. The reaction of passive diagnosis. Which of these reactions hemagglutination. must take place in this case? 649. In a patient with clinical signs of A. Enzyme Immunoassay immunodeficiency were conducted B. The reaction of indirect immunological studies. Revealed a hemagglutination significant decrease in the number of C. The reaction of complement cells that form rosettes with sheep fixation erythrocytes. What conclusion should D. Immunofluorescence reaction be based on data analysis? E. The reaction of neutralization A. Reduction of comlement 653. Patient at the age of 22 was B. Reduction of B-lymphocyte hospitalized at the clinic complaining C. Reduction of natural killer (NK- of a sharp weight loss, general cells) weakness, swollen lymph nodes. Over D. Reduction of T-lymphocytes the past few months suffering from E. Lack of effector cells of humoral diarrhea. Laboratory researches immunity established radiation in the faeces in 650. The patient has 25 years of large numbers is found multiple skin pustules planted Cryptosporidium. In a detailed Staphylococcus aureus in association clinical study of AIDS, diagnosis was with Staphylococcus epidermidis, the proved. Which of the immunological study of sputum revealed characteristics are important for this pnevmotsista Carini in the faeces - disease? cryptosporidium, vulgar Proteus and A Сhange in the correlation between

93 T-and B-lymphocytes Laboratory was provided for B. The sharp decline in activity of the diagnosis of HIV infection. What complement system diagnostic tools must a laboratory be С. Lymphopenia equipped with? D. Polyclonal activation B - A test system for detection of specific lymphocytes antibodies by ELISA E. The absolute reduction in the B. Cell cultures for virus isolation number of T - helper cells C. Sets of special culture media 654. In a patient with clinic D. Standard diagnostic sera pneumonia physician suspected HIV - E. Antigen and hemolytic system to infection. What laboratory tests can complement-fixation confirm a diagnosis of "HIV - 658. AIDS patient asked the dentist infection" in this patient? about the destruction of the oral A. ELISA screening of serum mucosa. The defeat was manifested B. Bacteriological examination of with white plaques, which are merged sputum into a continuous "layering cheese", С. Mycological examination of usually near the corners of the mouth sputum and on the back of the tongue. What is D. Virological examination of sputum the diagnosis of such lesions of the E. Electron examination of sputum oral mucosa of AIDS posed a doctor? 655. A young man at the age of 25 A. Candidosis was first applied to the study of trust B. Herpetic stomatitis with the request to delete his HIV C. Necrotizing stomatitis Vincent infection. What reaction can be used D. Foot and mouth disease to diagnose HIV infection in this E. Gonococcus stomatitis patient? 659. In the dental patient, a man at the A. Enzyme immunoassay (ELISA) age of 49, has been observed for B. Reaction immunoblotting necrotizing gingivitis, the defeat of С. Radioimunny analysis (RIA) the "thrush" caused by Candida, dry D. Immunoelectronnic microscopy mucous membranes, and bilateral (IEM) tumors salivary glands. Consequence E. Immunofluorescence reaction of which immune disorders most (IFA) likely to be that defeat? 656. After the examination of patients A. AIDS with recurrent aphthous stomatitis and B. Primary immunodeficiency candidosis associated physician combinable decided to exclude HIV infection. C. Allergic disease What research will help clarify the D. Autoimmune disease situation and a preliminary diagnosis? E. Selective gammaglobulinemia A. Enzyme Immunoassay 660. In connection with cases of B. Gel precipitation test diagnosis of HIV infection in the C. The reaction of hemagglutination dental clinic a meeting was held, at inhibition which the recommendations were D. The reaction of hemagglutination made by doctors, epidemiologists, 657. In the dental clinic, the AIDS Prevention Center. What

94 preventive measures could dentists enterovirus infection, foot and recommend preventing the possibility mouth disease of professional infection? 664. In the infectious diseases A. The requirement of each patient's hospital admitted the patient with presenting medical certificate clinical signs of encephalitis. In the B. Wash your hands after each patient history – the tick bite. In C. For each patient, using a new hemagglutination-inhibition tests sterile set of instruments revealed antibodies against the D. To work in personal protective pathogen in tick-borne encephalitis equipment (gloves, goggles) dilution of 1:20. Enter the following E. To quartz the room steps after receiving the doctor's 661. What method of testing blood results: donations should be applied to ensure A. Use a more sensitive reaction the absence of her immunodeficiency B. Use this serum again virus? C. Repeat the study with serum taken A. Electronic microscopic study of T4 after 10 days lymphocytes D. Repeat the study with other B. Detection of the virus by infecting diagnostic cultures of lymphocytes E. Confirm the diagnosis of tick C. Detection of IgM antibodies encephalitis D. To conduct nucleic acid agent 665. Circulation in the nature of E. Identification of pathogen antigens and arthropods 662. In an observational branch of the transmissible transmission hospital HIV-infected women gave characteristic of the man: birth to a child. The use of which of A Picornavirus the following methods of diagnosis of B. Ortomyxovirus HIV infection in the newborn is the C. Poxvirus most appropriate? D. Filovirus A. RIA E. Flavivirus B. ELISA 666. For serological diagnosis of C. RVN polio examined the paired sera of the D. DCs patient. What should be used as an E. PCR antigen in the neutralization of the 663. At what stage of HIV infection is cytopathogenic? a source of human infection? A. Antigens - haemagglutinin A. At all stages of HIV infection B. Complement-antigens B. Acute HIV infection C. The antigens inactivated with C. Chronic (latent) HIV infection formalin D. Persistent generalized HIV D. Living the three types of viruses E. AIDS - associated complex E. Antigens of the capsid proteins of the virus Laboratory diagnosis of 667. A child at the age of 5 enterovirus, aftovirus and hospitalized at the 2nd day of illness coronavirus infections. The defeat with complaints of fever, weakness, of the oral cavity in terms of gastrointestinal upset. In history:

95 contact with patients with Sebina: poliomyelitis. Preliminary diagnosis: A. Provide general humoral immunity polio. What is the pathological but do not provide local immunity of material taken for laboratory testing the intestine, is introduced and what method is used for parenterally microbiological diagnosis? B. Do not provide a general humoral A Virological, stool. immunity, but provide local immunity B. Liquor. Virological of the intestine, enter per os C. Blood. Biological C. Do not provide a general humoral D. Stools. immunity, but provide local immunity E. Blood. Virological of the intestine, is introduced 668. For the specific prevention of parenterally polio are used: D. Provide a common humoral and A. Ribosomal vaccine local immunity of the intestine, is B. Live vaccine introduced parenterally C. Toxoids E. Provide a common humoral and D. Bacteriophage local immunity of the intestine, enter E. Chemical vaccine per os 669. Enteroviruses belong to the 674. The properties of the Salk polio family: vaccine: A Picornavirus A. Provide general humoral immunity B. Poxvirus but do not provide local immunity of C. Paramyxovirus the intestine, is introduced D. Ortomyxovirus parenterally E. Reovirus B. Do not provide a general humoral 670. Polio viruses belong to the immunity, but provide local immunity genus: of the intestine, enter per os A. Rhinoviruses C. Do not provide a general humoral B. Cardiovirus immunity, but provide local immunity C. Aftovirus of the intestine, is introduced D. Rubivirus parenterally E. Enterovirus D. Provide a common humoral and 671. What is the serotype of the virus local immunity of the intestine, is of polio often causes epidemics: introduced parenterally A . I E. Provide a common humoral B. II immunity but do not provide local C. III immunity of the intestine, enter per os 672. Poliomyelitis virus has organ 675. Coxsackie viruses belong to the tropism: genus: A. Cardiovascular System A. Rhinoviruses B. Nervous System B. Enteroviruses C. Salivary glands C. Cardiovirus D. Liver D. Aftovirus E. Blood E. Rubivirus 673. Properties of poliovirus vaccine 676. The main route of transmission

96 of diseases caused by Coxsackie 682. ECHO viruses have tropism for virus: Organ: A. Alimentary A. Cardiovascular System B. The parenteral B. Nervous System C. Sexual C. The respiratory system D. Contact-home D. Gastrointestinal tract E. Transplacental E. Multiorgan tropism 677. Coxsackie virus has tropism for 683. What is the mechanism of Organ: transmission of polio: A. Cardiovascular System A. Spray B. Nervous System B. Fecal-oral C. The respiratory system C. Transmissible D. Gastrointestinal tract D. Contact E. Multiorgan tropism E. Vertical (transplacental) 678. For the specific prophylaxis of 684. The patient has 5 year old boy on diseases caused by Coxsackie virus is the fifth day after the onset of nasal used: mucosa and tonsils isolated agent of A. Live vaccine poliomyelitis. What kind of reaction B. Killed vaccine should be used to determine the C. Toxoids serotype of agent? D. Chemical vaccine A. The reaction was neutralized viral E. Not developed cytopathic effect 679. ECHO viruses belong to the B. Haemagglutination-inhibition tests genus: C. Reaction inhibition gemadsorbtion A. Rhinoviruses D. Indirect hemagglutination reaction B. Enteroviruses E. Complement fixation test C. Cardiovirus 685. The stools are sick with D. Aftovirus suspected intestinal virus infection E. Rubivirus treated with antibiotics for one hour at 680. For the specific prophylaxis of 40C. Then a suspension of infected diseases caused by viruses ECHO primary and continuous cell culture. apply to: After 2-3 days in the infected cells A. Live vaccine showed cytopathic effect of cultures. B. Killed vaccine How is the identification of C. Toxoids enteroviruses held? D. Chemical vaccine A. Immunofluorescence reaction E. Not developed B. Through neutralization of the 681. The main route of transmission cytopathic type-specific enterovirus of diseases that are caused by viruses sera of ECHO: C. Through the haemagglutination- A. The parenteral inhibition tests B. Alimentary D. Use of agglutination C. Contact and household E. With the precipitation reaction D. Sexual 686. From the patient with acute E. transplacental intestinal infection the virus was

97 divided which is referred to the genus class will dominate the response to Enterovirus. To establish the serotype the introduction of this vaccine? of the virus diagnostic serum is used. A. Іg A Indicate which of these antibodies B. Іg M should contain them? C. Іg E A. Against viral enzymes D. Іg D. B. Against the protein supercapsid E ------shell 690. Child, 5 months, carried out C. Against non-structural proteins of vaccination for prevention of polio the virus vaccine by oral administration. What D. Against the capsid proteins does the vaccine have in its E. Against viral haemagglutinin composition? 687. In the children's infectious A. Formalin Killed viruses disease section of the city hospital B. Live viruses was hospitalized group of children C. heat-killed virus with pre-set clinical diagnosis of D. Elements of the viral capsid "gastroenteritis rotavirus." What is the E. Virus reverse transcriptase basic principle laboratory diagnostic 691. As a consequence of applies in this case? contamination of cell cultures of A. Detection of viral antigen in the pathological material, which is taken test material from a sick child, isolated type I B. Identification of specific inclusions poliovirus? What is characteristic of in the cells. this type of virus is responsible? C. Infection of chick embryos A .DNA containing a simple virus D. Isolation of the virus in cell culture B. Retrovirus with subsequent identification C. RNA - containing a simple virus E. Infection of laboratory animals D. DNA - containing complex virus 688. Material from the patient A. E. RNA - containing complex virus 4y.o. with suspected enterovirus 692. Go to the dentist asked the infection was infected with cell patient (the milkmaid), with culture for the purpose of precipitation in the oral mucosa in the accumulation of the virus. What type form of the AFL. The doctor revealed of tissue culture cells was presented skin rash hands around the nail plate. tumor and is able to divide What pathogen caused the disease? indefinitely? A. The virus Coxsackie B A. Inculcate cell lines B. Herpesvirus B. Organ Culture C. Vesicular stomatitis virus C. Culture of primary cells D. FMDV D. Lines of diploid cells E. Cytomegalovirus E. None of these 689. The kindergarten children Laboratorial diagnostics of performed vaccination to prevent hepatitis A, B, C, D, E polimielita. The vaccine was 693. In epidemiology of which from administered orally. Synthesis of viral hepatitis is there the expressed antiviral immunoglobulins which seasonality?

98 A. Hepatitis G most dangerous as a source of B. Hepatitis B infection? C. Hepatitis C a. At the end of latent period D. Hepatitis D b. In full play of illness with the E. Hepatitis A period of jaundice 694. The main way of transmission of c. At the end of latent period and in hepatitis A is: full play of illness before jaundice A. Inoculable appearance B. Airborne d. At the end of latent period and in C. Parenteral full play of illness with the period D. Contact of jaundice E. Alimentary e. At the end of latent period, during 695. The virus of hepatitis A belongs all illness to the family: 699. The infection atrium for A. Picornaviruses hepatitis A virus is: B. Poxviruses a. Mucous membrane of respiratory C. Раrаmyxoviruses tracts D. Orthomyxoviruses b. Mucous membrane of digestive E. Reoviruses tract 696. The virus of hepatitis A belongs c. Mucous membrane of urinary to the genus: system A. Rhinoviruses d. Skin B. Cardioviruses e. Salivary glands C. Aphthoviruses 700. The viral hepatitis A usually D. Enteroviruses damage: E. Rubiviruses a. Children before 14 years old 697. Resistance of virus of hepatitis A b. Adults outrank 30 years old to the factors of environment is: c. Young people and adults (15-30 A. Unstable to the high temperature, years old) to the low temperature, to the action d. Elderly people (much than 60 of chlorine years old) B. Unstable to the high temperature, e. Age dependence does not exist to the action of chlorine, proof to the 701. Mechanism of HAV action to low temperature hepatocytes and clinical C. Proof to the high temperature, to manifestation of viral hepatitis A the low temperature, unstable to the are: action of chlorine a. Has direct cytopatical action to D. Proof to the high temperature, hepatocytes, serious forms of unstable to the low temperature, to the illness - acute or chronic with early action of chlorine formation of hepatic cirrhosis E. Proof to the high temperature, to b. Has direct cytopatical action to the low temperature, to the action of hepatocytes. Often disease is over chlorine with full recovery 698. In what periods of c. The mechanism of action is disease a patient on viral hepatitis A is unknown, 50 % of all forms of a

99 hepatitis pass in the chronic form, d. HBcAg and HBeAg and usually it leads to cirrhosis and e. HBcAg and HBsAg to a liver cancer 705. The main way of hepatitis D d. The mechanism of action is transmission is: unknown; the serious form of a. Transplacental infection may be especially at b. Alimentary pregnancy c. Contact e. Has meditational immunological d. Sexual action to hepatocytes; usually e. Parenteral chronically carriage is 706. The main way of hepatitis E characterized as a persistent or transmission is: chronically active hepatitis; may a. Parenteral be without symptoms b. Alimentary 702. A patient with hepatitis A is c. Contact hospitalized to the infection d. Sexual diseases hospital. What types of e. Transplacental antibodies will be synthesized first 707. The main way of hepatitis C during the reproduction of this transmission is: virus? a. Parenteral a. IgA b. Alimentary b. IgM c. Contact c. IgE d. Sexual d. IgG e. Transplacental e. IgD 708. The causative agent of 703. 703.There is a hepatitis eruption is hepatitis D (delta-agent) is a fixed in a settlement. The water factor defective virus. It can replicate of transmission is the main. What kind only in ceils already infected of viruses can be a cause of eruption? with one of the viruses. With a. Hepatitis D which one? b. Hepatitis C a. By virus of hepatitis B c. Hepatitis B b. By virus of hepatitis A d. Hepatitis A c. By Epstein-Barr virus e. No one of hepatitis A or hepatitis d. By virus of hepatitis E B e. By HIV 704. Three samples of blood sera 709. A patient with a very grave were delivered to the laboratory: course of hepatitis B has been from the patient with chronic administered the examination for hepatitis B, second one recovered detecting the possible after hepatitis B and third one concomitant agent, which without hepatitis B symptoms. complicates the course of the What kind of antigens will be basic disease. What agent is revealed in all of samples? this? a. HBsAg a. Virus of hepatitis A b. HBeAg b. Δ-virus c. HBcAg c. Virus of hepatitis G

100 d. Virus of hepatitis E d. Urine e. HBs-antigen e. All biological fluids 710. The main way of hepatitis B 715. Resistance of hepatitis B virus transmission is: to factors of external environment a. Parenteral is: b. Alimentary a. Nonresistant to high and low c. Contact temperature, to disinfectant d. Sexual substances e. Transplacental b. Nonresistant to high temperature 711. Patient after a motor-car and disinfectant substances; accident was delivered to the resistant to low temperature hospital. He needs blood c. Resistant to high and low transfusion. What reaction is it temperature, to disinfectant possible to use for a presence of substances the hepatitis B virus antigen in a d. Resistant to high temperature; donor blood revealing? nonresistant to low temperature a. Inhibition of hemagglutination test and disinfectant substances b. Immunoenzymatic test e. Resistant to high and low c. Inhibition of hemadsorption test temperature; nonresistant to d. Indirect-hemagglutination test disinfectant substances e. Complement-connecting test 716. The infection atrium for 712. During an operative hepatitis B virus is: intervention a patient had blood a. Mucous membrane of respiratory transfusion. In his blood the tracts presence of the following b. Mucous membrane of digestive causative agent should be tract checked: c. Mucous membrane sexual ways a. Hepatitis A virus d. Blood b. Enteroviruses e. Skin c. Hepatitis B virus 717. The patient with hepatitis B is d. Hepatitis C virus dangerous in: e. Adenoviruses a. The end of incubation period 713. The antigen structure of b. The end of incubation period and hepatitis B virus consists of height of disease before antigens: occurrence of jaundice a. HBaAg, HBbAg, HBcAg c. The end of incubation period and b. HBaAg, HBcAg, HBsAg height of disease with an jaundice c. HBcAg, HBeAg, HBsAg period d. HBaAg, HBcAg, HBdAg d. The height of disease with an e. HBaAg, HBeAg, HBsAg jaundice period 714. HBsAg of viral hepatitis B is e. The end of incubation period and detected in: during disease a. Patient‟s excrements 718. The viral hepatitis B usually b. Saliva damages: c. Mucous of nasopharynx a. Children before 14 years old

101 b. Adults outrank 30 years old d. Elderly people (much than 60 c. Young people and adults (15-30 years old) years old) e. Age dependence does not exist d. Elderly people (much than 60 722. RNA-viruses causes the: years old) a. Smallpox e. Age dependence does not exist b. Chickenpox 719. Mechanism of HIB action to c. Cytomegaly hepatocytes and clinical d. Hepatitis A manifestation of viral hepatitis B e. Hepatitis B are: 723. The viral hepatitis B DNA a. Has direct cytopatic action to presence is revealed for hepatocytes, serious forms of laboratorial diagnostics during last illness - acute or chronic with early years. What type of reaction is it? formation of hepatic cirrhosis a. Chain-polymerization test b. Has direct cytopatic action to b. Indirect-hemagglutination test hepatocytes. Often disease is over c. Complement-binding test with full recovery d. Immunoenzyme test c. The mechanism of action is e. Inhibition of hemagglutination test unknown, 50 % of all forms of a 724. High-sensitivity methods hepatitis pass in the chronic form, are used for checking the presence of virus hepatitis B and usually it leads to cirrhosis and antigens in donors' blood. to a liver cancer Which test should be used? d. The mechanism of action is a. Solid-phase Immunoenzyme test unknown; the serious form of b. infection may be especially at c. Indirect-hemagglutination test pregnancy d. Complement-binding test e. Has meditational immunological e. Neutralization test action to hepatocytes; usually 725. A patient with signs of hepatitis chronically carriage is is hospitalized to the infectious characterized as a persistent or compartment. She is a chronically active hepatitis; and stomatologist. What methods of without symptoms laboratorial diagnostics must be 720. The most sensible method for use for “Viral hepatitis” HBsAg revealing is: diagnosis? a. Precipitation in gel test a. Determination of HBs-antigen in b. Counter electrophoresis blood serum c. Passive hemagglutination test b. Virological examination of d. Inhibition of hemagglutination test excrements e. Immunoenzyme test c. Virological examination of urine 721. The viral hepatitis E usually d. Determination of functional damages: activity of hepar (bilirubin and a. Children before 14 years old cholesterol in blood) b. Adults outrank 30 years old e. Determination of enzymes activity c. Young people and adults (15-30 (aldolase, transamilase and other) years old) 726. The donor who for a long time

102 didn't donate the blood was e. Gene-engineering investigated with ELISA method. 730. The results of laboratorial tests 1. Anti-HBs antibodies were of patient with jaundice are: revealed. What does positive HBsAg “-”, HBeAg “-”, anti- result of ELISA in this case HBs(IgM) “-”, anti-HBs(IgG) mean? “+”, HCcAg “+”. What diagnosis a. Acute hepatitis B is confirmed by these results? b. Have been ill a hepatitis B a. Relapsis of hepatitis B c. Acute hepatitis C b. Hepatitis C d. Chronic hepatitis B c. Chronic hepatitis B with low e. Chronic hepatitis C replicative activity 727. HBs-antigen has been revealed d. Hepatitis C, hepatitis B in during enzyme immunoassay in anamnesis serum. What infection can be e. Reinfection of hepatitis B diagnosed by this antigen 731. Testing of donor‟s blood to detecting? HBs antigen is not completely a. Tuberculosis exclude a problem of b. Viral hepatitis A posttransfusional hepatitis. Which c. Viral hepatitis B of these viruses exclude hepatitis d. AIDS B virus most possibly can cause e. Syphilis posttransfusional hepatitis? 728. A patient is hospitalized with a. Hepatitis E virus the previous diagnosis “hepatitis b. Hepatitis A virus B”. To diagnose the disease a c. Hepatitis C virus serological reaction based on d. Epstein-Barr virus interaction of antigen and e. Cytomegalovirus antibody, chemically linked to 732. The patient is passing the peroxidase or alkaline clinical examination. Diagnostics phosphatase, was carried out. of hepatitis viruses was done. Name this reaction. Antibodies to HBs were revealed a. Immobilization in a blood serum. This positive b. Radio-immune assay result means: c. Immuriofluorescence test a. Tolerate hepatitis B d. Enzyme immunoassay b. Acute hepatitis B e. Complement fixation test c. Acute hepatitis C 729. The gene of hepatitis B virus d. Chronic hepatitis B was integrated to a genome of e. Chronic hepatitis C variolovaccine virus, what provide 733. A child with complains what the HBsAg creation. The serve for a doctor as a base for recombined virus will be use like diagnosis “viral hepatitis A” was a vaccine. This vaccine is refer to: hospitalize to infectious a. Synthetic department In a month after b. Combined returning from summer camp. c. Associate What is the most possible d. Chemical

103 infection mechanism of invasion a. Is possible to use a packed red by viral hepatitis A? cells only a. Contact b. Blood is suitably to transfuse b. Fecal-oral c. Is possible to use a blood plasma c. Airdust only d. Transmissible d. Blood is no-suitably to transfuse e. Parenteral e. Blood can be used after processing 734. There are one of markers of 737. There is B and D viruses of hepatitis B was been detected at hepatitis were detected at patient the blood analysis. Detecting what with a serious disease. As we of markers of this hepatitis don‟t know, Δ-virus can‟t reproduce in allow t use donor‟s blood for a hepatocytes independence. About hemotransfusion? what process the reproduction of a. HBc-antigen hepatitis D virus is possible in the b. HBs-antigen presence of hepatitis B? c. HAv a. Interference of viruses d. HBe-antibodies b. Complementation of viruses e. HBe-antigen c. Genetic reactivation of defective 735. Patient M. was hospitalized to viruses infectious department with the d. Phenotypic mixing next complains: general malaise, e. Mutations in Δ-virus genome rise in temperature of body to 738. HBsAg was detected at a 38ºC and jaundice. The doctor patient‟s blood which had suspect a viral hepatitis B jaundice 10 weeks ago. What of considering a hemotransfusion pathology it is typical for? what had be done a few months a. Viral hepatitis C ago. What are main methods for b. Viral hepatitis A hepatitis B diagnostics? c. Viral hepatitis B a. Serological and genediagnostics d. Viral hepatitis E b. Allocation of virus on a cells e. Viral hepatitis D culture and identification by CPA 739. There are the vaccine among of c. Revealing of virions in blood by biological drugs, what created by electron microscopy gene-engineering way. What of d. Isolation of virus in laboratorial infection it is prescribed for? animals a. Hepatitis A e. Isolation of virus in chicken b. Poliomyelitis embryo c. Measles 736. At control of blood, the HBsAg d. Parotitis was detected at one of the donors. e. Adenoviral infection Thus he said that recently he has a 740. About what infection the course of treatment with a detection of HBsAg in blood is parenteral entering of infusional display? solutions. Estimate a blood of this a. Herpetical infection patient for transfusion. b. Smallpox c. Viral hepatitis A

104 d. Poliomyelitis 745. To the hospital a woman at the e. Viral hepatitis B age of 23 was admitted who during pregnancy has not passed routine Laboratory diagnosis of diseases microbiological and virological caused by DNA - viruses survey. Her baby has immediately 741. For the diagnosis of generalized after birth the skin and mucous small herpes infection was studied blood bubbles on the face of limited swollen serum to detect specific antibodies of spots. Later developed the damage of specific class. Antibodies of which central nervous system and the baby class indicate the acute phase of viral was not saved. When sowing the infection? contents of the bubbles in the blood A. Ig M agar growth of the pathogen have B. Ig A been identified, and during infection C. Ig E of tissue culture they observed CPE D. Ig D (simplasty, intranuclear inclusions, E. Ig. G degeneration of cells). Which 742. The risk of congenital organism is the most likely causative malformations is 80% when infected agent of neonatal infections? pregnant women in the first trimester A. Staphylococcus of pregnancy by the virus: B. Gonococcus A. Flu C. Chlamydia B. Mumps D. Herpes simplex virus C. Rubella E. Cytomegalovirus D. Hepatitis A 746. The family Herpesviridae E. Coxsackie subfamily Alphaherpesvirinae 743. Which of the following belongs to: infections in the human cause A. Varicella-zoster virus adenoviruses? B. The virus of smallpox A. Myocarditis C. Coxsackie virus B. Hepatitis D. Cytomegalovirus C. Faringconjunctivitis E. Epstein-Barr virus D. Poliomyelitis 747. The main route of transmission E. Hemorrhagic fever of herpes simplex virus: 744. The patient, who received A. Alimentary immunosuppressive therapy for B. Contact systemic disease, signs of activation C. Parenteral of CMV infection. Which method D. Air and dust should be chosen to confirm the E. Transplacental diagnosis? 748. The family Herpesviridae A. RAC, RN with paired sera subfamily Betaherpesvirinae belongs B. The biological method to: C. Allergotest A. Herpes simplex virus D. Investigation of cellular immunity B. Varicella zoster virus E. Investigation of nonspecific C. Coxsackie virus resistance

105 D. Cytomegalovirus 755. In temperate countries Epstein- E. Epstein-Barr virus Barr virus usually causes: 749. The family Herpesviridae A. Hemorrhagic fever subfamily Gammaherpesvirinae B. Infectious mononucleosis belongs to: C. Lymphoma of Berkitt A. Herpes simplex virus D. Lymphocytic choriomeningitis B. Varicella zoster virus E. Nasopharyngeal carcinoma C. The virus of smallpox 756. In tropical countries the Epstein- D. Cytomegalovirus Barr virus usually causes: E. Epstein-Barr virus A. Hemorrhagic fever 750. As a rapid diagnosis of herpes B. Infectious mononucleosis infection apply: C. Lymphoma of Berkitt A. Enzyme Immunoassay D. Lymphocytic choriomeningitis B. The reaction of neutralization E. Nasopharyngeal carcinoma C. Immunofluorescence method 757. In Southeast Asia Epstein-Barr D. Radioimmunoassay virus usually causes: E. The reaction of hemagglutination A. Hemorrhagic fever 751. The main route of transmission B. Infectious mononucleosis of varicella: C. Lymphoma of Berkitt A. Alimentary D. Lymphocytic choriomeningitis B. Respiratory E. Nasopharyngeal carcinoma C. Contact 758. The penetration of rubella virus D. The parenteral in macroorganism often occurs E. Transplacental through: 752. The center of persistent infection A. Lining of the airways of shingles stored in: B. Intestinal mucosa A. Spinal cord C. Mucosa of genital organs B. Ganglia of spinal cord D. The salivary glands C. Salivary glands E. Damaged skin D. Nerve cells hippocampus 759. Herpes simplex virus type 2 has E. Anterior roots of spinal cord a role in the development of: 753. For the specific prophylaxis of A. Berkitt's lymphoma varicella developed: B. Nasopharyngeal carcinoma A. Live vaccine C. Cervical Cancer B. Killed vaccine D. Leukemia C. Toxoids E. Breast Cancer D. Chemical vaccine 760. Genomic DNA viruses cause: E. Idiotypic vaccine A. Mumps 754. CMV has tropism for cells: B. Tick-borne encephalitis A. The spinal cord C. Rubella B. Salivary glands D. Hepatitis A C. Lymphocytes E. Hepatitis B D. Airway epithelial cells 761. In the kindergarten several cases E. Epithelial cells of the of children‟s illness have been gastrointestinal tract occurred. The clinical picture was

106 characterized by fever and the toxoplasmosis, 2) rubiinfection, 3) appearance in the throat, mouth and cytomegaloinfection, 4) herpes skin rash vesicular. The preliminary infection. Which of these diseases are diagnosis - chicken pox. Which of the caused by viruses? following materials should be sent to A. Rubiinfection, the virology laboratory for rapid cytomegaloinfection, herpes infection diagnosis? B. Rubiinfection, herpes infection A. The contents of the vesicles C. Cytomegaloinfection, herpes B. Sputum infection C. Swabs from the hands D. All of the above D. Urine E. Rubiinfection E. Bile 765. The patient went to a doctor on 762. The contents of the mucous the third day of illness with membrane of the patient with vesicles complaints of a rash in the form of smallpox were sent to the virology vesicles on the lips and nose wings, laboratory. Which of the following painful when pressed. Appeared after changes were detected by smear pneumonia. What disease can be microscopy? suspected? A. Cells of Paschen A. Herpes simplex B. Cells of Babesh - Negri B. Shingles C. Cells of Guarnieri C. Atopic dermatitis D. Cells of Babesh- Ernst D. Erysipelas E. Syncytium E. Hives 763. The student of medical school, 766. The patient with infectious of hospitalized in the infectious disease mononucleosis identified the clinic, on the 2nd day of the disease is antibodies to antigens of Epstein-Barr suspected infectious of virus. Which of the following ways mononucleosis. What is the result of are the most common transmission for laboratory tests can confirm the this pathogen? diagnosis of a student on the day of A. Alimentary admission? B. Airborne A. Detection of Ig M-antibodies to C. Contact herpes simplex virus D. Sexual B. Detection of Ig M-antibodies to the E. Parenteral Epstein-Barr virus 767. At 17 years old male diagnosed C. Identification of 4-fold rise of gingivostomatit caused, as was antibodies to Epstein-Barr virus demonstrated by the FTA, the herpes D. Isolation (separation) of the herpes virus. What method can be virus established that this disease is the E. Detection of antibodies to result of primary or recurrent cytomegalovirus infection? 764. Laboratory of Diagnostic Center A. Radioimunny analysis received modern test systems to detect B. RAC with paired sera TORSN infection, which makes it C. Infection of chick embryos and possible to diagnose: 1) tissue culture

107 D. Determination ІgG and IgM in B. Statement of allergic tests serum C. Isolation of bacteria from the E. The immune electron microscopy vesicular fluid 768. The patient asked the dentist D. Videlenie virus from vesicular complaining of the appearance of fluid blisters on the lips with the liquid on E. Vesicular fluid microscopy the verge of the skin and mucous 772. Female at the age of 32 years old membranes. Which microorganisms suffers from recurrent viral infection. can cause such pathology? The disease appears vesicular A. Mycobacteria eruptions on the lips and mouth, B. Streptococci which are accompanied by fever and C. Herpes general intoxication. Which drug D. Ortomyxovirus should be used for specific E. Staphylococci prophylaxis of recurrence of the 769. A patient has fever and the disease? appearance of vesicles, which are A. Acyclovir located on the edge of the skin and B. Interferon mucous membranes. Through which C. Lysozyme research methods can confirm the D. Herpetic inactivated vaccine presence in the patient of herpes E. Donor gamma-globulin simplex virus? A. Microscopic and allergic Sanitary-microbiological B. Virological and biological examination of water, air, soil and C. Serological and biological food D. Microscopy and biological 773. When the sanitary- E. Virological and serological bacteriological examination of soil to 770. In the treatment of caries dentist determine perfringenstiter inoculation drew attention to the existence of was made dilutions of the suspension clusters of bubbles on the boundary of of the soil on blood agar in Petri cups. the skin and mucous membrane of Crops were grown in an incubator at a lips. Which micro-organisms can temperature of 37C. In a day colonies cause such pathology? of clostridia have been identified. A. Streptococci What is the most likely cause a result B. Herpes that is observed? C. Mycobacteria A. For soil bacteria requires a lower D. Ortomyxsovirus temperature E. Staphylococci B. The absence of necessary growth 771. The patient in the mucosa of the factors in a medium mouth, nose, lips, appeared vesicular C. Clostridium form colonies only vesicles. Dentist suspected vesicular after 3 days stomatitis. What diagnostic method D. The growth of clostridia was gives the opportunity to confirm the suppressed by microbes-antagonists, diagnosis? which are also present in the soil A. Infection with vesicular fluid of E. Have not been provided with animals anaerobic conditions

108 774. In determining the quality of E. All pathogenic and opportunistic water wells , was established that it‟s bacteria microbial count - 200, if titer - 100. 778. During the sanitary - Determine the index of water. bacteriological tests of tap water have A 100 the following results: the total number B. 10 of bacteria in 1.0 ml - 80 cells, the C. 5 index - 3. How to interpret the result D. 2 of the study? E .0.5 A. The water quality questionable 775. In well water revealed the B. Water suitable for drinking cholera bacteriophage 5x102 in the C. Water quality is very questionable number of phage particles in 1 ml. D. The water is contaminated Which method makes it possible to E. The water is very polluted get these figures in? 779. During the sanitary- A. precipitation reaction with bacteriological examination of water antiserum antiphage using membrane filters revealed two B. Electron Microscopy red colonies on a membrane filter C. The titration method of agar layers (Wednesday Endo), who conceded in D. density gradient ultracentrifugation 500 ml of water under study. E. Planting in culture medium Calculate the index and the titer of 776. In the bacteriological laboratory investigated water conducted a study of drinking water A. 4 and 250 quality. Its microbial count was B. 2 and 500 around 100. Which micro-organisms C. 250, and 4 considered in this? D. 500 and 2 A. All bacteria that have grown in a E. 250 and 2 nutrient medium 780. To assess the suitability of B. Escherichia coli drinking water conducted C. Bacteria pathogenic to humans and bacteriological research. Which animals indicator shows the number of D. Opportunistic microorganisms coliforms, which is contained in 1 E. Enteropathogenic bacteria and liter of drinking water? viruses A. Perfringens-titer 777. In determining the number of B. Coli-titer microbial air in the hospital it turned C. Coli phage titer out that it is 1500 cells/m3. Which D. Coli-index groups of organisms considered in E. Microbial number this? 781. When checking the condition of A. Staphylococci and haemolytic air in the operating room before streptococci surgery sedimentation method B. Bacteria and viruses - pathogens of revealed five small, round colonies, respiratory infections which are clearly visible around the C. All bacteria that have grown in a zone of hemolysis. What kind of nutrient medium impact has been made the crops on? D. Agents of hospital infections A. Levine

109 B. IPA occurred after ingestion of cooked C. Endo meat, which remained at room D. VSA temperature during the day. What is E. MPA blood the most probable result of seeding of 782. The study of air in the operating pathological material (gastric room revealed that it does not satisfy washings and feces) on the culture the sanitary and bacteriological media? standards. Isolation of A. The blue colonies on agar, alkaline microorganisms which allowed B. Crimson colonies on Endo medium making such a conclusion? C. Uncoated colonies on Endo A. Sartsiny 200 / m 3 medium B. Actinomycetes 100 / m 3 D. Yellow colonies on VSA C. E. coli 10 / m 3 E. Large slimy colonies on MPA D. Hemolytic streptococcus 5 / m 3 786. In the laboratory of sanitary- E. Staphylococcus aureus 3 / m 3 epidemiological station we study the 783. In the laboratory of food hygiene quality of drinking water. What of regional SES delivered withdrawn method can be determined if the index from the seller beef mince in the and the number of water-titer? market. The buyer has doubts about A. Filtration the quality of meat. He believes that B. Sedimentation the stuffing is made of dog meat. C. Serial dilutions What is an immunologic response can D. The method of titration check the quality of the delivered E. Microscopic product? 787. In carrying out sanitary- A. The reaction of precipitation bacteriological examination of air in a B. The reaction of Coombs dental office to determine the number C. agglutination of sanitary-indicative microorganisms D. Reaction of opsonization (Staphylococcus aureus and hemolytic E. Reaction imunofluoresence streptococci) in 1 m3 of air. What is 784. In the tray of a private the culture medium used for this? entrepreneur sold the minced meat, A. Blood and yolk-salt agar according to the seller of beef. B. MPA and MPB However, the sanitary inspector C. Endo and Ploskirev Medium suspected that stuffing is made of dog D. The yolk and sugar broth meat. Through which reaction can be E. Kitt Tarotstsi and Wilson-Blair identified this food? Medium A. Flocculation B. Binding of complement Normal microflora of the human C. Agglutination body D. Ringprecipitation 788. The patient, who has long taken E. ELISA antimicrobials, conducted 785. In the infectious ward three bacteriological examination of vaginal students were brought who lived in microflora and determination of pH. the same room, with a diagnosis of The absence of lactobacilli and "acute gastroenteritis." The disease alkaline conditions. What should be

110 assigned the patient to restore normal 5h108; lactobacilli - 109; microflora of the vagina? the total number of E. coli -107; A. Lactic acid bacteria Escherichia coli with reduced B. Suppositories with antibiotics enzymatic properties - 8%; C. A solution of potassium opportunistic enterobacteria - 5x10; permanganate staphylococci 104; hemolytic D. Sulfanilamide drugs staphylococci were not identified, E. Suppositories with antiseptic fungi of the genus Candida 102. To 789. Cancer patients removed most of what extent these data indicate the large intestine. Which drugs disbiosis? should be assigned to the patient to A.Norma replace the function of colon B. I degree microflora? C. II degree A. Vitamins D. III degree B. Antistaphylococcus plasma E. The resulting data are insufficient C. Polyvalent bacteriophage for assessing disbiosis D. Antibiotics 793. The patient at the age of 68, who E. Sulfonamide was a three-week course of 790. The patient after prolonged use antibiotics, bowel dysfunction is of antibiotics developed intestinal marked, abdominal pain. The study on dysbiosis. Which drugs should be fecal bacteria overgrowth yielded the appointed to restore the normal following results: microflora? bifidobacteria-5x 105, lactobacilli- A. Sulfonamide 102, the total number of E. coli - B. Eubiotics 1010, E. coli with reduced enzymatic C. Interferon properties - 40%; D. Antifungal hemolytic E. coli - 10% E. Cephalosporin opportunistic enterobacteria-105, 791. When dysbiosis, which is hemolytic aureus - 102, accompanied by the development of fungi of the genus Candida - 103. To putrefactive flora and increase the pH what extent these data indicate of faeces, it is necessary to assign dysbiosis? biological agents that oxidize medium A.Norma and exhibit an antagonistic effect. B. I degree What organisms are used for this? C. II degree A. Bifidumbacteria D. III degree B. Klebsiella E. The resulting data are insufficient C. Azotobacteria for assessing dysbiosis D. Enterobacteria 794. Child of 5 months, which is on E. Sarcina artificial feeding, recovering from 792. There is dysfunction of the kolienterit. Before discharge from intestine at 10 years old child for three hospital a study was conducted months. Investigation of fecal quantitative and qualitative material of disbiosis gave the composition of intestinal microflora. following results: bifidobacteria - Obtained the following results:

111 bifidobacteria - 5x 107; laktobatsily - The microflora of the mouth. 108; 797. 63 years old man turned to the total coliform-109, E. coli with dentist complaining of a complication reduced enzymatic properties - 10%; during chewing. The doctor suspected hemolytic E. coli - 5% the development of tetanus infection. opportunistic enterobacteria - 105; As it was found a week ago, the aureus - 104; fungi of the genus patient is deeply pierced his leg with a Candida - 103. What tools are most rusty nail. What material should be appropriate for corrections of sent to study in the bacteriological microbiotsenosis intestine in a child? laboratory to confirm the clinical A. Antibiotics that selectively inhibit diagnosis? the growth of staphylococci and A serum of patients yeasts B. Blood patient B. Inclusion of milk products in the С. Smear from the surface of the diet wound C. Bifidumbacterin D. Pieces of necrotic tissue D. Colibacterin E. Flush with subject matter that was E. Lactobacterin the cause of injury 795. Pregnant woman was diagnosed 798. 32 years old man in the lower with bacterial dysbiosis of vagina. jaw formed dense, flushed swelling, Which drug should be chosen in this pain is virtually nonexistent. After 2 case? weeks, pus appeared on the surface of A. Bacteriophage the hearth, formed a fistula, a chronic B. Antibiotic infectious process was slow and C. Interferon extends to the underlying tissue. D. Eubiotics Microscopy of pus was found beads E. Polyvitamins that represent a collection of threads, 796. In a patient of 56 years old after as well as individual rod-shaped and prolonged treatment with antibiotics сoco form. What is the most likely developed dysbiosis: weight loss, diagnosis? frequent stools, a significant amount A. Actinomycosis of fecal hemolytic E. coli, Proteus, B. Deep mycosis Staphylococcus. Which of the C. Candidosis following actions would eliminate the D. Nocardiosis imbalance autochthonous microflora: E. Mixed bacterial infection A. Replace the antibiotics and to 799. The man at the age of 21 turned phage therapy to the doctor with a dense rounded B. Cancel the antibiotics and sulfa ulcer on the mucous surface of the drugs to appoint cheek. The doctor suspected syphilis C. Cancel antibiotics and assign and gave direction to the study of eubiotics serum. Wassermann was negative. D. Provide chelators and The doctor put the negative diagnosis. immunomodulators. How to evaluate the tactics of a E. Provide drugs and adjuvants. doctor? A. You must repeat the study in a few

112 days alpha-hemolysis. Is this man a carrier B. A numerical method for the of some kind of pathogen of diagnosis and the correct diagnosis microorganism? C. There should also be staging the A Streptococcus salivarium sedimentary response by Cann B. Streptococcus pyogenes D. Must be microscopic study C. Streptococcus pneumonia E. The need for formulation of an D. Streptococcus feacalis allergic test E. Peptostreptococcus 800. At the dental examination of the 803. At the patient's buccal mucosa child of 3.5 years old of the mucous revealed net shaped ulcer with hard membrane of cheeks and palate doctor bottom and smooth edges. Which said raids of white and yellow colors organisms are most likely to be that merge to form a cheesy film. observed by microscopy of fluid from When removing the film - the mucous the surface of an ulcer? membrane is hyperemic, with a A. Thin, spiral, motile smooth surface. What picture will B. Acid FAST Sticks most likely be observed by C. Gr + cocci arranged in chains microscopy of smears from these D. Large spore bacillus films? E. Bipolar colored ovoid A. Gr + cocci, arranged in a cluster of 804. Dental instrument has been grapes sterilized by boiling for 5 minutes. B. Gr + cells elongate budding Which organisms, have got it out of C. Gr + cocci arranged in chains the mouth or the patient's blood, are D. Gr + rods with swellings at the likely to preserve? ends A. Diphtheroids E. Gr - small coccobacilli B. AIDS virus 801. Bacteriological examination of C. Streptococci 10 years old boy‟s mouth revealed a D. Fuzobacteries large number of β-hemolytic E. Hepatitis B virus streptococci. What is the most 805. Dentist analyzed the qualitative credible explanation for this composition of microflora of the phenomenon? mouth‟s 70-year-old man, and A. The diet of the boy too much concluded its worst condition. carbohydrate Identify what microorganisms B. The child is sick caries allowed to make such a conclusion? C. The child is sick periodontal A Streptococcus mutans D. The boy observes bad oral hygiene B. E. β-hemolytic streptococci are the C. Escherichia colі representatives of normal microflora D.Mycoplasma orale 802. From the oral cavity of clinically E. Candida albicans healthy man at the age of 25 years old 806. In the pus taken from a depth of has identified the culture of Gr + necrotic ulcer of the patient cocci, which are somewhat elongated, necrotizing ulcerative stomatitis, arranged in pairs or short chains form identified fuzobacteries. What other a capsule on blood agar yield of organisms are involved in the

113 development of this disease and to 810. Choose among the normal stand out in association with microrflora the oral obligate fuzobacteries? anaerobes. A Streptococci A. Streptococci, staphylococci B. Mycoplasmas B. Bacteroids, fuzobacteries C. Yeast-like fungi C. Corynebacterium, veylonelly D. Spirochetes D. Mycoplasmas, Borel E. Diphtheroids E. Leptospira, Candida 807. At 17 years old male diagnosed 811. Patient D. appealed to the dentist gingivostomatit caused, as was with complaints on halitosis demonstrated by the FTA, the herpes (unpleasant odor from the mouth). virus. Through a method to establish What bacteria dominate the whether the disease is caused by microflora of the oral cavity in this primary or recurrent infection? case? A. Radioimmunoassay A. Genus Proteus B. RAC with paired sera B. Type of Bacteroides C. Contamination of chicken embryos C. Type of Corynebacterium and tissue culture D. Type of Esherichia D. Determination of IgM and IgG in E. The genus Clostridium serum 812. Bacteriological examination of E. The immune electron microscopy the oral cavity of dental patients 808. From the dental patient was revealed a number of microorganisms. taken the pus from the abscess to Which of them belong to eukaryotes? investigate, which is located on the A. Mycoplasma orale lower jaw. Pathological material B. Streptpcoccus mutans planted on blood agar for detection of C. Candida albicans cocci and on Kitt Tarotstsi medium D. Treponema buccalis for detection of anaerobes. Which E. Bacteroides melaninogenicus method of microbiological diagnosis 813. In the mouth of 55 years old was used in this case? man was found staphylococcus, A. Bacteriological which forms enzymes hyaluronidase B. Smear and collagenase. What are the most C. Virological possible damage it may cause the D. Serological organism? E. Biological A. The destruction of the connective 809. Microscopy of dental plaque was tissue detected moving spiral-like bacteria. B. demineralization of tooth enamel Which method allowed us to establish C. The formation of tartar the mobility of these organisms? D. Change of periodontal tissue A. Fluorescent microscopy trophism B. Darkfield Microscopy E. Necrosis of the epithelium C. Staining with Romanovsky- 814. In a patient with recurrent Giemsa inflammatory lesions in the oral D. Staining of Gins-Burri cavity bacteriological examination E. Electron microscopy revealed a large number of

114 gramnegative bacteria. What glucan conclusion should be drawn regarding C. microbial insemination plaque the patient's condition? anaerobes A. Dysbiosis D. The formation of dental plaque B. T-cell immunodeficiency bacteria, organic acids C. Candidosis E. IgA class of antibodies against D. Recurrent herpes components of the microflora in the E. An allergic condition oral cavity 815. The patient has a dental clinic in 818. After opening the maxillary Flushing oral mucosa sowed E. coli. abscess, which is accompanied by Which members of the microflora of swelling of the tissues of the face, the the mouth, these bacteria belong to? temperature up to 39C, pus was taken A. Resident and sent to the bacteriological B. Transient laboratory. The study found that C. Causal agent of caries pathological material containing D. The causative agent of stomatitis microorganisms of the genus E. Pathogen of pulpitis Bacteroides. What are the types of 816. Patient B., aged 59, appealed to microorganisms of the usage of the dentist about halitosis. After oxygen they are? detailed examination the doctor found A. Optional aerobes that the cause of halitosis (halitosis) B. Aerobes was associated with a violation of the C. Anaerobes microflora of the mouth. Which of D. Gram-negative bacilli these microorganisms may play a E. Gram-positive cocci central role in the development of 819. One of the factors that contribute halitosis? the development of dental caries in A. Anaerobic bacteria (prevotely, humans is the formation of a bacteroids, peptostreptococci, significant number of microorganisms veiloneli) of lactic acid. Choose among the B. Bifidobacteria listed microorganisms that ferment C. The simplest (Entamoeba carbohydrates to form a large amount gingivalis, Trichomonas elongate) of lactic acid. D. Facultative anaerobic cocci A. Staphylococcus, Corynebacterium (staphylococci, streptococci) B. Veiloneli, Streptococcus C. Fuzobacteries, lactobacilli Microbiological and immunological D. Treponema, Neisser aspects of the etiology and E. Actinomycetes, fungi pathogenesis of dental caries. 820. Patient appealed to the dentist 817. Caries is the result of a complex with initial stage of pulpitis. What of harmful factors in the oral cavity. organisms are at this stage of the The initial phase of caries is a direct disease? consequence: A. Oral streptococci, lactobacilli A. The provision of the teeth of a film B. Beta-hemolytic streptococci, of saliva glycoprotein bacteroides B. Synthesis of Streptococcus mutans C. Proteus and Clostridium

115 D. E. coli all over her bottom, which is E. Anaerobic streptococci characteristic of acute pulpitis. What 821. Dentist conducts a conversation are the most common organisms among school children in caries involved in the development of prophylaxis. What are the substances pulpitis? that are contained in foods contribute A. Staphylococcus aureus. the growth and multiplication of B. Streptococcus salivarius. streptococci, and are of high C. Actinomyces viscosus. importance in the occurrence of dental D. Leptotrichia buccalis caries? E. Prevotella melaninogenica. A. Carbohydrates 825. In a patient with multiple caries B. Proteins was carried out immunological test C. Fats that will assess the humoral local D. Vitamins immunity in the oral cavity. Specify E. Micro-and macro- the ratio of IgA, IgG, IgM 822. Patient appealed to the dentist characteristic of healthy human complained of pain in the fifth lower saliva? right tooth, which appears in the use A. 4:2:1 of food as well as discomfort when B. 5:2:2 applied to a tooth cold or hot. While C. 1:4:2 examination, the doctor found a tooth D. 20:3:1 cavity. In probing the bottom of the E. 1:1:2 cavity particularly painful points were 826. Patient appealed to the dentist found, indicating a deep cavity. with multiple cavities. What Specify a group of cariogenic properties are the most common microorganisms. bacteria that cause tooth decay? A. S.salivarius, diphtheroids, A Lysogenic peptococci B. Resistance to antibiotics B. S.mutans, lactobacili, C. Lipolytic actinomycetes D. Acidogennye (saccharolytic) C. S.mitis, Treponema, Neisser E. Fagolizabelnye D. Stomatococci, leptotrihii, 827. Dentist conducts preventive Staphylococcus conversation among students on E. Veilonella, Treponema, Borrelia compliance with the rules of hygienic 823. Lactobacilli are cariogenic dental care. He told the children that microorganisms because they are: in violation of hygienic care of teeth A. Able to survive without nutrients formed on the surface of dental B. Stand out from the cavity plaque, which is dangerous to the C. The producers of acid teeth. It is formed in: D. In plaque A. Three to six months E. Cytohromoxidaznegative B. A few hours 824. 28 years old patient appealed to C. One month the dentist complaining of a throbbing D. Thirty minutes toothache, which gives the temple. E. A few days Found in probing the cavity felt pain 828. During a routine inspection of

116 schoolchildren dentist revealed that to baclabolatory? the majority of carious no damage to A swab from the oropharynx teeth. How secretory IgA to protect B. Blood, purulent exudates against cariogenic S.mutans? C. Cerebrospinal fluid A. It prevents adhesion D. Duration of the tonsils B. Cause lysis E. Blood, urine C. Phagocytose 833. In the occurrence of dental caries D. Neutralize exotoxin is very important plaque formation. E. Loosen the saccharolytic activity What is 829. Inoculation of pathological predominant microflora in the early material from the patient pulpitis was formation of dental plaque. made on Kitt Tarotstsi medium. A. Obligate anaerobes Which microorganisms are B. Streptococci, veilonella committed? C. Fuzobacteries A. Hemolytic D. Bacteroids, Candida B. Acid E. Leptotrihii C. Acidophilic 834. The patient with pulpitis has D. Anaerobic edema of the soft tissues of the upper E. Aerobic jaw. Which factor is the lead in the 830. While examination the patient's occurrence of edema? teeth dentist found many "white A. Tissue gipoosmiya spots" - areas of demineralization of B. Tissue gipoosmiya enamel. Indicate which C. Blood gipoosmiya microorganizm take part in the D. Blood gipoosmiya development of this process? E. Increased permeability of blood A. Streptococcus mutans vessels. B. Streptococcus salivarius 835. For anticaries immunization C. Streptococcus pyogenes offered the vaccine from a suspension D. Veilonella parvula of heat-killed S.mutans. What side E. Staphylococcus epіdermidis effects are possible when using this 831. The first stage of dental caries is drug? the formation of zones of A. Hepatic Impairment demineralization of enamel. What are B. Cardiac the microbial factors involved in the C. Mucous membranes development of this process? D. The defeat of the periodontal A. Coagulase of golden tissues staphylococcus E. Lung B. Collagenase anaerobic bacteria 836. Caries - one of the most common C. Streptococcus mutans acid dental illness .Through the formation development of methods of caries D. Letsitinaz of staphylococci prophylaxis in recent years their E. Streptococcal exotoxins hopes on the use of anticaries 832. Dental surgeon had a puncture vaccines of S. mutans. The primary phlegmon in a patient with fever defense mechanism of such the face. What material should be sent vaccination is associated with:

117 A. Education secretory Ig A, which diagnosed with suppurative prevents the adhesion of S. mutans complication. In the bacteriological B. Formation of delayed-type examination of material from a wound hypersensitivity culture was isolated, which formed C. Activation of saliva lysozyme the pigment of blue-green color. D. The production of antistreptolysin- Which of the following O microorganisms may be the causative E. Stimulation of Ig E production agent of infection in this case? 837. Local immunity in the oral cavity A. Proteus vulgaris is largely due to the content of B. Pseudomonas aeruginosa antibodies in saliva. Antibodies are a C. Bacillus subtilis class of saliva in higher concentration D. Klebsiella pneumonia than in blood, whereas the content of E. Staphylococcus erіdermidis arius the saliva and blood about the same? 841. The patient asked the dentist A. Ig G about the gingival and alveolar B. Ig A gnoeotdelenie caused by Gram- S. ІgM positive bacteria. Microorganisms are D. D Ig the kind most often cause a purulent- E. Ig E inflammatory processes of 838. Bacteriological examination of maxillofacial area? patients with dental caries has been A Staphylococcus allocated different microorganisms. B. Streptococcus Which of the following S. Veilonella microorganisms play a leading role in D. Lactobacillus the occurrence of caries in these E. Bacteroides patients? 842. Of the cavity and dental plaque A Streptococcus mutans streptococci isolated several species B. Staphylococcus aureus thought to be the main factors of this C. Candida albicans disease. To what family in the D. S. salivarius systematics of bacteria are called 839. As a result of delayed treatment Bergey bacteria? of pulpitis patient rozvilsya A. Veilonellaceae osteomyelitis of the mandible. What B. Streptococcaceae research will identify the agent and C. Mycoplasmataceae find an effective drug to treat a D. Bacteroidaceae patient? E. Micrococcaceae A. Identification of antigens of the 843. From the mouth of 5 years old pathogen child, the sick tooth decay, isolated Gr B. Detection of specific antibodies + organisms correct spherical form. In C. Microscopic examination of smears, they are in chains, on blood punctate agar give small translucent colonies D. Isolation of pure cultures without hemolysis. Which type is E. Complex most likely owned by the 840. In the department of microorganisms? maxillofacial surgery patient was A. Streptpcoccus mutans

118 B. Staphilococcus saprophyticus odontogenic disease? C. Mycoplasma orale A. Staphylococcus D. Candida albicans B. Actinomycetes E. Bacteroides melaninogenicus C. E. coli 844. 20 years young man caries led to D. Streptococcus complications: inflammation of the E. Against dental pulp. Dentist believes that it 847. In the maxillofacial unit of the was caused by streptococci pulpitis. regional hospital surgery on the What kind of fertile ground to sow the submandibular phlegmon site was pathological material (pulp content) in performed . With the help of what order to test this hypothesis? method of microbiological studies can A. IPA establish the etiology of this disease B. Blood agar in order to conduct an effective C. Endo Wednesday antibiotic in the out functional period? D. Folded serum A. Allergic E. The alkaline agar B. Bacteriological 845. In a patient with caries tooth C. Biological surfaces have been identified caries D. Tuberculosis microscopy white spots in the formation of which E. Serological microorganisms are involved. Dentist 848. Which microorganisms often act after treatment caries of teeth as agents of abscesses in maxillofacial suggested the patient to take area? prophylaxis measures for the A. Lactobacilli and diphtheroids prevention of tooth decay. What B. Actinomycetes and veilonelli factors should be sent these C. Escherichia coli and Streptococcus preventive measures on? D. Staphylococcus and Streptococcus A. The carbohydrate nutrition and S. E. Pseudomonas aeruginosa and mutans Proteus B. At the protein diet and S.aureus 849. In the department of C. The carbohydrate nutrition and maxillofacial surgery patient was S.aureus admitted with a solid phlegmon and D. At the protein diet and S.mutans multiple fistulas, of which stands out E. The carbohydrate nutrition and the pus with a foul odor. Which S.sanguis method should be applied to confirm 846. Based on the examination of the the diagnosis of actinomycosis? patient K. doctor of maxillofacial A. Smear department district hospital has put a B. Biological preliminary diagnosis of "acute C. Bacteriological odontogenic osteomyelitis." D. Mycological Microscopying the drug, made from E. Serological pathological material, laboratory 850. After tooth extraction in patients showed Gram-positive bacteria with progressive suppurative spherical shape, which is arranged in inflammation. Microscopic a random clusters ("grapes"). Which examination of pus revealed gram- organism is the causative agent of positive nonmotile, thick, surrounded

119 by a capsule bacillus. What method is pathogenesis of periodontal lesions necessary to confirm the preliminary 854. The patient appealed to the diagnosis - gas anaerobic infection? dentist with necrotizing ulcerative A. Allergic stomatitis. What organisms are B. Bacteriological naturally in the inflammation? C. Biological A beta-hemolytic streptococci, D. ELISA bacteroides E. Serological B. Fuzobacteries Vincent and 851. The patient appealed to the Treponema dentist with an acute pulpitis. Which C. Proteus and Clostridium microorganisms play a major role in D. E. coli the inflammation of the pulp? E. Anaerobic streptococci A. Streptococci of group D 855. The patient ,36 years old , B. Vibrio appealed to the dentist complaining of C. Spirillum pain in the lower six tooth when D. Sarcina chewing, bleeding from a tooth while E. Micrococci eating and feeling bad smell. What 852. As a result of delayed treatment caused halitosis? of pulpitis osteomyelitis of the A. Education microorganism‟s methyl mandible was developed in patient. and hydrogen sulfide. With some research you can set the B. Synthesis of microbial type of agent and find an effective hyaluronidase drug to treat a patient? C. Products of microorganisms of A. Isolation of pure cultures lactic acid B. Appearing of specific antibodies D. The products of the destruction of C. Microscopic examination punctate tissues by microorganisms D. Appearing of pathogen antigens E. The presence of endotoxin E. Integrated serology microorganisms 853. The patient appealed to the 856. After examining the patient the dentist with symptoms of dentist decided that the inflammation in the oral cavity. In bacteriological examination of smears that are made from the teeth pathological material taken from a and gums are revealed in the simplest patient is not feasible. Under what form of non-constant, which varies conditions could this be? due to the formation pseudolegs. A. To determine the sensitivity of Sizes - 6 to 30 microns. Indicate the microorganisms to antibiotics type of protozoa. abnormality A. Oral amoeba B. Under the conditions of specific B. Giardia diseases (eg, tuberculosis) C. Intestinal Amoeba C. Subject to the allergic nature of the D. Intestinal trichomonads pathological process E. Dysenteric amoeba D. In the case of purulent processes E. Under the conditions necessary to Microbiological and immunological clarify the nature of the infectious aspects of the etiology and lesions of the mucous membrane

120 857. The patient D. appealed to the and salivary glands dentist with complaints of pain in the B. In the lymph nodes gums, swelling of pus, redness, C. In the spleen formation of granulomas. Pus sent in D. In the thymus baclabolatory. In blood smears E. In bone marrow revealed gram-positive organisms in 861. It is known that periodontitis is the form of filaments at the ends of the result of the different species of which were controversial. What bacteria and their metabolic products. organisms cause inflammation? What type of bacteria produces A. Herpes collagenase, which breaks down B. Mycobacterium leprae collagen - the main protein C. Treponema pallidum component of the periodontium? D. Bacillus anthracis A. Prevotella melaninogenica E. Actіnomyces isrrailii B. Treponema vinsenti 858. The patient appealed to the C. Veilonella parvula dentist with complaints of pain, D. Leptospira dentium redness, swelling gums. Suspected E. Treponema dentium herpetic gingivostomatit. What virus 862. The patient as a result of could cause disease? activation of their own A. Herpes simplex virus type 1 microorganisms that are part of the B. Herpes simplex virus type 2 microflora of the oral mucosa, had C. Zoster virus purulent inflammation of periodontal D. Cytomegalovirus tissues. Which form of infection E. Epstein-Barr disease belongs to? 859. On examination, the patient's A. Exogenous infection oral cavity revealed a necrotizing B. Autoinfection gingivitis. Bacteriological C. Reinfection examination revealed Fusobacterium D. Superinfection nucleatum, F.necrorum. What are the E. Relapse serious complications of this infection 863. In the doctor's office, are at? periodontist for the appointment of A. Severe necrotic processes in the strengthening therapy, the patient destruction of tissues of the face studied factors of nonspecific () resistance of saliva and secretions of B. Gingivostomatity the mucous membrane. Which of the C. Periodontal Disease following factors of nonspecific D. Chronic lip fissures resistance in the first place will be E. Bridou studied in the material? 860. In patients with chronic A. Complement periodontitis scheduled for a study to B. Secretory IgA assess the local immunity of the C. Properdin mouth. Where exactly are localized D. Interferon plasma cells that synthesize E. Lysozyme immunoglobulins, oral fluid? 864. From the material studied patient A. In their own record of the mucosa periodontium in association with

121 gram-negative spirochetes were the body's response to the changed detected microorganisms rod with structure of the periodontium? pointed ends. The bacteria belong to A. Autoimmune the autochthonous microflora of the B. Immune oral cavity. Can form of peptone and C. Purulent and inflammatory glucose lactic acid. Which family D. Antibody owns the selected bacteria? E. Leukocytosis A. Lactobacillus 868. From a patient with apical B. Treponema periodontitis material is taken to C. Actinomices isolate pure cultures of the pathogen. D. Fusobacterium Where can patmaterial be sowed for E. Bacteroides isolated colonies? 865. During routine inspection the A. On the surface of the agar in a dentist revealed periodontal disease in Petri cup the pupil. It is established that B. On the surface slant periodontal disease is a complication C. The liquid culture medium of previously transferred to viral D. Injection into a column of dense infection, in which damaged small medium vessels. The disease is caused by a E. In the elective medium fluid virus of the family Paramyxoviridae consistency genus Morbillivirus. Virus complex, 869. From a patient with a diagnosis has hemagglutinin, neuraminidase of periodontal disease in the does not contain very sensitive to bacteriological examination of clinical sunlight. Outside the body is material were isolated gramnegative preserved up to 30 minutes. Which polymorphic bacilli, which grow only virus caused the disease transferred? under anaerobic conditions and A. Mumps virus whimsical to the culture medium. B. The influenza virus Which of the following C. Measles virus microorganisms may they be? D. The paragripp virus A. Bacteroids E. Respiratory syncytial virus B. E. coli 866. For microbiological diagnosis of C. Clostridium odontogenic disease dentist spends D. Mycobacteria fence material. In what period it is E. Rickettsia expected to do? 870. At the time of discharge from the A. After 3-4 hours after ingestion of gums microscopy patient who is food suffering from periodontal disease, B. After 8-10 hours after ingestion of identified the simplest form of pear- a meal shaped, 6-13 mm in length. In one of C. Before taking food the parasite nucleus, on the front end D. Immediately after taking the meal has 4 flagella, undulating membrane E. After the starvation is. What are protozoa found in the 867. During periodontitis the surface patient? structure of tissues changes. What A. Amoeba processes must be studied to assess B. Guardia

122 C. Leishmania 874. In the virology laboratory in the D. Balantidium study of fluid from the vesicles on the E. Trichomonas mucous membrane of cheeks, patient C was isolated vesicular stomatitis Microbiological and immunological virus. Which the family belongs the aspects of the etiology and virus to? pathogenesis of infectious lesions of A. Rinoviridae the oral mucosa D. Poxviridae 871. The young man was found C. Rhabdoviridae severe gingivostomatita and D. Reoviridae microbiological study of oral fluid E. Togaviridae was isolated fusiform bacteria and 875. In a study of patients with spirochetes oral. What congenital immunodeficiency, S. gingivostomatit arose in this patient? dentist found on the oral mucosa A. Gingivostomatit caused by the vesicles filled with turbid yellow fluid anaerobic microflora and diagnosed a viral stomatitis. What B. Gingivostomatit Vincent is the most common virus is the C. Bacterial gingivostomatit causative agent of this disease? D. Primary gingivostomatit A. Herpes simplex virus type 1 E. Viral gingivostomatit B. Herpes simplex virus type 2 872. Dentist found in the oral mucosa S. Herpes simplex virus type 3 of sick girl, 7 years old, ulcer 1.5 cm D. A virus Coxsackie in diameter with irregular edges and E. Coxsackievirus B saped gray bottom. After the paint 876. Patient K., age 67, after a few scrapings from the ulcer by Ziehl- months after the establishment of a Neelsen in the drug revealed thin rods fixed prosthesis teeth he asked the of ruby-red color, arranged in random dentist about the swelling, redness clusters and alone. For a pathogen is and irritation of the mucous characterized by such symptoms? membranes of the mouth, appears at A. Actinomycosis the corners of his mouth, " bridous ". B. Syphilis Which of the following organism is C. TB the cause of these complications after D. Diphtheria prosthesis? E. Candidiasis A. Veilonella 873. Of pathological material from a B. Staphylococcus sick child (scraping of the sores on the C. Streptococcus mucous membrane of the mouth) D. Candida medication prepared and stained by E. Treponema Ziehl-Neelsen. Identification of the 877. In the dental clinic brought a causative agent provided? child with fever, complaints of painful A. FMD sores on the mucous gums that look B. Actinomycosis like blisters with serous contents. In C. Syphilis the smear from vesicles stained by D. Herpes Romanovskiy-Giemsa, revealed giant E. TB multinucleated cells with intracellular

123 inclusions. Which virus caused the 881. On the oral mucosa of 20 years disease? old woman the dentist noticed a round A. Alpha-herpes ulcer with a thick bottom and smooth B. Beta-herpes edges, which resembles a chancre. C. Gamma-herpes What is the diagnostic method should D. HIV be used at this stage of the disease, to E. KSIP viruses confirm the etiology of syphilis? 878. Patient appealed to the dentist A. The biological with fever and characteristic small B. Bacteriological vesicles on the mucous membrane of C. Smear cheeks, palate and tongue. The doctor D. Serological suspected herpetic stomatitis. What E. Allergic additional research is necessary to 882. In the tested material from a confirm the diagnosis? patient with gingival pockets A. Infection of chick embryos in necrotizing stomatitis revealed thin horionalontois, the introduction of the spiral bacteria that are 8-14 curls and pathological material in the brain of Romanovsky-Giemsa stained a pale white mice pink color. Which organisms have B. Inoculation of 199 with the been identified in the material? addition of bovine serum A. Fusobacterium reriodonticum C. Rappaport media Inoculation B. Treponema vinsentii D. Statement of the precipitation C. Actinomices viscosus reaction D. Lactobacillus casei E. Needle media Inoculation E. 879. The dentist diagnosed with 883. For the treatment of odontogenic aphthous stomatitis, herpetic in two infections dentist appointed antibiotic month child. What way was a child therapy. What more appropriate infected? antibiotic should be used for the A. The contact prevention of candidosis of the B. Vertical mouth? C. The air-dust A. Nystatin D. Through fecal-oral B. Kanamycin E. At the time of delivery C. Levomiсetin 880. The patient with the ulcer, which D. Ristomiсin is located on the oral mucosa, staining E. Polymyxin by Romanovsky-Giemsa revealed thin 884. In a study of preschool children a spiral-like organisms pale pink with dentist found in the mucosa of the 12-14 coils and pointed ends. What mouth coating and film of gray-white pathogen is characterized by such color. At microscopy stained by Gram symptoms? drug identified microorganisms: A. Syphilis Gram-positive, ovoid shape, thread B. Causative agent of leptospirosis pseudomycelium. Which C. Causative agent of typhoid rotary microorganisms cause mucosal D. Agent of damage? E. Pathogen A. C.albicans

124 B. S.aureus C. Bacteriological, allergic C. B. anthracis D. Bacterioscopy, biological D. Leptotrichia E. Smear, bacteriological E. Actinomyces 889. The patient appealed to the 885. In studying the microflora of the doctor (the milkmaid), with a rash on oral cavity in a patient with stomatitis the mucosa of the mouth in the form E.soli found in large quantities. of the AFL. The doctor found a rash Which drug is advisable to apply for on the skin of the hands around the adjustment status dysbiotic mouth? nail plate. What pathogen caused the A. Bacterial immunomodulators disease? B. Bacteriophages A. Cytomegalovirus C. Antibiotics B. Coxsackie virus D. Sulfonamide C. FMDV E. Antisera D. Herpesvirus 886. Patient appealed to the dentist E. Vesicular stomatitis virus complained of vesicular rash around 890. The patient was diagnosed with the mouth. Which method should be dental offices - necrotizing Vincent's applied to confirm the diagnosis disease. What microorganisms are "herpes"? pathogens of this disease? A. Microscopic examination of A. Fuzobacteries and spirochetes smears B. Streptococci and staphylococci B. Haemagglutinin C. Actinomycetes C. Seeding on blood agar D. Bacteroides D. Seeding to Kitt Tarotstsi medium E. Herpes simplex virus E. Immunofluorescence reaction 891. During the microscopic 887. In a patient with smear examination of biopsies of the lesion microscopy of necrotic ulcers in the of the mucous membrane of the oral cavity revealed spiral organisms, mouth sticks were found arranged in which are stained in blue by clusters that resemble a pack of Romanowsky-Giemsa. Specify cigars. Ziehl-Neelsen stain in red. whether the organism is involved in What type of pathogen, the most the development of the necrotic reliably detected in the biopsy? process in this patient? A. A.bovis A Spirochete Vincent B. M.tuberculosis B. Treponema pallidum C. A.israilii C. Obermeyer pallidum D. M.leprae D. Leptospira E. M. avium E. Pallidum 892. Saliva has enzymes that 888. A patient with a lesion of oral neutralized the bacteria. Name it. mucosa was diagnosed - necrotizing A. Amylase stomatitis Vincent. What methods of B. Phosphatase laboratory diagnosis should be used to C. Lysozyme confirm the diagnosis? D. Carbamoilfosfatsintetase A. Biology, allergic E. Glucose-6-phosphate B. Smear, allergic dehydrogenase

125 Correct answers codes

Tests Code № Code № Code № Code № 1 B 38 B 75 B 112 E 2 C 39 E 76 A 113 E 3 E 40 B 77 E 114 E 4 Д 41 A 78 A 115 Д 5 A 42 A 79 B 116 B 6 Д 43 A 80 B 117 C 7 C 44 A 81 B 118 B 8 Д 45 C 82 B 119 B 9 Д 46 B 83 C 120 A 10 C 47 B 84 A 121 C 11 A 48 A 85 C 122 E 12 C 49 C 86 E 123 A 13 A 50 A 87 B 124 A 14 E 51 A 88 A 125 E 15 A 52 A 89 E 126 Д 16 A 53 E 90 E 127 A 17 B 54 E 91 A 128 A 18 E 55 Д 92 A 129 C 19 A 56 B 93 A 130 A 20 A 57 Д 94 C 131 C 21 A 58 E 95 A 132 E 22 A 59 A 96 A 133 C 23 E 60 B 97 C 134 A 24 A 61 A 98 Д 135 E 25 A 62 А 99 B 136 A 26 E 63 A 100 E 137 Д 27 Д 64 E 101 A 138 B 28 E 65 B 102 A 139 B 29 E 66 Д 103 B 140 A 30 E 67 A 104 C 141 B 31 E 68 Д 105 A 142 A 32 A 69 B 106 B 143 B 33 Д 70 А 107 E 144 B 34 B 71 C 108 A 145 Д 35 A 72 C 109 C 146 C 36 E 73 C 110 B 147 Д 37 A 74 A 111 B 148 A

126 № test Code № test Code № test Code № test Code task task task task 149 A 189 E 229 E 269 C 150 A 190 A 230 E 270 C 151 C 191 A 231 A 271 A 152 C 192 B 232 A 272 C 153 A 193 C 233 B 273 A 154 Д 194 A 234 C 274 Д 155 A 195 A 235 Д 275 E 156 A 196 E 236 B 276 A 157 E 197 A 237 A 277 B 158 A 198 Д 238 A 278 Д 159 C 199 Д 239 E 279 A 160 A 200 E 240 A 280 B 161 E 201 E 241 A 281 E 162 A 202 B 242 C 282 Д 163 A 203 A 243 B 283 E 164 C 204 Д 244 С 284 A 165 E 205 A 245 Е 285 C 166 B 206 B 246 E 286 Д 167 Д 207 A 247 Д 287 C 168 A 208 C 248 Е 288 Д 169 A 209 Д 249 А 289 B 170 A 210 A 250 E 290 B 171 Д 211 E 251 Д 291 C 172 E 212 E 252 A 292 B 173 E 213 Д 253 A 293 E 174 E 214 A 254 Д 294 E 175 B 215 A 255 A 295 Д 176 E 216 C 256 Д 296 C 177 C 217 A 257 A 297 A 178 А 218 E 258 B 298 A 179 A 219 Е 259 E 299 A 180 A 220 B 260 Д 300 A 181 C 221 Е 261 E 301 C 182 A 222 A 262 A 302 A 183 Д 223 В 263 A 303 C 184 A 224 А 264 C 304 B 185 Д 225 А 265 A 305 C 186 Д 226 A 266 A 306 B 187 B 227 A 267 Д 307 Д 188 A 228 A 268 A 308 В

127

Tests № Code № Code № Code № Code 309 Д 357 E 405 A 453 B 310 Д 358 B 406 A 454 A 311 B 359 A 407 E 455 A 312 B 360 C 408 Д 456 C 313 A 361 B 409 E 457 C 314 B 362 A 410 Д 458 Д 315 A 363 A 411 A 459 C 316 A 364 C 412 A 460 Д 317 Д 365 A 413 C 461 A 318 A 366 A 414 A 462 A 319 Д 367 A 415 A 463 A 320 Д 368 A 416 E 464 E 321 A 369 E 417 Д 465 A 322 E 370 A 418 C 466 B 323 A 371 B 419 B 467 A 324 A 372 A 420 B 468 A 325 C 373 A 421 E 469 B 326 C 374 E 422 B 470 Д 327 A 375 A 423 Д 471 Д 328 Д 376 Д 424 A 472 E 329 A 377 Д 425 Д 473 C 330 B 378 B 426 A 474 Д 331 B 379 E 427 B 475 C 332 A 380 A 428 E 476 E 333 E 381 C 429 C 477 E 334 Д 382 C 430 Д 478 C 335 A 383 E 431 C 479 A 336 A 384 A 432 Д 480 B 337 Д 385 C 433 A 481 E 338 Д 386 A 434 B 482 A 339 С 387 Д 435 B 483 A 340 E 388 A 436 C 484 C 341 E 389 Д 437 E 485 Д 342 A 390 A 438 A 486 Д 343 A 391 A 439 B 487 A 344 C 392 B 440 A 488 E 345 B 393 A 441 E 489 A 346 C 394 C 442 A 490 A 347 A 395 А 443 A 491 B 348 B 396 B 444 B 492 C 349 C 397 C 445 B 493 A 350 Д 398 A 446 Е 494 C 351 B 399 A 447 C 495 Д 352 A 400 B 448 A 496 A 353 B 401 C 449 E 497 E 354 A 402 C 450 Д 498 A 355 C 403 E 451 A 499 A 356 B 404 A 452 E 500 E

128

Tests Code № Code № Code № Code № 501 C 538 Д 575 А 612 С 502 Д 539 В 576 В 613 Е 503 E 540 Д 577 Д 614 Д 504 E 541 В 578 А 615 В 505 E 542 Е 579 Е 616 Е 506 B 543 В 580 А 617 Д 507 C 544 А 581 В 618 В 508 B 545 С 582 А 619 Д 509 A 546 С 583 Д 620 С 510 Д 547 Е 584 А 621 Е 511 A 548 Д 585 Е 622 А 512 C 549 С 586 В 623 Д 513 B 550 В 587 Е 624 Д 514 E 551 С 588 Д 625 С 515 A 552 А 589 В 626 А 516 E 553 В 590 С 627 В 517 E 554 А 591 А 628 Е 518 Д 555 Д 592 В 629 А 519 C 556 В 593 Е 630 Е 520 Д 557 А 594 Д 631 В 521 Д 558 Е 595 С 632 Е 522 Д 559 А 596 А 633 С 523 C 560 А 597 В 634 Д 524 C 561 А 598 С 635 Д 525 C 562 А 599 А 636 А 526 A 563 А 600 А 637 Д 527 B 564 Д 601 С 638 А 528 Д 565 А 602 С 639 Д 529 C 566 В 603 Е 640 В 530 B 567 С 604 А 641 Д 531 E 568 А 605 В 642 А 532 B 569 А 606 А 643 Д 533 B 570 А 607 С 644 А 534 A 571 С 608 В 645 С 535 A 572 В 609 Д 646 В 536 A 573 Е 610 С 647 А 537 A 574 В 611 В 648 А

129

Tests Code № Code № Code № Code № 649 Д 689 А 729 Е 769 Е 650 А 690 В 730 Д 770 В 651 А 691 С 731 С 771 Д 652 А 692 Д 732 А 772 Д 653 Е 693 Е 733 В 773 Е 654 А 694 Е 734 В 774 В 655 А 695 А 735 А 775 С 656 А 696 Д 736 Д 776 А 657 А 697 Е 737 В 777 С 658 А 698 С 738 С 778 В 659 А 699 В 739 А 779 А 660 Д 700 А 740 Е 780 Д 661 Д 701 В 741 А 781 Е 662 Е 702 В 742 С 782 Д 663 А 703 Д 743 С 783 А 664 С 704 А 744 А 784 Д 665 Е 705 Е 745 Д 785 С 666 Д 706 В 746 А 786 А 667 А 707 А 747 В 787 А 668 В 708 А 748 Д 788 А 669 А 709 В 749 Е 789 А 670 Е 710 А 750 С 790 В 671 А 711 В 751 В 791 А 672 В 712 С 752 В 792 А 673 Е 713 С 753 А 793 Д 674 А 714 Е 754 В 794 С 675 В 715 С 755 В 795 Д 676 А 716 Д 756 С 796 С 677 Е 717 Е 757 Е 797 В 678 Е 718 В 758 А 798 А 679 В 719 Е 759 С 799 Д 680 Е 720 Е 760 Е 800 В 681 В 721 В 761 А 801 Е 682 Е 722 Д 762 С 802 С 683 В 723 А 763 В 803 А 684 А 724 А 764 А 804 Е 685 В 725 А 765 А 805 С 686 Д 726 В 766 В 806 Д 687 А 727 С 767 Д 807 Д 688 А 728 Д 768 C 808 А

130 Tests Code № Code № Code № Code № 809 В 849 А 889 С 810 В 850 В 890 А 811 В 881 А 891 Д 812 С 882 А 892 С 813 А 883 А 814 А 884 В 815 В 885 А 816 А 886 С 817 Д 887 Е 818 С 858 А 819 С 859 А 820 А 860 А 821 А 861 А 822 В 862 В 823 С 863 Е 824 А 864 Д 825 Д 865 С 826 Д 866 А 827 Е 867 А 828 А 868 А 829 Д 869 А 830 А 870 Е 831 С 871 В 832 В 872 С 833 В 873 Е 834 Е 874 С 835 В 875 А 836 А 876 Д 837 В 877 А 838 А 878 А 839 Д 879 А 840 В 880 А 841 Д 881 С 842 В 882 В 843 А 883 А 844 В 884 А 845 А 885 А 846 А 886 Е 847 В 887 А 848 Д 888 Д

131